You are on page 1of 333

c

 c
  c
c c c
c
 c c
c

cc
c
 c
c

V  
 

V           
      
       V V   


            
! V   V  
      
      
     V             
 !  
 
   "     V
V   

   
# 
  
 V     

V                 #!   


        #!  $   $  V      
    

  V V  
V        #      %

&   V    
         '  
            #!   !  
'  V  % 
V  %     #  $     (    
! " !          V  %   '  !  
#   !          !
% V        !           
      (     "   
 "    
  
#   #
         
    $   $  V  
  !  !  V           
 !  #    
)
 *  +*       V     !
"
  V #  V    ,

-c%
 
.cV 
/c$ 
0c 
1c(  
2c/  

i  

i   
 
               
     

i    
            
 
      
      
 
    

  

  
   
   
   

  


         
  
     
    


      
  


  
    
     

   


 


   
    


      
     

  
 
    
 

 
    
 
    
      


    
    
            
    

 
      !      
     

   
  
   
   
      
       
  


" !       


   
    


                  

     

  
        
  
   !   

 
  

#i  
$
     

    
 
         %   


    &'' ('''
       


    

  
   

   
        

    
        
   
 )* 


 
           
  
   

   
       
     


  

+, 
   
 

+-
    


+           

+(    

+i      
   i   

+-   .
  

+'


  

+(    
 


+'   


+'
( /   
 


+     0     0  

+-
     


+'
 
     


+. 


+   i  


+0
   
 
     

+( /
 


+0 i  


+(    


+    
    
 
   


+& '   0 





Introduction

Coordinate Geometry is the unification of algebra and geometry in which algebra is


used in the study of geometrical relations and geometrical figures are represented by
means of equations. The most popular coordinate system is the rectangular Cartesian
system. Coordinates of a point are the real variables associated in an order to describe
its location in space. Here we consider the space to be two-dimensional. Through a point
O, referred to as the origin, we take two mutually perpendicular lines XOX and YOY
and call them x and y axes respectively. The position of a point is completely
determined with reference to these axes of means of an ordered pair of real numbers (x,
y) called the coordinates of P where |x| and |y| are the distances of the point P from the
y-axis and the x-axis respectively. X is called the x-coordinate or the abscissa of P and y
is called the y-coordinate or the ordinate of P.
Aepresentation of points in a plane
We are familiar with the representation of real numbers on a line, which we call a real
line. In this representation we fix a point O (called origin) and represent a real number
by a point A on this line such that its distance OA (see figure given below) is equal to
the value of real number. In the left side of O we represent negative real numbers and
in the right side of O we represent positive real numbers. Thus, not only the magnitude
of OA but the direction of the line OA is also considered for representation.

] ]
Hence OA = AO

Similarly ordered pairs are represented in a plane. To represent an ordered pair (a, b)
we take two reference lines which are mutually perpendicular. The ordered pair (a, b)
represents in such a plane, by a point P(a, b) such that (see figure given below) OA = a
and OB = b.

This system is called Cartesian co-ordinate system. Since elements of an ordered pair
are not inter changeable (i.e., (a, b) (b, a) unless a = b) so they are represented in
particular order, the first element a is represented on horizontal line called abscissa
and the second element b on a vertical line called ordinate. Like the real number
notation the positive side of the x-axis is the right side of O and positive side of O and
positive side of y-axis is upper side of O.
So, the two lines divide the region in 4 parts. These are called quadrants. These
quadrants are characterized as

I quadrant x > 0, y > 0

II quadrant x < 0, y > 0

III quadrant x < 0, y < 0

IV quadrant x > 0, y < 0

Here the point O represents x = 0 and y = 0, hence order pair becomes (0, 0).

There is a second type of representation called the polar co-ordinate system. In this
system a reference is fixed to a line (Called the initial line), and a point called the origin
in the system. Any point P is represented by ordered pair (r, ).

Such that

OP = r; The distance of point from origin.

and pPOX = The angular displacement of line OP from fixed line i.e. the initial
line, (in the anticlockwise direction)

Clearly a = r cos and b = r sin (see figure given below)


cc

Distance between two points


cc

The distance between two points P(x1, y1) and Q(x2, y2) is (see the figure
given below).

Length PQ = (x2 x1)2 + (y2 y1)2

Proof:

Let P(x1, y1) and Q(x2, y2) be the two points and let the distance between
them be d. Draw PA, QA parallel to y-axis and PA parallel to x-axis.

Angle QAP = 90o


d2 = PA2 + AQ2

d2 = (x2 x1)2 + (y2 y1)2

d = (x2 x1)2 + (y2 y1)2.

Section Formula

Let us say we want to know the co-ordinates of point which divides a line
segment between two points A(x1, y1) and B(x2, y2) in the ratio m : n.

The coordinates of such a point are given by

(nx1 + mx2/m+n, ny1 + my2/m+n) (for internal division)

Note:

This is called section formula.

Let P divide the line segment AB in the ratio m : n. If P is inside AB then


it is called internal division; if it is outside AB then it is called external
division.

However in each case AP/BP [or AP'/BP' or AP"/BP"] = m/n.

Proof:

Consider ? ABB

Since BB||PQ and AP:PB = m:n (see figure given below)


AQ/AB' = PQ/BB' = m/m+n (= AP/AB)

x x1/x2 x1 = m/m+n

x = nx1 + mx2/m+n and y = ny1 + my2/m+n

If P is outside AB (less assume it is at P)

We have

x x1/x2 x1 = m/m+n

x = nx1 + mx2/m+n and y = ny1 + my2/m+n

Similarly if P is at P then

x = mx2+m+n/nm, y = my2+ny1/m+n

Note:

m:n can be written as m/n or :1. So any point on line joining A and B
will be P(x2+x1/+1.y2+y1/+1). It is useful to assume :1 because it
involves only one variable.

illustration:

Find the ratio in which line segment A(2, 1) and B(5, 2) is divided by x-
axis.
Solution:

Let x-axis intersect line at point P(xp, 0) such that AP/BP = /1


yP = 0 = y2+1.ya/+1 = 2+(1)/+1 = 1/2
AP/BO = 1/2

Illustration:

Prove that altitudes of a triangle are concurrent and prove that the co-
ordinates of the point of con-currency are

(x1 tan A + x2 tan B + x3 tan C/tan A + tan B + tan C, y1 tan A + y2 tan B


+ y3 tan C/tan A + tan B + tan C),

Solution:

In triangle A(x1, y1), B(x2, y2) and C(x3, y3), draw AD perpendicular to BC.
Our effort now should be to find the co-ordinates of the point D.

To do that, we need to find BC/CD. (figure is given above)

tan B = AD/BD and tan C = AD/CD

BD/DC = tan C/tan B

Now we apply section formulae.


xD = x2 tan B + x3 tan C/tan B + tan C
(i)

yD = y2 tan B + y3 tan C/tan B + tan C


(ii)

We know that orthocenter will lie on AD. We need to find this point and
its co-ordinates.

We should select a point H1 on AD and take the ratio AH1/H1D in such a


manner so that xH1 and yH1 calculated form (i) should be symmetric in x1,
x2, x3, tan A, tan B and tan C. Think before you proceed

Let AH1/H1D = tan B + tan C/tan A

xH1 = x1 tan A + x2 tan B + x3 tan C/tan A + tan B + tan


C

and yH1 = y1 tan A + y2 tan B + y3 tan C/tan A + tan B + tan C

Since the result is symmetric, this point H1 will lie on other altitude as
well i.e. the altitudes are concurrent

xH = xH1 and yH = yH1

Illustration:

Prove analytically that in a right angled triangle the midpoint of the


hypotenuse is equidistant from the three angular points.

Solution:

While proving a problem analytically take most convenient co-ordinates


of known points.

In the present case triangle is assumed as AOB with coordinates as


shown in figure given below, C is midpoint of AB.
So co-ordinates of C will be (a/2, b/2)

Now AB = a2 + b2
CA = CB = AB/2 (C is mid point of AB)

= a2 + b2

and, we know that the distance between two points C and O is given by

CO = (a/2 0)2 + (b/2 0)2 = a2 + b2/2

Hence CA = CB = CO

Coordinates of the point P dividing the join of two points A(x1, y1) and

B(x2, y2) internally in the given ratio 1 : 2 i.e., AP/BP


= 1/2 areP(2x1+1x2/2+1, 2y1+1y2/2+1).

Coordinates of the point P dividing the join of two points A(x1, y1) and
B(x2, y2) externally in the ratio 1 : 2 i.e., Ao/BP = 1/2 areP(2x1+1x2/2
1, 2y1+1y2/21).

Centroid of Triangle
c
The centroid of a triangle is the point of concurrency of the medians. The centroid G of
the triangle ABC, divides the median AD, in the ratio of 2 : 1.

Illustration:
Find the centroid of the triangle the coordinates of whose vertices are given by A(x1,
y1), B(x2, y2) and C(x3, y3) respectively.

Solution:

AG/AD = 2/1

Since D is the midpoint of BC, coordinates of D are (x2+x3/2, y2+y3/2)

Using the section formula, the coordinates of G are

(2(x2+x3/2)+1.x1/2+1, 2(y2+y3/2)+1.y1/2+1)

Coordinates of G are (x1+x2+x3/3, y1+y2+y3/3).

Incentre of Triangle

The incentre I of a triangle is the point of concurrency of the bisectors of the angles of
the triangle.

Illustration:

Find the incentre of the triangle the coordinates of whose vertices are given by A(x1,
y1), B(x2, y2), C(x3, y3).
Solution:

By geometry, we know that BD/DC = AB/AC (since AD bisects A).

If the lengths of the sides AB, BC and AC are c, a and b respectively, then BD/DC =
AB/AC = c/b.

Coordinates of D are (bx2+cx3/b+c, by2+cy3/b+c)

IB bisects B. Hence ID/IA = BD/BA = (ac/b+c)/c = a/c+b.



Let the coordinates of I be (x, y).

Then x = ax1+bx2+cx3/a+b+c, y = ay1+by2+cy3/a+b+c.

Circum Centre of Triangle

This the point of concurrency of the perpendicular bisectors of the sides of the triangle.
This is also the centre of the circle, passing through the vertices of the given triangle.

Orthocentre of Triangle

This is the point of concurrency of the altitudes of the triangle.

Excentre

Excentre of a triangle is the point of concurrency of bisectors of two exterior and third
interior angle. Hence there are three excentres I1, I2 and I3 opposite to three vertices of a
triangle.

If A(x1, y1), B(x2, y2) and C(x3, y3) are the vertices of a triangle ABC,
coordinates of centre of ex-circle opposite to vertex A are given as

I1(x, y) = (ax1+bx2+cx3/a+b+c/a+b+c, ay1+by2+cy3/a+b+c).

Similarly co-ordinates of centre of I2(x, y) and I3(x, y) are

I2(x, y) = (ax1bx2+cx3/ab+c, ay1by2+cy3/ab+c)

I3(x, y) = (ax1+bx2cx3/a+bc, ay1+by2cy3/a+bc)

Area of a triangle
c
Let (x1, y1), (x2, y2) and (x3, y3) respectively be the coordinates of the
vertices A, B, C of a triangle ABC. Then the area of triangle ABC, is

|1/2[x1(y2 y3) + x2(y3 + y1) + x3(y1 y2)]| = .


It follows that the three points (x1, y1), (x2, y2) and (x3, y3) will be collinear

if = 0.

Area of a polygon of n sides

First of all we plot the points and see their actual order. Let A1(x1, y1),
A(x2, y2), , An(xn, yn) be the vertices of the polygon in anticlockwise
order.

Then area of the polygon =

Illustration:

Calculate area of a triangle shown in figures given below.


Solution:

Using the just derived formula

Area of a triangle

1/2 [3(2 4) + (1) (4 6) + 5 (6 2)]

= 1/2 [ 6 2 + 20] = 6

Similarly, area of the triangle shown in the figure given above.

Area of a ?ABC is

= 1/2 [3(4 2) + (5) (2 6) + (1) (6 4)]

=1/2 [6 20 + 2] = 6

Caution:
Thus we observe that the area of a triangle is positive vertices are taken
in the anticlockwise direction and negative when the vertices are taken
the clockwise direction.

Note:

Area of a triangle can also be expressed as = 1/2

= 1/2 [x1 y2 y1 x2 + x2 y3 y2 x3 + x3 y1 y3 x1]

This form is important. It can be used to find area of a quadrilateral,


pentagon, hexagon and polygons.

Important:

If three points P1, P2 and P3 are collinear then the determinant at must
vanish i.e. the area of triangle formed must be zero.

Note:

If the vertices are in clockwise order then take modulus.

Illustration:
Prove that the area of the triangle with vertices at (p 4, p + 5),
(p + 3, p 2) and (p, p) remains constant as p varies.

Solution:

The area of the triangle is

which remains constant for all values of p.

Straight Line
c
Any equation of first degree of the form Ax + By + C = 0, where A, B, C
are constants always represents a straight line (at least one out of A and
B is non zero).

Slope
If is the angle at which a straight line is inclined to the positive
direction of the x-axis, then m = tan, (0 < < 180o) is the slope of the line.

Standard equations of the Straight Line


Slope Intercept From:

y = mx + c, where

m = slope of the line


c = y intercept

Intercept Form:
x/a + y/b = 1
x intercept = a
y intercept = b

Slope point form

(a) One point on the straight line

(b) The gradient of the straight line i.e., the slope m of the line

Equation:

y y1 = m(x x1), where (x1, y1) is a point on the straight line.

Illustration:
Pause:
Equation of line in figure (ii) is x = 3, because x-co-ordinate of each point
on the line is 3.
Equation of line in figure (iv) is y = 2, because y-co-ordinate of each point
on the line is 2.

Although every line satisfied the above given basic definition, a line can
be represented in many forms, some of which are given hereunder.

Two points form:


Let there be two points A(x1, y1) and B(x2, y2) in a co-ordinate plane. If
any point P(x, y) lies on the line joining A and b then m = tan = yy1/x
x2 = y2 y1/x2 x1, (see figure given below).
y y1 = y2 y1/x2 x1 (x x1) which is the equation of the given line.
Equation of line can also be written as

y y2 = y2 y1/x2 x1 (x x2) or = 0.

Different Forms of line


c
c
c
Illustration:

The ends of a rod of length l move on two mutually perpendicular lines. Find the locus
of the point on the rod, which divides it in the ratio 2 : 1.

Solution:

Suppose the two perpendicular lines are x = 0 and y = 0 and let the end of the rod lie at
the point (0, a) and (b, 0).

The point P has coordinates given by

h = b+2.0/2+1, k = 2.a+1.0/2+1 a = 3k/2, b = 3h.

Also l2 = a2 + b2.

l2 = (3k/2)2 + (3h)2.
Thus the required locus is

x2 + y2/4 = l2/9, which represents an ellipse.

Similarly, take the ratio AP : BP as 2 : 1 and proceed. We get the result as y2 + x2/4 =
l2/9.

Illustration:

Find the locus of the point of intersection of the liens xcos; + ysin; = a and xsin;
ycos; = b. where ; is a variable.

Solution:

Let P(h, k) be the point of intersection of the given lines.

Then hcos; + ksin; = a. (1)

hsin; kcos; = b. (2)

Here a is a variable. So we have to eliminate a.

Squaring and adding (1) and (2),

We get, (hcos; + ksin;)2 + (hsin; kcos;)2 = a2 + b2

h2 + k2 = a2 + b2.

Hence locus of (h, k) is x2 + y2 = a2 + b2.

Two intercept form:

If intercepts of a line on x and y-axis are known then equation of the line can also be
found in two-intercept form. Intercepts are OA and OB on x and y-axis respectively,
where A(a, 0) and B(0, b) are two points through which line is passing. Treating it as a
special case of two-point form, one can write a unique equation of the line as

y0/xa = 0b/a0, where P(x, y) is any point on the line (figure given below)
y/b = x/a + 1.

x/a + y/b = 1.

This is intercept from of the equation of a straight line.

Parametric form:

Consider line PQ with points Q(x1, y1). Then Co-ordinates of any points P(x, y) are

x = x1 + r cos (see figure given below)

y = y1 + r sin

Equation of the line is obtained as follows:-

xx1/cos = yy1/sin = r

This is parametric form of the equation of a straight line.

Note:

That tan = m = slope of line.


Normal form:

Consider line l as shown in figure given below

ON R l and |ON| = p

We have in ?ONA

OA = p/cos ;

A and B are intercept points of line l. So intercepts on x and


y-axes are p/cos ; and p/sin ; respectively. So equation of line l will be (both intercept
form)

x cos ;/p + y sin ;/p = 1

x cos ; + y sin ; = p

This is the equation of a straight line in normal form, where p is perpendicular distance
of the line from origin.

Caution:

tan a m (slope of line)

Note:

P is always measured away from the origin and is always positive in value, ;a is a
positive angle less than 360o measured from the positive direction OX of the x-axis to
the normal from the origin to the line.

General form:

The general form of the equation of a straight line is


Ax + By+ C = 0

This equation can be reduced to any of the above forms with some rearrangements.

Examples based on straight line


Illustration:

Aeduce 3x 4y + 5 = 0 to all other forms.

Solution:
(a) Slope intercept form

y = 3/4 x + 5/4
where, m = slope = 3/4
c = 5/4 (y intercept)

(b) Intercept form


3x 4y = 5
3x/5 + 4y/5 = 1
x/5/3 + y/5/4 = 1 (intercept form)
x intercept = 5/3
y intercept = 5/4

(c) point slope form

Let x = 1, then y = 3/4 + 5/4 = 2


y 2 = 3/4 (x 1)

(d) parametric form

x1/cos = r, where tan = 3/4


x1/4/5 = y2/3/5 = r

(e) Normal form

3x 4y = 5
3x + 4y = 5
Dividing by ((3)2 + 4)
3/5 x + 4/5 y = 5/5 = 1
x cos ; + y sin ; = p, p > 0
Where cos ; = 3/5, sin ; = 4/5, p = 1

Illustration:
The straight line drawn through the point P(0, 3) and making an angle of
30o with positive x-axis, meets the line x + y = 6 at Q. Find the length PQ.

Solution:

Method 1.
Equation of the line through the point P is
x0/cos 30 = y 3/sin 30 = r
xQ = 3/r r, yQ = 3 + r/2, r = distance PQ
Point Q lies on x + y = 6

r3/2 + (3 + r/2) = 6
r = 6/3+1

Method 2.
Equation of the line through the point P is y = 1/3 x + 3
[because here m = tan 30o = 1/3, c = 3]

Solving this line, with x + y = 6, we get

xQ = 33/3+1, yQ = 33+6/3+1
distance PQ = (xQ xP)2 + (yQ yP)2

= 6/3+1

Illustration:
Find the equation of the line whose perpendicular distance from the
origin is 4 units and the angle which the normal makes with positive
direction of x-axis is 15o.

Solution:
Here, we are given p = 4 and = 15o.
Now cos 15o = 3+1/22
and sin 15o = 31/22
By the normal form, the equation of the line is

x cos 15o + y sin 15o = 4 or 3+1/22 x + 31/22 y = 4


or (3 + 1) x + (3 1)y = 82.

This is the required equation.

Illustration:
Given a line 2x 3y + 5 = 0. Write various forms of the line.

Solution:

Slope intercept form:


y = 2x/3 + 5/3, C = 5/3

and m = (coeffcient of x/coefficient of y) = 2/3


Intercept Form:
x/(5/3) + y/(5/3) = 1, a = 5/2; b = + 5/3.

Normal Form:
sin ; = 3/13, cos ; = 213
2/13 p = 5/13.

Illustration:

Find the equation to the straight line which passes through the point (5,
4) and is such that the position of it between the axes is divided by the
give point in the ratio 1 : 2.

Solution:
Let the required straight line be (x/a) + (y/b) = 1.
Using the given conditions, P (2a+1.0/2+1, 2.0+1.b/2+1) is the point which
divides (a, 0) and (0, b) internally in the ratio 1 : 2.

But P is (5/4)
Hence 5 = 2a/3, 4 = b/3 a = 15/2, b = 12.

Hence the required equation is x/(15/2) + y/2 < < .


tan = 8 = slope of line.
We know that the equation of the straight line passing through the point
(x1, y1) having slope m is y y1 = m(x x1).
Therefore the equation of the required line is y 2 = 8 (x 1)
8 x + y 8 2 = 0.

Illustration:
Find the equation of the line joining the points (1, 3) and (4, 2).

Solution:
Equation of the line passing through the points (x1, y1) and (x2, y2) is
y y1 = y1y2/x1x2 (x x1)
Hence equation of the required line will be
y 3 = 3+2/14 (x + 1) x + y 2 = 0.

Illustration:
Aepresent the straight-line y = x + 2 in the parametric form.

Solution:
Slope of the given line is = 1 = tan /4.
Equation of the straight line can be written as y 2 = x.
or y2/1/2 = x/1/2= r.
Any point on the line is (r/2, 2 + r/2).
The point (x, y) is at a distance r from the point (0, 2).

Illustration:
A line joining two points A(2, 0) and B(3, 1) is rotated about A in the
anticlockwise direction through an angle of 15o. Find the equation of the
line in the new position. If B goes to C, what will be the coordinates of C,
in the new position?

Solution:
Slope of BAB(m) = 1 m = tan = 1 = 45o.
= tan (60o) (because angle between AB and AC = 15o).
Also AB = AC = 2 and A is (2, 0).

Hence equation of the line AC is


x2/cos60o = y0/sin 60o

or x2/1/2 = y/3/2 = r = 2
C is (2 + 2.1/2, 0 + 2.3/2) i.e. C is (2 + 1/2, 3/2).

Angle between two straight lines


Angle bisector of two lines (the line which bisects the angle between the
two lines) is the locus of a point which is equidistant (having equal
perpendicular distance) from the two lines.

Say, we have two lines


L1 : A1x + B1y + C1 = 0
L2 : A2x + B2y + C2 = 0

If point A(p, q) lies on the bisector, then length of perpendicular from P


to be both lines should be equal.

i.e.

generalizing for any point (x, y) the equation of the angle bisector is
obtained as:
A1x+B1y+C1/A12 + B12 = + A2x+B2y+C2/A22 + B22
Note:

1. This equation gives two bisectors: one-acute angle bisector and the
other obtuse bisector.

2. Aule to decide the particular bisector

To determine a bisector which lies in the same relative position with


respect to the lines as a given point S(x3, y3) does, make the signs of the
expressions A2x3 + B1y3+ C1 and A2x3 + B2y3 + C2 identical. (say positive)
then A1x+B1y+C1/A12 + B12 = +A2x+B2y+C2/A22 + B22 gives the bisector
towards this point. If the signs are different multiply one of the
equations with 1 throughout, so that positive sign is obtained. Then
above equation with changed equations of lines will given the required
bisector.

3. If (x3, y3) (0, 0) and A2A1 + B2B1 > 0 then the bisector towards the
origin is the obtuse angle bisector.

4. Alternative method: to determine whether the bisector is of the obtuse


or acute angle, determines both the bisectors and calculate angle
between one of them and the initial line. The bisector for which |tan | > 1
is the obtuse angle bisector.

cLocus

If a point moves according to some fixed rule, its co-ordinates will


always satisfy some algebraic relation corresponding to the fixed rule.
The resulting path (a curve) of the moving point is called the locus of the
point. The locus i.e. the curve now contains all the points satisfying the
specified condition and no point outside the curve satisfies the condition.
When a point moves in a plane under certain geometrical conditions, the
point traces out a path. This path of the moving point is called its locus.

Equation of Locus
The equation to the locus is the relation which exists between the
coordinates of all the point on the path, and which holds for no other
points except those lying on the path.

Procedure for finding the equation of the locus of a point

(i) If we are finding the equation of the locus of a point P, assign


coordinates (h, k) to P.

(ii) Express the given conditions as equations in terms of the known


quantities and unknown parameters.

(iii) Eliminate the parameters, so that the eliminant contains only h, k


and known quantities.

(iv) Aeplace h by x, and k by y, in the eliminant. The resulting equation


is the equation of the locus of p.

The problem of determining the equation of locus of points every pair of


which has constant slope. (see figure given below)
Slope is the tangent (i.e. tan q) of the angle made by a line with the
positive x-axis (remember positive) taken in anticlockwise direction from
x-axis to the line. For any two points P(x1, y1) and Q(x2, y2).
cc

Examples on Angle Between two straight


lines.
Illustration:

Draw the lines 3x + 4y 12 = 0 and 5x + 12y + 13 = 0. Find the equation of the bisector of
the angle containing the origin. Also find the acute angle bisector and obtuse angle
bisector.

Solution:
Let us make the expression on the left-hand side of the given equations of the same sign
or + ve. After substituting x = 0 and
y = 0.

L.H.S. of (i) is 3.0 + 4.0 12 = 12 = ve

A.H.S. of (ii) is 5.0 + 12.0 + 13 = 13 = + ve

So, multiply equation (i) by (1), we get

3x 4y + 12 = 0 (1)

Equation of the bisector of the angle containing origin is given by +ve sign i.e. 3x
4y+12/5 = + 5x+12y+13/13

64x + 112y 91 = 0 (3)

Again, the given lines are


3x 4y + 12 = 0 (1)

5x + 12y + 13 = 0 (2)

To find out whether this is an acute angle bisector or obtuse angle bisector, let us find
the sign of a1 a2 + b1 b2 from equation (1) and equation (2).

a1 a2 + b1 b2

= (3) (5) + (4) (12) = 15 48 = 63 = ve

the bisector containing the origin is the acute angle bisector.

Now, For obtuse angle bisector, we take ve origin.

i.e. 3x 4y+12/5 = + 5x+12y+13/13

i.e. 14x 8y 221 = 0 (4)

Well, to confirm all this, let us find angle between one of the lines and one of the
bisectors i.e.

5x + 12y + 13 = 0 (2)

64x + 112y 91 = 0 (3)

Slope of line (2) is m2 = 5/12

Slope of line (3) is m3 = 64/112

Let q be the angle between these two lines

tan = <1

64x + 112y 91 = 0 is an acute angle bisector.

If is the angle between two lines, then tan =


where m1 and m2 are the slopes of the two lines.

(i) If the two lines are perpendicular to each other then m1m2 = 1.

Any line perpendicular to ax + by + c = 0 is of the form


bx ay + k = 0.

(ii) If the two lines are parallel or are coincident, then m1 = m2.

Any line parallel to ax + by + c=0 is of the form ax ay + k=0.

Let there be two-lines l1 and l2 with slopes m1 and m2 respectively. So tan ; = m1, tan =
m2 Angle between them is either

; or (; ) depending on the side on considers


Now, tan (a b) = tan ; tan /1+tan ; tan

tan () = m1+m2/1+m1m2 (; = say)

Since lines can be taken in any order and

tan( ) = tan . So only the magnitude of can be obtained.

Further tan ( ) = tan .

Since magnitude also includes the other angle i.e.

Supplementary angle. So is given by

tan acute =

Important:

1. If lines are parallel

tan = 0 m1 = m2

2. If lines are perpendicular

tan = tan (/2) =

1 + m1 m2 = 0 m1 m2 = 1

3. Equation of a line parallel to y = mx + c is y = mx + k, i.e. Equation of a line


parallel to ax + by + c = 0 is ax + by + k = 0

4. Equation of a line perpendicular to y = mx + c is y = 1/m x + k i.e. Equation of a


line perpendicular to ax + by + c = 0 is
bx ay + k = 0

5. Lines a1x + b1y + c1 = 0 (i)

a2x + b2y + c2 = 0 (ii)

represents

(i) intersecting lines if a1/a2 b1/b2


(ii) parallel lines if a1/a2 = b1/b2

(iii) Coincident lines if a1/a2 = b1/b2 = c1/c2

Length of the Perpendicular from a Point


on a Line
cc
The distance of a point from a line is the length of the perpendicular drawn from the
point on the line. Given the equation of the line are different forms, the length of the
perpendicular can be obtained in different forms.

First form:

The normal equation helps us in finding the distance of a point from a straight line.
Suppose we have to find the distance of the point P(x1, y1) from the line l1 whose
equation is x cos ; + y sin ; = p. Let l2 be the line through P parallel to the line l1. Let d
be the distance of P from l1. Then, the normal from O to l2 is of length p + d. Hence the
equation of l2 is

x cos ; + y sin ; = p + d.

Since P(x1, y1) lies on it.

x1 cos ; + y1 sin ; = p + d

d = x1 cos ; + y1 sin ; p.

Note:

1. Aule to find the perpendicular distance of a given point from a given line in
normal form.

In the left side of the equation (right side being zero), substitute the coordinates of
the point. The result gives the perpendicular distance.
2. Complete distance formula. If the point P and the origin O, instead of lying on the
opposite sides of l as in figure given above, lie on the same side of line l1 it may be
proved by proceeding exactly in the same manner that

d = (x1 cos ; + y1 sin ; p)

Hence, the complete distance formula is

d = + (x1 cos ; + y1 sin ; p)

The distance of a point from a line is the length of the perpendicular drawn from the
point to the line. Let L : Ax + By + C = 0 be a line, whose distance from the point P(x1, y1)
is d. Draw a perpendicular PM from the point P to the line L. If the line meets the x
and y-axes at the points Q and A respectively, then coordinates of the points are Q(
C/A, 0) and A(0, C/B). Thus the area of the triangle PQA is given by

area(?PQA) = 1/2 PM QA, which gives PM = 2 are (PQA)/QA (1)

also, area (?PQA) = 1/2 |x1 (0 + C/B) + (C/A) (C/B y1) + 0(y1 0)|
= 1/2 |x1 C/B + y1 C/A + C2/AB|

Or 2 area (?PQA) = |C/AB| |Ax1 + By1 + C| and

QA = (0 + C/A)2 + (C/B 0)2 = |C/AB| A2 + B2

Substituting the values of area (?PQA) and QA in (1), we get

PM = |Ax1+By1+C|/A2+B2.

Or d = |Ax1+By1+C|/A2+B2.

Thus the perpendicular distance (d) of a line Ax + By + C = 0 from a point

(x1, y1) is given by d = |Ax1+By1+C|/A2+B2.

Distance between two parallel lines

We know that slopes of two parallel lines are equal. Therefore, two parallel lines can be
taken in the form

y = mx + c1 (1)

and y = mx + c2 (2)

Line (1) will intersect x-axis at the point A (c1/m, 0) as shown in figure.
Distance between two lines is equal to the length of the perpendicular from point A to
line (2). Therefore, distance between the lines (1) and 92) is

|(m)(c1/m)+(c2)|/1+m2 or d = |c1c2|/1+m2.

Thus the distance d between two parallel liens y = mx + c1 and y + mx + c2 is given by d


= |C1C2|/A2+B2.

Illustration:

Find the distance of the point (3, 5) from the line 3x 4y 26 = 0.

Solution:

Given line is 3x 4y 26 = 0. (1)

Comparing (1) with general equation of line Ax + By + C = 0, we get

A = 3, B = 4 and C = 26.

Given point is (x1, y1) = (3, 5). The distance of the given point from given line is d =
|Ax1+By1+C|/A2+B2 = 3/5.

Illustration:

Find the distance between the parallel lines 3x 4y + 7 = 0 and


3x 4y + 5 = 0.
Solution:

Here A = 3, B = 4, C1 = 7 and C2 = 5.

Therefore, the required distance is d = |75|/32+(4)2 = 2/5.

Illustration:

The coordinates of the vertices A, B, C of a triangle are (6, 3), (3, 5) and (4,2)
respectively and P is any point (x, y). Show that the ratio of the areas of the triangles
PBC and ABC is |x + y 2| : 7.

Solution:

Equation of the line BC is x + y 2 = 0. Let PG and AD be perpendiculars from p and A


on BC.

Aatio of areas of triangles PBC and ABC is

1/2.BC.PG/1/2.BC.AD = PG/AD. But PG is the length of the perpendicular form P(x, y)


on
x + y 2 = 0 and AD is the length of the perpendicular from A(6, 3) on x + y 2 = 0.

Aatio of the areas of ?PBC and ?ABC.

|x+y2/2|/|6+32/2| = |x+y2/7|.

Illustration:
Find the distance from the line 3x 4y + 35 = 0 of the point (0, 0).

Solution:
Writing the given equation in the normal form, we get
3/5 x + 4/5 y 7 = 0

Substituting x = 0, y = 0 in it, d = 3/5 (0) + 4/5 (0) 7 = 7

Changing sign the required distance = 7.

Second form:

To find the perpendicular distance of the point (x1, y1) from the line
ax + by + c = 0.

Let us convert the given equation in the normal form.

a/a2+b2 x + b/a2+b2 y + c/a2+b2 = 0

The perpendicular Distance of (x1, y1) is

d = a/a2+b2 x1 + b/a2+b2 y1 + c/a2+b2

d = ax1+by1+c/a2+b2

This formula can also be obtained independently as under:

Let PM be the perpendicular form P on AB. Then coordinates of A and B are

(c/a, 0) and (0, c/b) respectively,

AB = c2/a2+c2/b2

= c/ab (a2+b2)

Area of ?PAB (Aecall from page M4 M 4)

= 1/2 [x1 (c/b 0)+0(0 y1) c/a (y1 + c/b)]

= c/2ab (ax1 + by1 + c) (1)


Also, area of ?PAB = PM.AB

= 1/2 PM.c/ab (a2+b2) (2)

From (1) and (2), we have

1/2 PM.c/ab (a2+b2) (ax1 + by1 + c)

PM = ax1+by1+c/a2+b2

Neglecting the negative sign, as the length of a segment is always positive, we have

PM = |ax1+by1+c/a2+b2|

c
Note:

1. Actually
d = + ax1+by1+c/a2+b2

2. To find the distance of a point from the given line, in the left side of the equation
(right side being zero) substitute the co-ordinates of the point, and divide the result
by (coefficient of x)2 + (coefficient of y)2

3. When to use complete perpendicular Distance formula? The complete


perpendicular Distance formula is used when the length of the perpendicular to the line
is given.

Illustration:
Find the distance of the point P(2, 3) from the line AB which is
x y = 5.

Solution:
The equation of the line is

x y 5 = 0 [Making right side zero (note this step)]

Perpendicular Distance of the point (2, 3)

= (2)35/(1)2+(1)2 = 10/2

= 52

Changing the sign, perpendicular Distance in magnitude = 52.

Enquiry: We can now find the distance of a point form a line but how can we
determine as to which side of the line does the point lie?

From the figure, we observe that

ax0 + by0 + c = 0 (? point (x0, y0) lies on the line) (1)

Consider, ax1 + by1 + c

= (ax0 + c) + by1 [x0 = x1 = x2]

= b(y1 y0)

= ve
Consider, ax2 + by2 + c

= (ax0 + c) + by2

= b(y2 y0)

= +ve

Thus we observe that the point is on one side of the line, if put in the expression of line
is gives one sign, while the point is on the other side of the line, if put in the expression
of line it gives opposite sign.

Illustration:

Final the condition so that the points (x1, y1) and (x2, y2) lie on the same side, of the line
ax + by + c =

Solution:
Since, (ax1 + by1 + c) and (ax2 + by2 + c)

Should be of the same sign.

Their product should be positive i.e.

(ax1+by1+c)(ax2+by2+c) > 0, which is the required condition.

Family of lines
cc
The general equation of the family of lines through the point of intersection of two
given lines is L + L = 0, where L = 0 and L = 0 are the two given lines, and is a
parameter.

Illustration:

A variable line through the point of intersection of the lines


x/a + y/b = 1 and x/b + y/a = 1 meets the coordinate axes in A and B. Show that the
locus of the midpoint of AB is the curve 2xy(a + b) = ab(x + y).

Solution:
Let (h, k) be the midpoint of the variable line AB.

The equation of the variable line AB is

(bx + ay ab) + (ax + by ab) = 0

Coordinates of A are (ab(1+)b+a).

Coordinate of B are (0, ab(1+)b+a).

Mid point of AB is (ab(1+)2(b+a), ab(1+)2(a+b))

h = ab(1+)2(b+a); k = ab(1+)2(a+b) 1/2h = b+a/ab(1+); 1/2k =


a+b/ab(1+)

1/2h + 1/2k = a+b/ab (h + k)ab = 2hk (a + b).

Hence the locus of the midpoint of AB is (x + y) ab = 2xy (a + b).

To find the equation to the straight lines which pass through a given point (x1, y1) and
make equal angles with the given straight line y = m1x + c.

If m is the slope of the required line and ; is the angle which this line makes with the
given line, then tana = + m1m/1+m1m.
(i) The above expression for tan;, gives two values of m, say mA and mB.

(ii) The required equations of the lines through the point (x1, y1) and making equal
angles ; with the given line are y y1 = mA (x x1), y y1 = mB(x x1).

Illustration:

Find the equations to the sides of an isosceles right-angled triangle, the equation of
whose hypotenuse is 3x + 4y = 4 and the opposite vertex is the point (2, 2).

Solution:

The problem can be restated as:

Find the equations to the straight lines passing through the given point (2, 2) and
making equal angles of 45o with the given straight line 3x + 4y 4 = 0

Slope of the line 3x + 4y 4 = 0 is m1 = 3/4


tan 45o = + m1m/1+m1m, i.e., 1 = + m+3/4/13/4m

So that mA = 1/7, and mB = 7.

Hence the required equations of the two lines are

y 2 = mA (x 2) and y 2 = mB(x 2).

7y x 12 = 0 and 7x + y = 16.

Illustration:

The straight lines 3x + 4y = 5 and 4x 3y = 15 intersect at the point A. On these lines


points B and C are chosen so that AB = AC. Find the possible equations of the line BC
passing through (1, 2).

Solution:

The two given straight lines are at right angles.

Since AB = AC, the triangle is an isosceles right angled triangle.

The required equation is of the form y 2 = m(x 1) (1)

with tan 45o = + m+3/4/13m/4 = + m4/3/1+4m/3

1 = + m+3/4/13m/4 = + m4/3/1+4m/3 m = 7, 1/7.

Substitute the value of m in (1). We get the required equations.

Illustration:

Find the equation of the straight line passing through (2, 7) and having intercept of
length 3 units between the straight lines
4x + 3y = 12 and 4x + 3y = 3.

Solution:

Distance AC between the two given parallel lines


= |c1c2/a2+b2| = 123/16+9 = 9/5.

Let AB be the intercept of length 3 units.

BC = 12/5. If is the angle between BC and AB, then tan = 9/12 = 3/4.

Slope of the parallel lines = 4/3 = m2.

If m1 is the slope of the required line, then tan = m1m/1+m1m 3/4 = + m14/3/1+4/3m1

i.e. m1 + 4/3 = 3/4 (1 4/3 m1) and m1 + 4/3 = 3/4 (1 4/3 m1).

The slopes are

(i) m1 = 7/24

(ii) m1 =

(the line is parallel to the y-axis).

The required equations of the lines are 7x + 24y + 182 = 0 and


x + 2 = 0.

Alternative solution:

Equation of the line, through P(2, 7) and making angle with the x-axis, is x+2/cos =
y+7/sin = r.

If this line intersects the given lines at A and B, with AB = 3, the points A and B are
A(2 + r1 cos , 7 + r1 sin ) and B (2 + (r1 + 3) cos , 7 + (r1 + 3) sin ).

Since A and B lie on the lines 4x + 3y = 3 and 4x + 3y = 12, we have

4r1 cos + 3r1 sin = 32 and


4r1 cos + 3r1 sin + 12 cos + 9 sin = 41, so that

12 cos + 9 sin = 0 or 4 cos + 3 sin = 3.

Solving this equation we find that = /2 and tan = 7/24.

Hence the required lines are x + 2 = 0

and y + 7 = 7/24 (x + 2) i.e. 7x + 24y + 182 = 0.

c Concurrency of Straight Lines


cc
The condition for 3 lines a1x + b1y + c1 = 0, a2x + b2y + c2 = 0,
a3x + b3y + c3 = 0 to be concurrent is

(i) = 0.

(ii) There exist 3 constants l, m, n (not all zero at the same time) such that IL1 + mL2 +
nL3 = 0, where L1 = 0, L2 = 0 and L3 = 0 are the three given straight lines.

(iii) The three lines are concurrent if any one of the lines passes through the point of
intersection of the other two lines.

Illustration:

Check if lines

a1 x + b1 y + c1 = 0 (1)

a2 x + b2 y + c2 = 0 (2)

(2a1 3a2)x + (2b1 3b2)y + (2c1 3c2) = 0 (3)

are concurrent?
Solution:
We can try to find ;, and  by observation as follow:
L3 2L1 + 3L2 = 0

Enquiry: many lines can pass through the intersection of two lines. Can we find a
general equation of these lines?

If L1 = 0 and L2 = 0 are two lines then equation of family of lines passing through their
intersection is given by

L1 + L2 = 0 (A)

Where is any parameter. (Equation A is satisfied by the point of intersection of L1= 0


and L2 = 0)

Note:

To determine a particular line one more condition is required so as to determine or


eliminate .

Illustration:

If x (2q + p) + y(3q + p) = 0

(x + y 1) + q/p (2x + 3y 1) = 0, p 0

Solution:
This equation represents the family of lines passing through the intersection of lines x +
y 1 = 0 and 2x + 3y 1 = which is fixed point i.e. (2, 1).

If p = 0 then equation becomes

q(2x + 3y 1) = 0

this also represents a line which passes through fixed points (2, 1).

Hence the given equation represents family of lines passing through a fixed point (2, 1)
for variable p, q.

Illustration:

Find the equation of a line, through the intersection of 2x + 3y 7 = 0 and x + 3y 5 = 0


and having distance from origin as large as possible.
Point of intersection of two lines is A(2, 1)

Now, with OA as radius and O itself as centre draw a circle.

There will be infinitely many lines through A and each except one of them
produces a chord of circle and hence their distance from origin i.e. centre of circle is
less than OA i.e. radius of circle.

But the exceptional one which infact is a tangent to circle at A will be at a


distance OA from O.

Thus, tangent to circle at A will be the line through A and is farthest from origin.

Now, OA R tangent at A.

(slope of OA) (slope of tangent at A) = 1

Or, 10/20 (slope of tangent at A) = 2

equation of required line is

(y 1) = 2(x 2)

Or 2x + y 5 = 0

Illustration:
Find the point of concurrency of the altitudes drawn from the vertices (at1t2, a(t1 + r2)),
(at2t3, a2t2 + t3)) and (at3t1, a(t3 + t1)) respectively of a triangle ABC.

Solution:

Slope of AD = t3.

Equation of AD is y a(t1 + t2) = t3(x + at1t2). (1)

Equation of CF is y a(t3 + t1) = t2(x at3t1). (2)

Subtracting (1) from (2), we get

x = a y = a(t1 + t2 + t1t2t3).

Hence the point of concurrency of the altitudes is

(a, a(t1 + t2 + t3 + t1t2t3)).

Position of two points with respect to a


given line
cc
Let the line be ax + by + c = 0 and P(x1, y1), Q(x2, y2) be two points.

Case 1:
If P(x1, y1) and Q(x2, y2) are on the opposite sides of the line
ax + by + c = 0, then the point A on the line ax + by + c = 0 divides the line PQ
internally in the ratio m1 : m2, where m1/m2 must be positive.

Co-ordinates of A

are (m1x2+m2x1/m1+m2, m1y2+m2y1/m1+m2).

Point A lies on the line ax + by + c = 0.

m1/m2 = ax1+by1+c/ax2+by2+c > 0

So that ax1 + by1 + c and ax2 + by2 + c should have opposite signs.

Case 2:

If ax1 + by1 + c and ax2 + by2 + c have the same signs then m1/m2 = ve, so that the point
A on the line ax + by + c = 0 will divide the line PQ externally in the ratio m1 : m2 and
the points P(x1, y1) and Q(x2, y2) are on the same side of the line ax + by + c = 0.

Illustration:

Find the range of in the interval (0, ) such that the points (3, 5) and (sin, cos) lie on
the same side of the line x + y 1 = 0.

Solution:

3 + 5 1 =7 > 0 sin + cos 1 > 0


sin(/4 + ) > 1/2

/4 < /4 + < 3/4

0 < < /2.

Illustration:

Find a, if (;, ;2) lies inside the triangle having sides along the lines

2x + 3y = 1, x + 2y 3 = 0, 6y = 5x 1.

Solution:

Let A, B, C be vertices of the triangle.

A (7, 5), B (5/4, 7/8),

C (1/3, 1/9).

Sign of A w.r.t. BC is ve.

cc
If p lies in-side the ABC, then sign of P will be the same as sign of a w.r.t. the line BC

5; 6;2 1 < 0. (1)

Similarly 2; + 3;2 1 > 0. (2)

And, ; + 2;2 3 < 0. (3)


Solving, (1), (2) and (3) for ; and then taking intersection,

We get ; ? (1/2, 1) (3/2, 1).

Illustration:

The equations of the perpendicular bisectors of the sides AB and AC of a triangle ABC
are respectively x y + 5 = 0 and x + 2y = 0. If the co-ordinates of A are (1, 2), find the
equation of BC.

Solution:

From the figure,

E (x1+1/2, y12/2),

F (x2+1/2, y22/2).

Alt text : equations of the perpendicular bisectors of sides of triangle

Since E and F lie on OE and OF respectively,

x1 y1 + 13 = 0 (1)

and x2 + 2y2 3 = 0 (2)

Also, slope of AB = 1 and slope of AC is 2, so that

x1 + y1 + 1 = 0. (3)

And 2x2 y2 4 = 0 (4)

Solving these equations, we get the co-ordinates of B and C as

B (7, 6) and C (11/5, 2/5)


Equation of BC is 14x + 23y 40 = 0.

Illustration:

Two fixed points A and B are taken on the co-ordinate axes such that OA = a and OB =
b. Two variable points A and B are taken on the same axes such that OA + OB = OA +
OB. Find the locus of the point of intersection of AB and AB.

Solution:

Let A (a, 0), B (0, b), A (a, 0), B (0, b).

Equation of AB is x/a' + y/b' = 1. . (1)

and equation of AB is x/a + y/b' = 1. . (2)

Subtracting (1) from (2), we get, x (1/a 1/a') + y(1/b' 1/b) = 0.

x(a'a)/aa' + y(bb')/bb' = 0. [Using a a = b b]

x/a(bb'+a) + y/bb', 0 b = a(a+b)y/aybx. .. (3)

From (2) bx + ay = (4) we get x + y = a + b

which is the required locus.

Angle Bisectors
cc
To find the equations of the bisectors of the angle between the lines

a1x + b1y+ c1 = 0 and a2x + b2y + c2 = 0.

A bisector is the locus of a point, which moves such that the perpendiculars drawn
from it to the two given lines, are equal.

The equations of the bisectors are

a1x+b1y+c1/a12+b12 = + a2x+b2y+c2/a22+b22.
AP is the bisector of an acute angle if,

Tan (pPAN) = tan (/2) is such that |tan /2| < 1.

AP is an obtuse angle bisector if,

Tan (pPAN) = tan (/2) is such that |tan /2| > 1.

Notes :

When both c1 and c2 are of the same sign, evaluate a1a2 + b1b2. If negative, then acute
angle bisector is a1x+b1y+c1/a12+b12 = + a2x+b2y+c2/a22+b22.

When both c1 and c2 are of the same sign, the equation of the bisector of the angle
which contains the origin is a1x+b1y+c1/a12+b12 = + a2x+b2y+c2/a22+b22.

Bisectors of the angle containing the point (;, ) is


a1x+b1y+c1/a12+b12 = +a2x+b2y+c2/a22+b22 if a1; + b1 + c1 and a2; + b2 + c2 have the same
sign.

Bisectors of the angle containing the point (;, ) is


a1x+b1y+c1/a12+b12 = +a2x+b2y+c2/a22+b22 if a1; + b1 + c1 and a2; + b2 + c2 have the
opposite sign.

Illustration:

For the straight lines 4x + 3y 6 = 0 and 5x + 12y + 9 = 0 , find the equation of the
(i) bisector of the obtuse angle between them,

(ii) bisector of the acute angle between them,

(iii) bisector of the angle which contains (1, 2)

Solution:

Equations of bisectors of the angles between the given lines are

4x+3y6/42+32 = + 5x+12y+9/52+122

9x 7y 41 = 0 and 7x + 9y 3 = 0.

If is the angle between the line 4x + 3y 6 = 0 and the bisector 9x 7y 41 = 0, then


tan = > 1.

Hence

(i) The bisector of the obtuse angle is 9x 7y 41 = 0.

(ii) The bisector of the acute angle is 7x + 9y 3 = 0.

For the point (1, 2)

4x + 3y 6 = 4 1 + 3 2 6 > 0.

5x + 12y + 9 = 12 12 + 9 > 0.

Hence equation of the bisector of the angle containing the point (1, 2) is 4x+3y6/5 =
5x+12y+9/13 9x 7y 41 = 0.

Pair of Straight Lines


cc
The equation ax2 + 2hxy + by2 + 2gx + 2fy + c = 0. Aepresents a second degree equation
where a, h, b doesnt variables simultaneously.

Let a 0.

Now, the above equation becomes

a2 x2 + 2ax (hy + g) = aby2 2afy ac


on completing the square on the left side, we get,

a2 x2 + 2ax (hy + g) = y2 (h2 ab) + 2y (gh af) + g2 ac.

i.e. (ax + hy + g) = + y2(h2ab)+2y(ghaf)g2ac

We cannot obtain x in terms of y, involving only terms of the first degree, unless the
quantity under the radical sign be a perfect square. The condition for this is,

(gh af)2 = (h2 ab) (g2 ac)

i.e. g2h2 2afgh + a2f2 = g2h2 abg2 abg2 ach2 + a2bc

cancelling and diving by a, we have the required condition

abc + 2fgh af2 af2 bg2 ch2 = 0

Illustration:

What is the point of intersection of two straight lines given by general equation ax2+
2hxy + by2 + 2gx + 2fy + c = 0?

Solution:
The general solution is

ax2 + 2hxy + by2 + 2gx + 2fy + c = 0 (1)

Let (;, ) be the point of intersection we consider line paralleled transformation.

x = x + ;, y = y +

From (1) we have

a(x + ;)2 + 2h(x + ;) (y + ) + b(y + )2 + 2g(x + ;) + 2f(y + ) + c = 0

ax2 + 2hxy + by2 + a ;2 + 2h; + b2 + 2g; + 2f + 2x(a ; + h + g) + 2y + 2y (h; +


b + f) = 0

ax2 + 2hxy + by2 + 2x(a; + h + g) + 2g + 2y (h; + b + f) = 0


Which must be in the form

ax'2 + 2hxy + by = 0

This cannot be possible unless

a; + h + g = 0

h; + b + f = 0

Solving

;/hfbg = /hgaf = 1/abh2

; = hfbg/abb2, = hgaf/abh2

Illustration:

Aepresent lines y = 2x and y = 3x by a homogeneous equation of second degree

Solution:
(y 2x) (y 3x) = 0

Or 6x2 5xy + y2 = 0

Illustration:

Aepresent lines parallel to y = 2x and y = 3x by a second degree equation


i  
(y 2x c1) (y 3x c2) (where c1 ad c2 are c sas)

= 6x2 5xy + y2 + (3c1 + 2c2) x + ( c1 c2) y + c1 c2 = 0

N e

1. H m gee s par s same as f r he eqa  f ab ve sra . Theref re, he


h m gee s par f a geera sec d degree eqa  deermes he s pe f he es
.e. es parae  ax2 + 2hxy + by2 + c = 0 ad hr gh he rg are represeed by
he eqa  ax2 + 2hxy + by2 = 0

2. The eqa  ax2 + 2hxy + by2 + 2fy + c = 0 represes a par f parae sragh
es f h/a = b/h = f/g r bg2 = af2

The dsace bewee hem s gve by 2g2ac/a(a+b) r f2bc/b(a+b)

Isra 

D es he sec d degree eqa  x2 + 3xy + 2y2 x 4y 6 = 0 represes a par f


es. If yes, fd her p  f ersec .

i  
We bserve ha

a = 1, h = 3/2, b = 2, g = 1/2, f = 2, c = 6

\ abc + 2fgh af2 bg2 ch2 = 12 + 3 4 1/2 + 27/2 = 0

Theref re he gve sec d-degree eqa  represes a par f es, x2 + 3xy + 2y2 x
4y 6 = (x + 2y + 2) (x + y 3).

C sder he eqa s f rmed by frs w r ws f .

.e. ax + hy + g = 0 ad hx + by + f = 0

.e. x + 3/2 y 1/2 ad 3/2 x + 2y 2 = 0

i vg hese, we ge he reqred p  f ersec .


i.e. 2x + 3y 1 = 0

3x 4y 4 = 0

Solving the above equation, we get x = 8, y = 5.

Note

(2x + 3y 1)(3x + 4y 4) x2 + 3xy + 2y2 x 4y 6.

c Angle between pair of linescrepresented by ax 2


+ 2hxy +
by = 0
2

cc
ccccccccComparing the coefficients of x2, y2 and xy, we get

b(y m1x) (y m2x) = ax2 + 2hxy + by2

m1 + m2 = 2h/b and

m1 m2 = a/b

tan acute = |m2m1/1+m1m2|

= |(m1m2)2 4m1m2/1+m1m2|

= |2h2ab/a+b|

Caution

1. If two lines through the origin are represented by

y = mx and y = m2x, we cannot write

(y m1x) (y m2x) ax2 + 2hxy + by2

Because coefficient of y2 on left hand side is one on right hand side, it is b.

2. The given equation represents real lines only when h2 ab > 0

Note

1. If two lines are coincident then tan = 0 h2 = ab


2. If two lines are perpendicular then

m1m2 = 1 a + b = 0

i.e. x2 + 2hxy y2 always represents pair of mutually perpendicular lines through


origin.

3. Two lines are equally inclined to axes but are not parallel. For such a case let us
take a line l1 which is inclined at an angle , then l2 is inclined at ( ).

tan ( ) = tan .

which is the condition for two lines inclined equally to axes.

4. m1 = h+h2ab/2 and m2 = hh2ab/b

Illustration

What is the equation of the pair of lines through origin and perpendicular to ax2 + 2hxy
+ by2 = 0

iolution
Let ax2 + 2hxy + by2 = 0 represents the lines y = m1x (i) and y = m2x (ii)

Lines perpendicular to the lines (i) and (ii) are y = 1/m1 x and y = 1/m2 x respectively
and passing through origin

i.e. m1y + x = 0 and m2y + x = 0

Their combined equation is given by

(m1y + x) (m2y + x) = 0

m1m2 y2 + (m1 + m2) xy + x2 = 0

a/2 y2 2h/b xy + x2 = 0

bx2 2hxy + ay2 = 0 is the equation of the pair of lines perpendicular to pair of lines
ax2 + 2hxy + by2 = 0

Note

Coefficient of x2 and y2 are interchanged and the sign of xy term is reversed.


V 
         
    
  +  + = 0
 

L   +  +  = 9    

 1 = 0 
  = 0

L  P(;, )           .

(m1;)/1+m12 = + m2;/1+m22

(1 + m22) ( m1;)2 (1 + m12) ( m2;)2 = 0

Locus of P(;, ) is

x2 y2 = 2hxy 1m1m2/m1+m1

x2y2/ab = xy/h; is required equation of angle bisectors (1)

Note

1. If a = b, then bisectors are x2 y2 = 0 i.e. x y = 0, x + y = 0

2. If h = 0, the bisectors are xy = 0 i.e. x = 0, y = 0

3. If in (i), coefficient of x2 + coefficient of 2 = 0, then the two bisectors are always


perpendicular to each other

Illustration

Prove that the angle between one of the lines given by


ax2 + 2hxy + by2 = 0 and one of the lines given by ax2 + 2hxy+ by2 + K(x2 + y2) = 0 is
equal to the angle between the other two lines of the system.

iolution
Let L1 and L2 be one pair and L3 and L4 be the other pair of lines.

If the angle between L1 and L3 is equal to the angle between L2 and L4 then pair of
bisectors of L1 and L2 would be same as that of L3 and L4. Pair of bisectors of L3 and L4 is

x2y2/(a+k)(b+k) = xy/h

x2y2/ab = xy/h
Which is the same as the bisector pair of L1 and L2.

6. Angle bisectors of ax2 + 2hxy + by2 + 2gx + 2fy + c = 0 (i) are given by (xx0)2(y
y0)2/ab = (xx0)(yy0)/h where (x0, y0) is the point of intersection of (i)

En iry If we shift the origin of the coordinate system how can the coordinates of a
point be known in the new system? What will happen if we rotate the axis?

(i) Linear Transformation

CP = x, DP = y; AP = x, BP = y

If origin is shifted to (h, k) then the coordinates of a point


P(x, y) in the new system are

X=xh

Y=yk

Yo can check it by ptting (h = 0, k = 0) that it gets redced to the same original
coordinate system.

(ii) Aotation of Axes


If axes are rotated anticlockwise by angle q then the coordinates of a point P(x, y)
changes to say P(x, y):

OL = x, PL = y

OM = X, PM = Y

OP = A (say)

cos (; + ) = x/A (i)

sin (; + ) = Y/A (ii)

cos ; = x/A (ii)

sin ; = Y/A (iv)

Eliminating ; from (i), (ii), (iii) and (iv) we get

X = x cos + y sin

Y = x sin + y cos

It can be checked that if = 0; then coordinates remain unchanged.

Illustration:

Find the equation of the line 2x + y = 7 when co-ordinates system is shifted to the point
(3, 1)
iolution:
x = 3 + X and y = 1 + Y

Equation of line becomes

2(3 + X) + (1 + Y) = 7

or 2X + Y = 0

Note:

ilope of the line remains the same.

The general equation of second degree ax2 + 2hxy + by2 + 2gx + 2fy + c=0 represents a

pair of straight lines if = 0.


abc + 2fgh af2 bg2 ch2 = 0 and h2 > ab.

The homogeneous second degree equation ax2 + 2hxy + by2 = 0 represents a pair of
straight lines through the origin.

If lines through the origin whose joint equation is ax2 + 2hxy + by2 = 0, are y = m1x and
y = m2x, then y2 (m1 + m2)xy + m1m2x2 = 0 and y2 + 2h/b xy + a/b x2 = 0 are identical.
If is the angle between the two lines, then tan = + (m1+m2)24m1m2/1+m1m2 = + 2h2
ab/a+b.

The lines are perpendicular if a + b = 0 and coincident if h2 = ab.

Joint Equation of Pair of Lines Joining the


Origin and the Points of Intersection of a
Line and a Curve

If the line lx + my + n = 0, (n 0) i.e. the line not passing through origin) cuts the curve
ax2 + by2 + 2gx + 2fy + c = 0 at two points A and B, then the joint equation of straight
lines passing through A and B and the origin is given by homogenizing the equation of
the curve by the equation of the line. i.e.

ax2 + 2hxy + by2 + (2gx + 2fy) (kx+my/n) + c (lx+my/n)2 = 0.

is the equation of the lines OA and OB.

Illustration:
Prove that the straight lines joining the origin to the points of intersection of the
straight line hx + ky = 2hk and the curve (x k)2 + (y y)2 = c2 are at right angles if h2 +
k 2 = c2.

Solution:

Making the equation of the curve homogeneous with the help of that of the line, we get

x2 + y2 2(kx + hy) (hx+ky/2hk) + (h2 + k2 c2) (hx+ky/2hk) = 0

or 4h2k2x2 + 4h2k2y2 4h2x(hx+ky) 4h2ky(hx + ky) + (h2 + k2 c)(h2x2 + k2y2 + 2hxy)


= 0.

This is the equation of the pair of lines joining the origin to the points of intersection of
the given line and the curve. They will be at right angles if

Coefficient of x2 + coefficient of y2 = 0 i.e.

(h2 + k2) (h2 + k2 c2) = 0 h2 + k2 = c2 (since h2 + k2 0).


cc

V 

V     
     %   V       

   !   
 '  
       
!    
    
              
       
  !    #   
  
# 
  

)               ! 
#
    
     "   
 
    # 

3V 4       
        V     
           

   
   " ! 
      %
 &  '   V 

     #    ./ '      

  



-c + V 


.c V    V 
/c V   V 
0c V  

1c 5   V 
2c $ !   $  ! 6   V 
7c 6 
8c % #   


V         #   

       ! 
  !       V  9         :  
     '         #       
 #           
         
       V V   


  

+ V  c
c
Locus has been defined as the path of a point satisfying some geometrical condition; i.e.
constraint equations. The path represents a curve, which includes all the points
satisfying the given condition.

Similarly a circle can be defined as: The locus of a point which moves in such a way
that its distance from a fixed point is always constant and positive. The fixed point is
called the centre of the circle and the given distance the radius of the circle. In real life,
when you rotate a stone tied with one end of a string then the path followed by stone is
exactly a circle whose centre is your finger an radius is length of the string.

The equation of a circle with its centre at C(xc, yc) and radius r is:

(x xc)2 + (y yc)2 = r2

Proof:

Let P(x, y) be any point on the circle. Then by the definition of the locus the
constant distance is (see figure given below)
|PC| = r ((x-xc) )2+(y-yc) )2 ) = r

(x xc)2 + (y yc)2 = r2

which is the required equation of the circle

Note:

(1) If xc = yc = 0 (i.e. the centre of the circle is at origin) then equation of the
circle reduce to x2 + y2 = r2

(2) If r = 0 then the circle represents a point or a point circle.

Equation of the circle in Various Forms

(i) The simplest equations of the circle is x2 + y2 = r2 whose centre is (0, 0) and radius r.

(ii) The equation (x a)2+ (y b)2 = r2 represents a circle with centre (a, b) and radius r.

(iii) The equation x2 + y2 + 2gx + 2fy + c = 0 is the general equation of a circle with
centre (g, f) and radius (g2+f2-c).

(iv) Equation of the circle with points P(x1, y1) and Q(x2, y2) as extremities of a diameter
is (x x1) (x x2) + (y y1)(y y2) = 0.
Equation of a circle under Different Conditions

Parametric Equation of a circle

Let us consider a circle of radius r and centre at C(xc, yc) we have:

(y-yc)/r = sin (see figure given below)

y = yc + r sin

Similarly x = xc + r cos

This gives the parametric from of the equation of a circle.

General equation of a circle in polar co-ordinate system

Let O be the origin, or pole, OX the initial line, C the centre and a the radius of the
circle.

Let the polar co-ordinates of C be A and ;, so that OC = A and pXOC = ; .

Let a radius vector through O at an angle with the initial line cut the circle at P
and Q. Let OP be r.

Then we have

CP2 = OC2 + OP2 2OC . OP cos COP

i.e. a2 = A2 + r2 2 Ar cos ( ;)

i.e. r2 2 Ar cos ( ;) + A2 a2 = 0 (1)

This is the required polar equation.

Particular cases of the general equation in polar coordinates.

Note:

1. Let the initial line be taken to go through the centre C. Then ; = 0, and the equation
becomes

r2 2Ar cos + A2 a2 = 0.

2. Let the pole O be taken on the circle, so that

A = OC = ;

The general equation the becomes

r2 2ar cos ( ;) = 0,

i.e. r = 2a cos ( ;).

3. Let the pole be on the circle and also let the initial line pass through the centre of the
circle. In this case
; = 0, and A = a

Now, the general equation reduces to the simple form r=2a cos

This is at once evident from the figure given above.

For, if OCA were a diameter, we have

OP = OA cos ,

r = 2a cos .

Let us consider a circle such that points P(x1, y1) and Q(x2, y2) are on it and PQ is one
of the diameters of the circle.

If A(x, y) is any point on the circle then

Aecall:

pPAQ = /2 (Angle subtended by diameter at any point on the circle is a right


angle).

QA R PA
(Slope of QA) x (Slope of PA) = 1

(y-y2)/(x-x2 )(y-y1)/(x-x1 ) = 1

(x x1) (x x2) + (y y1) (y y2) = 0

Which gives the required equation.

Note:

This equation can also be obtained considering

PA2 + QA2 = PQ2

The general from of the equation of a circle is:

x2 + y2 + 2gx + 2fy + c = 0 (1)

(x + g)2 + (y + f)2 = g2 + f2 c

Comparing this equation with the standard equation (x xc)2 + (y yc)2= r2

We have:

Centre of the circle is (g, f), Aadius = (g2+f2-c).

Equation (1) is also written as S = 0.

Note:

1. If g2 + f2 c > 0, circle is real

2. If g2 + f2 c = 0, circle is a point circle.

3. If g2 + f2 c < 0, the circle is imaginary.

4. Any second-degree equation ax2 + 2hxy + by2 + 2gx + 2fy+c=0 represents a circle
only when h = 0 and a = b i.e. if there is no term containing xy and co-efficient of
x2 and y2 are same, provided abc + 2fgh af2 bg2 ch2 0

Illustration:

Find the centre and the radius 3x2 + 3y2 8x 10y + 3 = 0.

Solution:
We write the given equation as x2 + y2 8/3 10/3 y + 1 = 0.

g = -4/3, f = -5/3 , c = 1

Hence the centre is (4/3,5/3) and the radius is

(16/9+25/9-1)=(32/9)=(42)/3.

Illustration:

Find the equation of the circle with centre (1, 2) which passes through the point (4,
6).

Solution:

The radius of the circle is ((4-1)2+(6-2)2 )=25 = 5.

Hence the equation of the circle is (x 1)2 + (y 2)2 = 25

x2 + y2 2x 4y = 20.

Illustration:

A circle has radius 3 units and its centre lies on the line y = x 1. Find theequation
of the circle if it passes through (7, 3).

Solution:

Let the centre of the circle be (;, ). It lies on the line y = x 1

= ; 1. Hence the centre is (;, ; 1).

The equation of the circle is (x ;)2 + (y ; + 1)2 = 9. It passes through (7, 3)

(7 ;)2 + (4 ;)2 = 9 2;2 22; + 56 = 0

;2 11; + 28 = 0 (; 7) = 0 ; = 4, 7.

Hence the required equations are

x2 + y2 8x 6y + 6 = 0 and x2 + y2 14x 12y + 76 = 0.

Illustration:
Find the equation of the circle whose diameter is the line joining the points (4, 3)
and (12, 1). Find also the intercept made by it on the y-axis.

Solution:

The equation of the required circle is

(x + 4) (x 12) + (y 3) (y + 1) = 0.

On the y-axis, x = 0 48 + y2 2y 3 = 0.

y2 2y 51 = 0 y = 1 52.

Hence the intercept on the y-axis = 2252 = 413.

Illustration:

Find the equation of the circle passing through (1, 1), (2, 1) and (3, 2).

Solution:

Let the equation be x2 + y2 + 2gx + 2fy + c = 0.

Substituting the coordinates of three points, we get

2g + 2f + c = 2,

4g 2f + c = 5,

6g + 4f + c = 13.

Solving the above three equations, we obtain:

f = 1/2; g = 5/2, c = 4.

Hence the equation of the circle is

x2 + y2 5x y + 4 = 0.

Illustration:

Write general equation of a circle centered at a point on x-axis.

Solution:
Circle is: x2 + y2 + 2gx + c = 0, g2 c 0

Its centre is (g, 0) and radius (g2-c)

Or

(x + g)2 + (y 0)2 = r2

Its centre is (g, 0) and radius r. (figure given above)

Illustration:

Write general equation of a circle passing through the origin.

Solution:

Point (0, 0) must satisfy x2 + y2 + 2gx + 2fy + c = 0

C=0

circle is : x2 + y2 + 2gx + 2fy = 0 (figure given above)

Illustration:

Write the equation of a circle centered at x-axis at (x1, 0) and touching y-axis at
the origin. (figure given below)

Solution:
(x x1)2 + (y 0)2 = (x1)

Illustration:

Write the equation of a circle passing through O (0, 0) A (a, 0) and B (0, b)?
Obviously AB is the diameter of the circle. (Figure given below)

Solution:

(x a) (x 0) + (y 0) (y b) = 0

Illustration:

Find the equation of circle shown in figure given below in polar form.

Solution:

OP = OA cos

r = 2a cos , /2 /2, a is raius of circle

Illustration:

Find the co-ordinates of the centre of the circle represented by


r = A cos + B sin .

Solution:

r = A cos + B sin

= [A/(A2+B2 ) cos +B/(A2+B2 ) sin ] (A2+B2 )

= cos ( ;) ((A2+B2 ))

centre is (1/2 (A2+B2 ),tan-1 (B/A) )

Note:

1. The equation of the circle through three non-collinear points

2. The circle x2 + y2 + 2gx + 2fy + c = 0 makes an intercept on x-axis if x2 + 2gx + c =


0 has real roots i.e. if g2 > c. And, the magnitude of the intercept is 2(g2-c).

The Position of a Point with respect to a Circle

The point P(x1, y1) lies outside, on, or inside a circle S x2 + y2 + 2gx + 2fy + c = 0,
according as S1 x12 + y12 + 2gx1 + 2fy1 + c > = or < 0.

Chord of a circle

The equation of the chord of the circle x2 + y2 + 2gx + 2fy +c=0 with M(x1, y1) as the
midpoint of the chord is

xx1 + yy1 + g(x + x1) + f(y + y1) = x12 + y12 + 2gx1 + 2fy1

i.e. T = S1

Illustration:

Find the equation of the circle whose centre is (3, 4) and which touches the line 5x
+ 12y = 1.

Solution:

Let r be the radius of the circle. Then

r = distance of the centre i.e. point (3, 4) from the line 5x + 12y = 1

= |(15+48-1)/(25-44)|=62/13.

Hence the equation of the required circle is (x 3)2 + (y 4)2 = (62/13)2.

x2 + y2 6x 8y + 381/169 = 0.

Illustration:

Find the co-ordinates of the point from which tangents are drawn to the circle x2 +
y2 6x 4y + 3 = 0 such that the mid-point of its chord of contact is (1, 1).

Solution:

Let the required point be (P(x1, y1). The equation of the chord of contact of P with
respect to the given circle is

xx1 + yy1 3(x + x1) 2(y + y1) + 3 = 0. (1)

The equation of the chord with mid-point (1, 1) is

x + y 3(x + 1) 2(y + 1) + 3 = 1 + 1 6 4 + 3

2x + y = 3.

Equating the ratios of the coefficients of x, y and the constant terms and solving for
x1, y1 we get x1 = 1, y1 = 0.
Chord of contactcc
c
Let AP and AQ be tangents to circle from point P(x1, y1). Then equation of PQ is known
as equation of chord of contact.

For circle x2 + y2 = a2, it is

x1x + y1y = a2

For general circle, it is

x1x + y1y + g(x1 + x) + f(y1 +y) + c = 0

Note:

1. It is also written as T = 0

2. The equation of chord AB [A (A cos ;, A sin ;); B (A cos , A sin )] of the circle
x2 + y2 = A2 is given by

x cos ((; + )/2) + y sin ((; - )/2) = a cos ((; - )/2)

3. If a line y = mx + c intersects the circle x2 + y2 = a2 in two distinct points A and B


then length of intercept AB = 2((a2 (1+m2 )-c2)/(1+m2 ))

Caution:

The equation of a chord of contact and the equation of the tangent on a point of the
circle and both given by T = 0. The difference is that while in the case of a tangent the
point (x1, y1) lies on the circle. In the case of a chord of contact (x1, y1) lies outside the
circle.
Illustration:

Write the equations of tangents to the circle x2 + y2 = 9 and having slope 2.

Solution:

Tangents with slope m are given by y = mx + a (1+m2 ) i.e. y = 2x


3 (1+4).

Note:

These are two parallel tangents to the circle at the end of the diameter.

Illustration:

Write the equation of tangents to the circle x2 + y2 = 25 at the point (3, 4)?

Solution:

Point (3, 4) lies on the circle.

Aequired equation of the tangent is : 3x + 4y= 25 using

x1x + y1y = a2, where (x1, y1) (3, 4)

Illustration:

Write the equation of normal to x2 + y2 = 25 at (3, 4)?

Solution:

Aecall:

A normal to the circle passes through the centre of the circle.

Normal is: y 0 = 4/3 (x 0) (using two point form of straight line) i.e. 3y 4x = 0
c

Common Tangents
c
a) Direct common tangents:
(i) The direct common tangents to two circles meet on the line of centres and divide it
externally in the ratio of the radii.

(ii) The transverse common tangents also meet on the line of centres and divide it
internally in the ratio of the radii.

Notes:

When one circle lies completely inside the other without touching, there is nocommon
tangent.

When two circles touch each other internally 1 common tangent can be drawn to the
circles.

When two circles intersect in two real and distinct points, 2 common tangentscan be
drawn to the circles.

When two circles touch each other externally, 3 common tangents can be drawn to
the circles.

When two circle neither touch nor intersect and one lies outside the other, then 4
common tangents can be drawn.

P is the point of intersection of two direct common tangents to the circles with
centres C1 and C2 and radii r1, r2 respectively. C1A1, C2A2 are perpendiculars from C1 and
C1 to one of the tangents (figure given below)
LPC1A1 and LPC2A2 are similar

(C1 P)/(C2 P)=(C1 A1)/(C2 A2 )=r1/r2 i.e. P is a point dividing C1C2 externally in the ratio r1 :
r2 For finding direct common tangents of two circles, find the point P dividing the line
joining the centre externally in the ratio of the radii. Equation ofdirect common
tangents is SS1 = T2 where S is the equation of one circle.

Caution:

Length C1C2 > |r1 r2|

(b) Transverse Common tangents

P is the point of intersection of two transverse tangents to two non-intersecting circles


with centres C1 and C2 and radii r1, and r2 respectively. Then P lies on the line joining
the centres. C1A1 and C2A2 are perpendiculars from C1 and C2 to one of these tangents.
(Figure given below)

Since triangles C1A1P and C2A2P are similar.

So (C1 P)/(C2 P)=(C1 A1)/(C2 A2 )=r1/r2

i.e. P divides the line joining C1 and C2 internally in the ratio r1:r2

Equation of transverse Common tangents is SS1 = T2 where S is the equation of one of the
circle.


Common Tangents

(a) Direct common tangents:

(i) The direct common tangents to two circles meet on the line of centres and divide it
externally in the ratio of the radii.
(ii) The transverse common tangents also meet on the line of centres and divide it
internally in the ratio of the radii.

Notes:

When one circle lies completely inside the other without touching, there is nocommon
tangent.

When two circles touch each other internally 1 common tangent can be drawn to the
circles.

When two circles intersect in two real and distinct points, 2 common tangentscan be
drawn to the circles.

When two circles touch each other externally, 3 common tangents can be drawn to
the circles.

When two circle neither touch nor intersect and one lies outside the other, then 4
common tangents can be drawn.

P is the point of intersection of two direct common tangents to the circles with
centres C1 and C2 and radii r1, r2 respectively. C1A1, C2A2 are perpendiculars from C1 and
C1 to one of the tangents (figure given below)
LPC1A1 and LPC2A2 are similar

(C1 P)/(C2 P)=(C1 A1)/(C2 A2 )=r1/r2 i.e. P is a point dividing C1C2 externally in the ratio r1 :
r2 For finding direct common tangents of two circles, find the point P dividing the line
joining the centre externally in the ratio of the radii. Equation ofdirect common
tangents is SS1 = T2 where S is the equation of one circle.

Caution:

Length C1C2 > |r1 r2|

(b) Transverse Common tangents

P is the point of intersection of two transverse tangents to two non-intersecting circles


with centres C1 and C2 and radii r1, and r2 respectively. Then P lies on the line joining
the centres. C1A1 and C2A2 are perpendiculars from C1 and C2 to one of these tangents.
(Figure given below)

Since triangles C1A1P and C2A2P are similar.

So (C1 P)/(C2 P)=(C1 A1)/(C2 A2 )=r1/r2

i.e. P divides the line joining C1 and C2 internally in the ratio r1:r2
Equation of transverse Common tangents is SS1 = T2 where S is the equation of one of the
circle.

Enquiry: What do we understand by co-axial circles and limiting points?

A system of circles every pair of which has the same radical axis is called a
coaxial system.

The centres of circles of a coaxial system, which are of zero radiuses, are called the
limiting points o the coaxial system.

Let the equation of a system of coaxial circles be

x2 + y2 + 2gx + c = 0

Where g is a parameter and c is a constant.

Its radius (g2-c)and centre is (g, 0)

If g2 c = 0 or g = + c, then radius become zero and for these two values of g we


have two circles of zero radius whose centres are ( c, 0).

These circles of zero radius are just points and according to definition given
above are the limiting points of the co-axial system.
1. If the system of circles is intersecting one, then c is negative and these limiting points
are two imaginary points.

2. If the system of circles are non intersecting then c is positive and these limiting
points are both real.

3. If c = 0, points of intersection are coincident to (0, 0) i.e.

Circle touch each other at (0, 0).

Limiting points coincide at (0, 0).

Enquiry: When do two circles intersect orthogonally?

The angle of intersection between two curves intersecting at a point is the angle
between their tangents drawn at that point. The curves are said to be intersecting
orthogonally, if the angle between their tangents are common point is a right angle.

Consider two circles

S1 x2 + y2 + 2g1x + 2f1y + C1 = 0

S2 x2 + y2 + 2g2x + 2f2y + C2 = 0

They intersect at point P such that tangent PT1 and PT2 are at right angle (see
figure given below)

Since radius of a circle is perpendicular to the tangent. So C1P and C2P are also
perpendicular.

(C1C2)2 = (C1P)2 + (C2P)2 (g1 g2)2 + (f1 f2)2 = r12 + r22


g12 + g22 2g1g2 + f12 + f22 2f1f2=g12 + f12 C1 + g22 + f22 c2

2g1g2 + 2f1f2 = C1 + C2

which is the required condition for the orthogonal intersection of two circles.

At what angle do the circles shown in figure intersect?

From the triangle C1C2P it is clear that angle can be written as:

cos = (r12+r22-d2)/(2r1 r2 )

Enquiry: What do we understand by pole and polar?

Let P(x1, y1) be a fixed point and chords be drawn through this point to a fixed
circles S (see figure given below). The locus of the point of intersection of tangents
drawn at the end points of chords is a line which is called the polar of point P(x1, y1)
(Point P is called the pole) w.r.t. the circle. So there is a fixed polar for a fixed point and
a fixed pole for a fixed line.
The equation of polar of a fixed point P(x1, y1) with respect to the circle x2 +
y2 + 2gx + 2fy + c = 0 is

xx1 + yy1 + g(x + x1) + f(y + y1) + c = 0

i.e. of the form T = 0

Family of Circlescc
c
Enquiry: If the numbers of conditions for a circle to be drawn are less than three then
what shall we get?

The general equation of a circle is x2 + y2 + 2gx + 2fy + c = 0. Since this equation


involves three unknowns i.e. g, f and c so we need at least three conditions to get a
unique circle.

For example, if we are given two circles and we want to determine the third circle
touching both of them. We shall need one more condition. Without the condition we get
the equation of family of circles which satisfies the two given conditions. Imposition of
a third condition will result in the equation representing a particular circle.

Let us now see some of the ways of the providing only two condition and equations of
the family of circles resulting under these conditions.

1. Family of circles having a fixed centre

This equation is given by (x h)2 + (y k)2 = r2 (Figure given below)

Since (h, k) is fixed, so only parameter varying is r. This is one parameter family
of circles, and is the equation of the family of concentric circles. Fixation of the radius
will give a particular circle.

2. Equation of Family of circles passing through intersection of two circles S1= 0 and
S2 = 0.

The general equation of the family of circles passing through the intersection of
S1 and S2 in given by S1 + kS2 = 0. Here again we have the one-parameter (k) equation of
family of circles. The particular value of the parameter gives a unique circles.

Caution:

If k = 1, we get equation of common chord i.e. straight line instead of circle.

Let S1 x2 + y2 + 2g1x + 2f1y + c1 = 0

S2 x2 + y2 + 2g2x + 2f2y + c2 = 0

Since, point lies on both the circles,

x2A + y2A + 2g1xA + 2f1yA + c1 = 0

x2A + y2A + 2g2xA + 2f2yA + c2 = 0

x2A + y2A + 2g1xA + 2f1yA + c1 + (x2A + y2A + 2g1xA + 2f1yA + c1)=0

Point A(xA, yA) lies on S1 + S2 = 0 c A

Similarly point B(xB, yB) lies on S1 + S2 = 0 c A


S1 + S2 = 0 is the family of circles through the intersection of S1 = 0 and S2 = 0

3. Family of Circles passing through intersection of line L and circle S:

This equation is given by S + L = 0

The particular value of the parameter gives a unique circle.

4. Family of circles touching the circle S = 0 and line L = 0 at their point of contact

Equation S + L = 0, where is the required family.

5. Family of circle passing through two given points A(x1, x1) and B(x2, y2)

Aequired family of circles is

6. Family of circles touching a given line L = 0 at a point (x1, x1) on the line is (x x1)2 +
(y x1)2 + L = 0, the particular value of the parameters gives a unique circle.

Family of Circles

(i) If S x2 + y2 + 2gx + 2fy + c=0 and S x2 + y2 + 2gx + 2fy + c=0 are two intersecting
circles, then S + S = 0, 1, is the equation of a family of circlespassing through the
points of intersection of S = 0 and S = 0.

(ii) If S = x2 + y2 + 2gx + 2fy + c = 0 is a circle which is intersected by the straight line 


= ax + by + c = 0 in two real and distinct points, then S +  = 0 is the equation of
a family of circles passing through the points of intersection of S = 0 and  = 0.

(iii) The equation of a family of circles passing through two given points (x1, x1) and (x2,
y2) can be written in the form

where is a parameter.

(iv) The equation of the family of circles which touch the line y x1 = m

(x x1) at (x1, x1) for any values of m is (x x1)2 + (y x1)2 +

[(y x1) m(x x1)] = 0.

Notes:

The two circles are said to intersect orthogonally if the angle of intersection of the
circles i.e., the angle between their tangents at the point of intersection is 90o.

The condition for the two circles to cut each other orthogonally is 2gg1+ 2ff1 = c + c1
where (g, f) and (g1, f1) are the centres of the respective circles, S = 0 and S1 = 0.

Illustration:

Find the equation of the circle described on the common chord of the circles x2 +
y2 4x 5 = 0 and x2 + y2 + 8y+ 7 = 0 as diameter

Solution:

Equation of the common chord is S1 S2 = 0

x + 2y + 3 = 0

Equation of the circle through the two circles is S1 + S2 = 0


x2 + y2 -4/(1+ ) x+8y/(1+ )+(7 -5)/(1+ ) = 0.

Its centre (2/(1+ ),-4/(1+ )) lies on x + 2y + 3 = 0

2/(1+ )-8/(1+ ) + 3 = 0 2 8 + 3 + 3 = 0 = 1.

Hence the required circle is x2 + y2 2x + 4y + 1 = 0.

Illustration:

The line Ax + By + C = 0 cuts the circle x2 + y2 + ax + by + c = 0 in P and Q. The Line


A?x + B?y + C? = 0 cuts the circle x2 + y2 + a'x + b'y + c' = 0 in A and S. If P, Q, A, S are
concyclic, prove that

Solution:

The equation of the circle through the first line and the first circle, i.e. through P
and Q is

x2 + y2 + ax + by + c + 1 (Ax + By + C) = 0. ........................ (1)

The equation of the circle through A and S is

x2 + y2 + a'x + b'y + c + 2 (A'x + B'y + C') = 0.................... (2)

Since P, Q, A, S are concyclic, (1) and (2) are identical.

1 = (a+1 A)/(a'+2 A' )=(b+1 B)/(b'+2 B' )=(c+1 C)/(c'+2 C' )

1A-2A' + a - a' = 0,

1B -2B' + b - b' = 0,

1C- 2C' + c - c' = 0.


Illustration:

Show that the circle passing through the origin and cutting the circles x2 + y2 - 2a1x -
2b1y + c = 0 and x2 + y2 - 2a2x - 2b2y + c = 0 orthogonally is

Solution:

Let the equation of the circle passing through the origin be

x2 + y2 + 2gx + 2fy = 0. ..................... (1)

It cuts the given two circles orthogonally

-2ga1 - 2fb1 = c1 c1 + 2ga1 + 2fb1 = 0 ? (2)

and -2ga2 - 2fb2 = c2 c2 + 2ga2 + 2fb2 = 0 ? (3)

Illustration:

Find that member of the family of circles having centre at (2, 3) which has radius of
5 unit.

Solution:

Family of circles having fixed centre (2, 3) is

(x - 2)2 + (y - 3)2 = r2
here we want that particular circle that has radius 5 units i.e. r = 5, the required circle
is,

(x - 2)2 + (y - 3)2 = 25

Illustration:

Find a circle passing through the intersection of x2 + y2 - 4 = 0 and x2 + y2 - 6x + 5 =


0 which passes through the point (2, 1)?

Solution:

Family of required circles is S1 + S2 = 0

(x2 + y2 - 4) + (x2 + y2 - 6x + 5) = 0

Since the required circle passes through the point (2, 1), the previous equation is
satisfied for the point (2, 1)

(4 + 1 - 4) + (4 + 1 - 12 + 5) = 0

1 - 2 = 0 = 1/2

Equation of the required circle is

(x2 + y2 - 4) + 1/2 (x2 + 2y - 6x + 5) = 0

x2 + y2 - 2x - 1 = 0

External and Internal Contacts of Circles

If two circles with centres C1(x1, y1) and C2(x2, y1) and radii r1 and r2 respectively, touch
each other externally, C1C2 = r1 + r2. Coordinates of the point of contact are A
((r1 r2+r2 r1)/(r1+r2 ),(r1 y2+r2 y1)/(r1+r2 )).
The circles touch each other internally if

C1C2 = r1 r2.

Coordinates of the point of contact are

T ((r1 x2+r2 x1)/(r1+r2 ),(r1 y2+r2 y1)/(r1+r2 )).

Illustration 11:

Examine whether the two circles x2 + y2 2x 4y = 0 and x2 + y2 8y 4 = 0 touch


each other externally or internally.

Solution:

Let C1 and C2 be the centres of the circles.

C1(1, 2) and C2(0, 4). Let r1 and r2 be the radii of the circles

r1 = 5 and r2 = 25. Also C1C2 = (1+4)=5.

But r1 + r2 = 35 and r2 r1 = 5 = C1C2.

Hence the circles touch each other internally.


Illustration 12:

Find all the common tangents to the circles x2 + y2 2x 6y + 9 = 0 and x2 + y2 6x


2y + 1 = 0.

Solution:

The centres and the radii of the circles are

C1 (1, 3) and r1 = (1+9-9) = 1, C2(3, 1) and r2 = (9+1-1) = 3,

C1C2 = 20, r1 + r2 = 4 = 16 and C1C2 > r1 + r2.

Sense the circles are non-intersecting. Thus there will be four common tangents.

Transverse common tangents are tangents drawn from the point P which divides
C1C2 internally in the ratio 1 : 3.

Direct common tangents are tangents drawn from the point Q which divides
C1C2 externally in the ratio 1 : 3.

Coordinates of P are

((1(-3)+3.1)/(1+3),(1.1+3.3)/(1+3)) i.e. (0,5/2).

and coordinates of Q are (3, 4).

Transverse tangents are tangents through the point (0,5/2).

Any line through (0,5/2) is

y 5/2 = mx (1)

mx y + 5/2 = 0

Applying the usual condition of tangency to any of the circle, we get

(m.1 3 + 5/2 = 0)/((m2+1) ) = 1 (m-1/2)2 = m2 + 1


m 3/4 = 0 or 0.m2 m 3/4 = 0.

m = 3/4 and as coefficient of m2 is zero.

Therefore from (1),

(y-5/2)/x = m = and -3/4 x = 0 is a tangent and

3x + 4y 10 = 0 is another tangent.

Direct tangents are tangents drawn from the point Q(3, 4).

Now proceeding as for transverse tangents their equations are

y = 4, 4x 3y = 0.

Power of a point with respect to a circle


The power of a point P(x1, y1) with respect to a given circle defined as

Power = PA PB

Where A and b are the points on the circle where the line PAB intersects it. (See
figure given below).

Clearly when A and B coincide to T then we have

Power = PT2 = Square of the length of tangent from point P

(x1, y1) to the circle.


Aadical Axis
This radical axis of two circles is the locus of a point from which the tangent segments
to the two circles are of equal lengths.

Equation of the Aadical Axis

In general S S = 0 represents the equation of the Aadical Axis to the two circles

i.e. 2x(g g) + 2y(f f) + c c = 0

where S x2 + y2 + 2gx + 2fy + c = 0

and S x2 + y2 + 2gx + 2fy + c = 0

(i) If S = 0 and s = 0 intersect in real and distinct points then S S = 0 is the equation of
the common chord of the two circles.

(ii) If S = 0 and S = 0 touch each other, then S S = 0 is the equation of the common
tangent to the two circles at the point of contact.

Properties of the Aadical Axes

The radial axis of two circles is perpendicular to the line joining the centres.

The radical axis of three circles taken two at a time are concurrent and the point of
concurrency is known as the radical centre.

The radical axis of two circles bisects their direct common tangents.
If two circles cut a third circle orthogonally, then the radical axis of the two circles
will pass through the centre of the third circle.

Solved Examples

Example 1:

Find the equation of the circle circumscribing the triangle formed by the lines x +
y = 6, 2x + y = 4, x + 2y = 5.

Solution:

Method 1.

Consider the equation

(x + y 6) (2x + y 4) + 1 (2x + y 4) (x + 2y 5) + 2

(x + 2y 5) (x + y 6) = 0 (1)

This equation is satisfied by the points of intersection of any two of the given
three lines, i.e. it is satisfied by the vertices of the triangle formed by the given lines.

Aesult:

Now if (i) represents a circle then

(a) coefficient of x2 = coefficient of y2 and

(b) coefficient of xy = 0

And

3 + 51 + 32 = 0 1 = 6/5

Substituting these values in (i) and simplifying


We get

x2+ y2 17x 19y + 50 = 0 (Ans.)

Which is the equation of the required circle.

Method 2.

Solve the lines in pair to find the vertices of the triangle and then obtain
theequation of the circle through these three points.

Example 2:

Find the locus of the point of intersection of perpendicular tangents to the circle
x + y2 = 4.
2

Solution:

Method 1.

y = mx + 2 (1+m2 ) is tangent to x2 + y2 = 4 for all values of m.

It passes through (h, k) if

k = mh + 2 (1+m2 )

(k mh)2 = 4 (1 + m2)

m2 (h2 4) 2h km + (k2 4) = 0 (i)


The roots m1 and m2 of this quadratic equation are the slopes of PT and PT. If PT
and PT are at right angle then m1m2 = 1.

From (i)

m1m2 = (k2-4)/(h2-4)

(k2-4)/(h2-4) = 1

h2 + k2 = 8

Locus of P(h, k) is x2 + y2 = 8

Method 2:

Equation of tangent at the point T(2 cos , 2 sin )

is 2 cos x + 2 sin y = 4

i.e. x cos + y sin = 2 (i)

Tangent at T would be perpendicular to the tangent at T


If p TOT = 90o

i.e. pAOT = 90 +

Co-ordinates of T are (2 cos (90 + ), 2 sin (90 + ))

Equation of tangent at the point T is

2 sin x + 2 cos y = 4

or, x sin + y cos = 2 (ii)

Think:

How to get the locus of point P(h, k)?

Caution:

Do not simply square and add (i) and (ii). Though we get the required result, but
that it not the right approach.

Well, lines (i) and (ii) both pass through the point P(h, k)

h cos + k sin = 2 and

h sin + k cos = 2

._Now we want to get a relation between h and k and also eliminate

The best way to do this is to square and ad above two

Equations and we get h2 + k2 = 8

Locus of P(h, k) is x2 + y2 = 8

Method 3:

OP2 = OT2 + TP2 ( LOTP is a right angled triangle)


OP2 = OT2 + OT2

OP2 = 2A2

h2 + k2 = 2(4) = 8

Locus of P(h, k) is x2 + y2 = 8.

Note:

x2 + y2 = 8 is director circle of the circle x2 + y2 = 4

Example 3:

Find the condition that the line 3x + 44y p = 0 is tangent to the circle x2 + y2 4x
6y + 9 = 0

Solution:

Aadius of the given circle is 2 and centre is (2, 3). So for line 3x + 4x p = 0 to be
tangent to the circle we have,

|(3.2+4.3-p)/(32+42 )| = 2.

|18 p| = 10

18 p = 10 p = 8

18 p = 10 p = 28

Hence the required condition is p = 8 or p = 28. (Ans.)

Example 4

A circular plot of land in the form of a unit circle is to be divided into two equal
parts by the arc of a circle whose centre is on the circumference of the circular plot.
Show that the radius of the circular arc is 2 cos , where is given by sin 2 2 cos 2
= /2.

Solution:
Let O be the centre of the given circular plot of radius 1 i.e. OA = OB = OC = 1 and
A be any point on its circumference. Again BDC be the arc of the circle with centre A
and dividing the given circle into two equal parts. Let r the radius of the new circle,
then AB = AC = AD = r.

Let pAOB =

Then pOBA = pOCA = , and pAOB = ( 2).

Now area ABDCA must be = 1/2 area of unit circle = ( (1)2)/2 = /2 (i)

r/sin( -2 ) =1/(sin ) r = 2 cos .

Aequired Area APBDCQA = Area of sector ABDCA + area of sector OCQAPB 2


area of OAB

/2 = 1/2 r2 (2) + 1/2 (1)2 (2 4) 2 1/2 (1)2 sin ( 2)

sin 2 2 cos 2 = /2 (Ans.)

Example 5:

Find the equation of the circle which passes through the point (2, 0) and whose
centre is the limit of the point of intersection of lines 3x + 5y = 1, (2 + c)x + 5c2 y = 1.

Solution:

Solving, 3x + 5y = 1, (2 + c)x + 5c2y = 1

We get,
when c ] 1

Pause:

We will study limits in detail in module 5.

Now, we want to find out the equation of the circle which passes through (2, 0) and
has its centre at (2/5), 1/25).

Equation of the circle is

(x-2/5)2+(y+1/25)2=(2/5-2)2+(1/25)2

This is the required equation of the circle.


c

Example 6:

Find the equation of circle having the lines x2 + 2xy + 3x + 6y = 0 as its normals
and having size just sufficient to contain the circle, x(x 4) + y(y 3) = 0

Solution:

The combined equation of two normal of the circle is given by

x2 + 2xy + 3x + 6y = 0

(x + 3)(x + 2y) = 0

x = 3, x = 2y

Aecall:
A normal to a circle always passes through the centre of the circle. Now solving
these, we get the co-ordinates of the centre of the circle as (3, 3/2); because the two
normal intersect at the centre of the circle

The required circle just contains the circle

x(x 4) + y(y 3) = 0

i.e. x2 + y2 4x 3y = 0 (i)

Hence the required circle will touch the circle given by (1) internally.

Let r be the radius of the required circle. Now the two circles given by (1)

= ([22+(3/2)2 ] )=5/2 and centre = (2,3/2)

Now the required circle will touch the circle (i) internally, if

We have distance between the centre of the two circles = difference between their
radii.

((-3-2)2+(3/2-3/2)2 )=(r-5/2)

r = 15/2

Hence the equation of the required circle is given by

(x + 3)2 + (y-3/2)2=(15/2)2

x2 + y2 + 6x 3y 45 = 0 (Ans.)

Example 7:

A tangent is drawn to each of the circles x2 + y2 = a2, x2 + y2 = b2. Show that if these
two tangents are perpendicular to each other, the locus of their point of intersection is
a circle concentric with the given circles.

Solution:

Method 1:
Let P (x1, y1) be the point of intersection of the tangents PA and PB where A, B
are points of contact with the circles respectively.

As PA is perpendicular to PB, the corresponding radii OA and OB are also


perpendicular. Let pAOX =

pBOX = 90 +

Using the parametric from of the circle we can take

A (a cos , a sin )

p B (b sin , b cos )

The equation of PA is

x (a cos) + y (a sin ) = a2

y cos x sin = b

Since P(x1, y1) lies on these tangents

x1 cos + y1 sin = a and y1 cos x1 sin = b

. Squaring and adding above equation (we get)._as is a variable quantity; we


eliminate

x12 + y12 = a2 + b2

locus of p is x2 + y2 = a2 + b2, which is concentric with given circles.


Method 2:

OAPB is a rectangle

OP2 = OA2+ AP2

x12 + y12 = a2 + b2

Locus of P(x1, y1) is x2 + y2 = a2 + b2

Example 9:

The circle x2 + y2 = 1 cuts x-axis at P and Q. Another circle with centre at Q and
variable radius intercepts the first circle at A above x-axis and the line segment PQ at
S. Find the maximum area of the triangle QSA.

Solution:

Method 1.

Equation of circle centred at Q is (x + 1)2 + y2 = 2

Since point A (cos , sin ) lies on this circle

2 = (cos + 1)2 + (sin )2 = 2 + 2 cos

= 2 (2cos2 /2)  = 2 cos /2

A = Area of LQSA = 1/2 base altitude


= 1/2  sin

= cos /2 sin (i)

dA/d = cos cos /2-1/2 sin /2 sin

Pause:

We will study maxima and minima in module 5.

For max./min. dA/d = 0

tan = 2 cot /2

2t/(1-t2 )=2/t (where t=tan /2)

t = tan /2=1/2

(d2 A)/(d2 ) = sin cos /2-1/2 cos sin /2-1/2 sin /2 cos 1/4 cos /2 sin

(A"( )(at tan /2=1/2) ) = ve

From (i)

Maximum area = cos /2 sin

= 2 sin /2 (cos /2)2

4/(33) sq. units. (Ans.)

Method 2.
Equation of circle I is

x2 + y2 = 1 It cuts x-axis

where P (1, 0) and

Q (1, 0)

Let QA =  then equation of the circle II.

Centred at Q(1, 0) and radius =  is given by

(x + 1)2 + y2 = 2 (1)

Solving it with x-axis; we get S ( 1, 0).

Also solving the two circles, we get the co-ordinates of A

as [(2/2)-1,/2 ((4-2 ) )]

The area of LQAS = 1/2 QS AL

= 1/2  /2 ((4-2 ) )

= A (say)

Now A is max./min. means A2 is max./min.

Let A2 = Z.

Then Z = 4/16 (4 )2

dz/d=1/4. 43 (65)/16

(d2 z)/d2 = 33 30 (4/16)

For max./min. of A i.e. max./min. of A2 or Z. we get

dZ/d = 0  = (8/3) and then

(d2 z)/d2 = 3.8/3-30/10.64/9 = ve


For  = ((8/3))

Thus area is max when  = ((8/3))

Also max. area of LQAS

1/4.8/3 ((4/3))=(4/33) sq. units.


c
Example 10:

Distances from the origin to the centres of three circles x2 + y2 2x = c2(where c


is constant and is variable) are in G.P. Prove that the lengths of tangents drawn from
any point on the circle x2 + y2 = c2 to the three circles are in G.P.

Solution:

The equation of the three circles is

x2 + y2 2x = c2 (1)

where = 1, 2, 3. Their centres are:

( 1
, 0), (2, 0) and (3, 0)

Now distances of these points from the origin are 1, 2 and 3

13 = 22 (2)

Now, let P(h, k) be any point on the circle

x2 + y2 = c2, then h2 + k2 = c2 (3)

If r1, r2, r3 are the lengths of the tangents from P(h, k) on

The three circles, we then obtain

r12 = h2 + k2 21h c2 = c2 h c 22, using (3)

r12 = 2h
Similarly r22 = 22h

r32 = 23h

= r12 r32 = 41 2 h2 = 422 h2 = (r22)2

= r2r3 r22 r1, r2, r3 are in G.P. (Proved).

Example 11:

Show that the equation x2 + y2 4x ky 5 = 0 represents (for variable k) a family


of circles passing through two fixed points A and B. Find the equation of the circle
belonging to this family and cutting circle x2 + y2 6x 5y = 0 at right angles.

Solution:

x2 + y2 4x ky 5 = 0

(x2 + y2 4x 5) + k ( y) = 0

This is the equation of family of circles passing though the intersection point of x2+
y2 4x 5 = 0 (a circle) and a straight line putting y = 0, in x2 + y2 4x 5 = 0 gives

x2 4x 5 = 0

x = 1, 5

Hence the given circle passes through two fixed points (1, 0) and (5, 0)

For given family of circles

x2 + y2 4x ky 5 = 0 (1)

g = 2, f = k/2, c = 5

One member of family (1) and circle x2 + y2 6x 5y 0 (2)

Intersect orthogonally
For circle (2)

g = 3, f = 5/2, c = 0

For two circle to be orthogonal

2(gg + ff) = (c + c)

2[(2) (3) + (k/2)(5/2)] = 5 + 0

2 [6+5K/4] = 5

12 + 5K/2 = 5

5K/2 = 17

K = 34/5

Aequired equation of circle is

x2 + y2 4x + 34/5 y 5 = 0 (Ans.)

Example 12:

Lines 5x + 12y 10 = 0 and 5x 12y 40 = 0 touch a circle C1 of diameter 6. If the


centre of C1 lies in first quadrant, find the equation of circle C2 which is concentric with
C1 and cuts intercept of length 8 on these lines.

Solution:

Aecall:

If a circle touches two lines L1 and L2 then the centre of the circle lies on the angle
bisectors of the lines.

Angle bisector of given lines are

(5x+12y-10)/13=(5x-12y-40)/13

Taking +ve sign: y = (-5)/4


Taking ve sign: x = 5

Given lines L1 and L2 intersect at (5,-5/4)

Since the centre of C1 lies in the first quadrant, it can lie on x = 5 only.

Let the centre of C1 be (5, y1)

|(5(5)+12y1-10)/(52+122 )| = 3

15 + 12y1 = 39

y1 = 2 or y1 = -54/12 (Neglect)

Centre of C1 is (5, 2)

Since C2 is concentric with C1, its centre is also (5, 2)

C2 cuts intercept of length 8 on lies

5x + 12y 10 = 0 and 5x 12y 40 = 0

AB = CD = 8

Aecall:

Perpendicular from centre bisects the chord.

AN = 4

C2 N = 3 (given)
Aadius of C2 = r (say)

r2 = (AN)2 + (C2N)2 = 16 + 9

r=5

Equation of C2 is (x 5)2 + (y 2)2 = 25

Example 13:

Find the equation of the circle which passes through the point (2a, 0) and whose
radical axis with respect to the circle x2 + y2 = a2 is the lines x = a/2.

Solution:

Aecall:

Aadical axis of two circles is S1 S2= 0.

Let equation of circle is

x2 + y2 + 2gx + 2fy + c = 0 (i)

radical axis of circle (i) and circle x2 + y2 a2 = 0 is given by

x2 + y2 + 2gx + 2fy + c = x2 + y2 a2

2gx + 2fy + c + a2 = 0 (ii)

It is given that radical axis is x a/2 = 0 (iii)

Comparing (ii) and (iii)

We get f = 0, (c+a2)/2g=-a/2

ag + a2 + c = 0 (iv)

Circle (i) passes through (2a, 0)


4ag + 4a2 + c = 0 (v)

From (iv) and (v)

3ag + 3a2 = 0

g = a

c = (ag + a2) from (iv)

= (a2 + a2) = 0

Equation of circle is

x2 + y2 2ax = 0 (Ans.)

Example 14:

Show that x2 + y2 + 4y 1 = 0, x2 + y2 + 6x + y + 8 = 0 and x2 + y2 4x 4y 37 = 0


touch each other.

Solution:

S1 x2 + y2 + 4y 1 = 0

S2 x2 + y2 + 6x + y + 8 = 0

S3 x2 + y2 4x 4y 37 = 0

C1 (0, 2), C2 (3, 1/2), C3 (2, 2)

r1 = (4+1) = 5

r2 = (9+1/4-8)=5/2

r3 = (4+4+37)=45=35

C2C3 = ((2+3)2+(2+1/2)2 )=(25+25/4)=5/2 5

C1C3 = ((2-0)2+(2+2)2 )=25


r1 + r2 = 3/2 5 = C1C2

r3 r2 = 5/4 5 = C2C3

r3 r1 = 25 = C1C3

Aecall:

Two circles touch each other.

(i) externally if C1C2 = r1 + r2

(ii) internally if C1C2 = r1 r2

Example 15:

Find the four common tangents to the circles x2 + y2 22x + 4y + 110 = 0 and x2 +
y2 22x 4y 100 = 0

Solution:

S1 x2 + y2 22x + 4y + 100 = 0

S2 x2 + y2 + 22x 4y 100 = 0

C1 (11, 2), C2 (11, 2)


r1 = ((11)2+(-2)2-100) = 5

r2 = ((-11)2+(2)2+100) = 15

Out of four common tangents are transverse tangents and other two direct tangents.
(1) and (2) are direct common tangents while (3) and (4) are transverse common
tangents.

Aecall:

Transverse common tangents divide line joining centres in ratio of radii internally
while direct tangents divides line joining centres in ratio of radii externally.

Let T1, T2 divide C1C2 in ratio of r1 : r2 internally and externally respectively.

Co-ordinates of T1 are (1511+5(-11))/(15+5) and (15(-2)+52)/(15+5)

That is T1 is the point (11/2,-1)

Co-ordinates of T2 are (1511-5(-11))/(15-5) and (15(-2)-52)/(15-5)

that is T2 is the point (22, 4)

Let the equation to either of the tangents, passing through T1 be

y + 1 = m (x 11/2) (A)

Then the perpendicular from the point (11, 2) on it is equal to + 5 and hence

(m(11-11/2)-(-2+1))/(1+m2 ) = 5

On solving, we have m = -24/7 or 4/3


The required tangents through T1 are therefore

24x + 7y = 125, and 4x 3y = 25

Similarly the equation to the tangents through T2 is

y + 4 = m (x 22) (B)

where (m(11-22)-(-2+4))/(1+m2 ) = 5

On solving, we have m = 7/24 or 3/4

On substitution in (B) the required equations are therefore

x 24y = 250 and 3x + 4y = 50

The four common tangents are therefore found. (Ans.)

R ccc
c
ccccccccc c
c c cc
cc c cccccccc ccccccc
 ccc
cc cccccc cc
c c cc
c c c
!
c c "#$ ccc%c &cc
c
c
Solution:

Method 1.

The equation of the incircle can be put in the from (x 2)2 + (y 2)2 = 4

This implies that the inradius r = 2 (i)

Let the hypotenuse of the triangle meet OX and OY at A (a, 0) and B (0, b)
respectively.

r = (Area of LAOB) / ((1/2)(Sum of sides of LAOB)) = ((1/2)ab) /


((1/2)(a+b+(a2+b2 )))

2 = ab/(a+b+(a2+b2 ))

Let M (x1, y1) be the circumcentre of LOAB. Since L OAB is right angled, its
circumcentre is the mid point of hypotenuse.

2 = (4x1 y1) / (2x1+2y1+2(x12+y12 ))

x1 + y1 + (x12+y12 ) = x1y1

Hence the equation of the locus of M(x1, y1) is

x + y xy + (x2+y2 ) = x1y1

Comparing it with the given equation of the locus, we find that

k = 1. (Ans.)

Method 2.

Equation of AB is x/a + y/b = 1 (1)

(x1, y1) (a/2,b/2), where M(x1, y1) is the circumcentre of LOAB i.e. midpoint of the
hypotenuse.

(1) becomes: x / (2x1 )+y / (2y1 ) = 1 (2)

In circle (x 2)2 + (y 2)2 = 4 touches line (2)

(2/(2x1 )+2 / (2y1 )-1) / ((1/2x1 )2+ (1/2y1 )2 ) = 2

x1 + y1 x1 y1 + (x12+y12 ) = 0

Locus of M(x1, y1) is

x + y xy + (x2+y2 ) = 0

Comparing it with the given equation of the locus, we find that k = 1.

Note:

Distance from (2, 2) to the line x/2x1 +y/2y1 -1 = 0 has been taken 2, because origin
and this point lies on the same side of the origin.
 
c
ccccccc
  


  is the chief and easiest chapter in the 


 
 of Co-ordinate Geometry in Mathematics.
Let us start analyzing the thought A point always moves such that the ratio of its distances from a fixed
point and a fixed line is constant. Can we get some meaningful result? Yes we get a    , which
follow the above rule/thought. We can represent the curve mathematically using the co-ordinates and this
curve is useful in finding out the many physical/practical phenomenons.

If we take a Right Circular Cone and cut it by a horizontal plane, we get a cross-section which is circular. If
we cut this cone by planes of different orientations, different planes produce different type of curves. As
all these curves are sections of a right circular cone, we call them 
 
. When we analyze these
curves in detail we find that when the ratio of the distances of a point on the curve from a fixed point to
its distance from a fixed line, is equal to 1 we have one type of curve, for ratio less than 1, we have second
type of curve and for ratio more than 1, we have third type of curve. With this basis we can analyze these
three curves.

In this sections well discuss the curve of 1st type for which the ratio is equal to 1 i.e. the distance from the
fixed point is always equal to the distance from the fixed line. We call this a   

  is one of the easiest and important chapters of Conic Sections of Co-ordinate Geometry in the
Mathematics syllabus of IIT JEE, AIEEE and other engineering examinations. The chapter is important
because it fetches 1-2 questions in most of the engineering examination





1.c 
 
   


2.c X  
3.c 
4.c @

    
5.c =     
6.c  

   
7.c  

  is important from the perspective of scoring high in IIT JEE as there are few fixed pattern on
which a number Multiple Choice Questions are framed on this topic. You are expected to do all the
questions based on this to remain competitive in IIT JEE examination. It is very important to master these
concepts at early stage as this forms the basis of your preparation for IIT JEE, AIEEE, DCE, EAMCET and
other engineering entrance examinations.


 

Conics or conic sections are the curves corresponding to various plane sections of a right circular
cone by cutting that cone in different ways.

Each point lying on these curves satisfies a special condition, which actually leads us towards the
mathematical definition of 
 
.

If a point moves in plane in such a way that the ratio of its distance from a fixed point to its
perpendicular distance from a fixed straight line, always remains constant, then the locus of that point I
called a Conic Section.

The fixed point is called the focus and the fixed line is called directrix of the conic. The constant ratio is
called the  
  and is denoted by e.

According to the value of there are three types o conic i.e. for e = 1, e < 1 and e > 1 the
corresponding conic is called    , ellipse and hyperbola respectively.

A 
 
or conic is the locus of a point, which moves so that its distance from a fixed point is in a
constant ratio to its distance from a fixed straight line, not passing through the fixed point.

The fixed point is called the focus.

The fixed straight line is called the    .

The constant ratio is called the  


  and is denoted by e.

When the  
  is unity i.e. e = 1, the conic is called a    ; when e < 1, the conic is called an
ellipse; and when e > 1, the conic is called a hyperbola.

The straight line passing through the focus and perpendicular to the     is called the axis of
the    .

The point of intersection of a conic with its axis is called vertex.


The chord passing through focus and perpendicular to axis is called   .

Any chord of the    which is perpendicular to the axis is called double ordinate.

The straight line perpendicular to axis of the    passing through vertex is called tangent at the
vertex.

   


The line through focus and perpendicular to the     is called the axis of the conic. The intersection
point o conic with axis is known as the vertex of the conic.


  !!  
    
!      "

The locus of the point, which moves such that its distance from a fixed point (i.e. focus) is always
equal to its distance from a fixed straight line (i.e. directrix), is called    

 
   

Let S be the focus, V be the vertex, ZM be the     and x-axis be the axis of    . We
require therefore the locus of a point P, which moves so that its distance from S, is always equal to PM i.e.
its perpendicular distance from ZM. After appropriate configuration let S = (a, 0)

2 2
We have ten SP = PM

2 2 2
(x a) + y = (a + x)

2
y = 4ax This is the standard  
    


There are four common forms of parabola according to their axis, with their vertex at origin (0, 0).

Let us understand some other features of a    .

# $X  


The distance of a point on the parabola from its focus is called the focal distance of the point Focal
distance of P = SP = x + a.

#$X  

A chord of the parabola, which passes through its focus, is called Focal chord.
# $% & 

The chord through focus and perpendicular to the axis of the parabola is called the latus rectum.

The co-ordinates of the end point of the latus rectum are (a, 2a) and (a, 2a) and length of latus rectum
= |4a|.

X '
   

Let us consider origin (0, 0) as the vertex A of the    and two equidistant points S(a, 0) as focus
and Z(a, 0) a point on the directrix now let P(x, y) be the moving point. Draw SZ perpendicular from S on
the directrix. Then SZ is the axis of the parabola. Now the middle point of SZ, that is A, will lie on the locus
of P.

i.e. AS = AZ.

The x-axis along AS, and the y-axis along the perpendicular to AS, as A, as in the figure. Now by definition
2 2
PM = PS MP = PS

So, that, (a + x)2 = (x a)2 + y2.

Hence, the equation of horizontal    is y2 = 4ax.


       

Let us consider origin (0, 0) as the vertex A of the parabola and two equidistant points S(0, b) as focus
and Z(0, b) a point on the directrix now let P(x, y) be the moving point. Draw SZ perpendicular from S on
the directrix. Then SZ is the axis of the    . Now the middle point of SZ, that is A, will lie on the
locus of P i.e. AS = AZ.

The y-axis along AS, and the x-axis along the perpendicular to AS at A, as in the figure.

Now by definition PM = PS

MP2 = PS2

2 2 2
So that, (b + y) = (y b) + x .

Hence, the equation of vertical parabola is x2 = 4by.


X

 

 & 

For finding the end points of latus rectum LL of the parabola y2 = 4ax, we put x = a as latus rectum passes
through focus (a, 0) therefore we have

y2 = 4a2
y = + 2a

Hence the end points are (a, 2a) and (a, 2a).

Also LSL = |2a (2a)| = 4a = length of double ordinate through the focus S.


= 

Two     are said to be equal when their   are equal.

@ 

(
  
    

= 

1. The points and lines of two     can be interchanged by transformations.

2. If a > 0 & a < 0 the    will be forward opening and backward opening respectively.

3. If b > 0 & b < 0 the    will be forward opening and downward opening respectively.


@ 

(
      
The forms of the horizontal and vertical parabola having vertex at (h, k) can be obtained by shifting the
origin at (h, k) as below

  


Find the vertex, axis,    , tangent at the vertex and the length of the  of
2
the    2y + 3y 4x 3 = 0.

 


The given equation can be re-written as (y-3/4)2= 2 (x+33/32)

2
Which is of the form Y = 4aX.

Hence the vertex is (-33/32,-3/4).

The axis is y + 3/4 = 0 y = 3/4.

The     is X + a = 0.

x + 33/32+1/2 = 0 x = -49/32.

The tangent at the vertex is x + 33/32 = 0 x = 33/32.


Length of the    = 4a = 2.


  


The extreme points of the    of a parabola are (7, 5) and (7, 3). Find the equation of
the    and the points where it meets the coordinate axes.

 


Focus of the    is the mid-point of the   

S is (7, 4). Also axis of the parabola is perpendicular to the    and passes through the
focus. Its equation is

y 4 = 0/(5-3) (x 7) y = 4.

Length of the   = (5 3) = 2.

Hence the vertex of the parabola is at a distance 2/4 = .5 from the focus. We have two parabolas, one
concave rightwards and the other concave leftwards. The vertex of the first parabola is (6.5, 4) and its
equation is

2
(y 4) = 2(x 6.5) and it meets the x-axis at (14.5, 0).

The equation of the second parabola is (y 4)2 = 2 (x 7.5).

15). 2 It meets the x-axis at (0.5, 0) and the y-axis at (0, 4 +

   X    

2
Suppose that the equation of a tangent to the    y = 4ax. (i)

is y = mx + c. (ii)

2
The abscissa of the points of intersection of (i) and (ii) are given by the equation (mx + c) = 4ax. But
the condition that the straight line (ii) should touch the parabola is that it should meet the parabola in
coincident points hence discriminant should be zero

2 2 2
(mx 2a) = m c (iii)
c = a/m.

Hence, y = mx + a/m is a tangent to the    y2 = 4ax, whatever be the value of m.

Equation (mx + c)2 = 4ax now becomes (mx a/m)2 = 0.

x = a/m2 and y2 = 4ax

y = 2a/m.

Thus the point of contact of the tangent y = mx + a/m is (a/m2 ,2a/m).

Taking 1/m = t where t is a parameter, i.e., it varies from point to point. The   y2 = 4ax as a
parametric form is given by the co-ordinate (at2, 2at) and we refer to it as point t.

  


Prove that the area of the triangle inscribed in the    y2 = 4ax is a2 |(t1 t2) (t2 t3) (t3 t1)|
where t1, t2 and t3 are the vertices.

 


2 2 2
The three points on the    are (at1 , 2at1), (at2 , 2at2) and (at3 , 2at3).

@ )=&%*+@,=,X&-,%

We shall now obtain the equation of a parabola when the focus is any point and the dircectrix is any line.

Let (h, k) be the focus S and lx + my + n = 0 the equation of the     ZM of a parabola. Let (x, y) be
the coordinates of any point P on the parabola. Then the relation, PS = distance of P from ZM, gives
2 2 2 2
(x h+) + (y k) = (lx + my + n) /(l + m )

2
(mx ly) + 2gx + 2fy + d = 0.

This is the general  


    . It is clear that second-degree terms in the equation of a
parabola form a perfect square.

The converse is also true, i.e. if in an equation of the second degree, the second-degree terms from a
perfect square then the equation represents a parabola, unless it represents two parallel straight lines.


= 

2 2
The general equation of second degree i.e. ax + 2hxy + by + 2gx + 2fy + c = 0 represents a parabola
if 0 and h2 = ab. ( = abc + 2fgh af2 bg2 ch2).

  

Let the vertex be (, ) and the axis to be parallel to the x-axis. Then the equation of parabola is given by
2 2
(y ) = 4a (x ) which is equivalent to x = Ay + By + C.

If three points are given we can find A, B and C.

2
Similarly, when the axis is parallel to the y-axis, the equation of the    is y = Ax + Bx + C.

  


Find the equation of the    whose focus is (3, 4) and    


x t + 5 = 0.

 

Let P(x, y) be any point on the    . Then

2
((x-3) (y+4) )=|x-y+5|/(1+1)

2 2 2
(x 3) + (y + 4) = (x-y+5) /2

x2 + y2 + 2xy 22x + 26y + 25 = 0.

2
(x + y) = 22x 26y 25.

  


Find the equation of the    having focus (6, 6) and verte (2, 2).

 


Let S(6, 6) be the focus and A(2, 2) the vertex of the    . On SA take a point K (x1, y1) such
that SA = AK. Draw KM perpendicular on SK. Then KM is the     of the    .

Since A bisects SK, ((-6+x1)/2,(-6+y1)/2) = (2, 2)

6 + x1 = 4, and 6 + y1 = 4

Or (x1, y1) = (2, 10).

Hence the equation of the     KM is y 10 = m (x + 2). (1)

Also gradient of

SK = (10-(-6))/(2-(-6) )=16/8 = 2; m = (-1)/2

So that equation (1) becomes y 10 = 1/2 (x 2)


or x + 2y 22 = 0 is the directrix.

Next, let PM be a perpendicular on the directrix KM from any point

P(x, y) on the parabola.

Form SP = PM, the equation of the    is

({(x+6)+(y+6)2 } )=(x+2y-22)/((12+22 ) )

or 5 = (x2 + y2 + 12x + 12y + 72) = (x + 2y 22)2

or 4x2 + y2 4xy + 104x + 148 y 124 = 0.

or (2x y)2 + 104x + 148y 124 = 0.

  


If the point (2, 3) is the locus and x = 2y + 6 is the     of a    , find

(i) the equation of the axis,

(ii) the co-ordinates of the vertex,

(iii) length of the   ,

(iv) equation of the   .

 


(i) We know that the axis of a    is the line through the focus

And perpendicular to the directrix.

The equation of any line passing through the focus (2, 3) is

y 3 = m (x 2) mx y = 3 2m

If the line be perpendicular to the directrix x 2y = 6, we have,


m (1/2) = 1 m = 2.

Hence the equation of the axis is y 3 = 2 (x 2) 2x + y = 7.

(ii) The co-ordinates of the point of intersection (say) A of the directrix x 2y = 6 and the axis 2x + y = 7
are obtained by solving the two equations; thus they are (4, 1). Since the vertex is the middle point of A
(4, 1) and the focus S(2, 3); the co-ordinates of the vertex are ((4+2)/2,(3-1)/2), i.e. (3, 1).

(iii) Since OS = ((3-2)2+(1-3)2 )=5,

The length of the   = 4OS = 45.

(iv) Since the    is the line through the focus parallel to the directrix, its equation is x 2y + c =
0, where c is given by 2 2.3 + c = 0, i.e. c = 4.

X  cc
c
Any to y2 = 4ax which passes through the focus is called a    of the    y2 = 4ax.

Let y2 = 4ax be the equation of a    and (at2, 2at) a point P on it. Suppose the coordinates of the
other extremity Q of the focal chord through P are (at12, 2at1).

Then, PS and SQ, where S is the focus (a, 0), have the same slopes

(2at-0)/(at2-a)=(2at1-0)/(at12-a)

2
tt1 t = t1 t2 (tt t1 1 + 1) (t1 t) = 0.
Hence t1 = 1/t, i.e. the point Q is (a/t2, 2a/t).

The extremities of a   of the    y2 = 4ax may be taken as the points t and 1/t.

  


Prove that the circle with any focal chord of the    y2 = 4ax as its diameter touches its
directrix.

 


2 2
Let AB be a   . If A is (at , 2at), then B is (a/t ,-2a/t).

Equation of the circle with AB as diameter is

2 2
(x at ) (x-a/t ) + (y 2at) (y+2a/t) = 0.

For x = a, this gives (a2 (1+t2 )2)/t2 + y2 2ay (t-1/t) 4a2 = 0.

2 2 2
a (t-1/t) + y 2ay(t 1/t) = 0

[y a(t 1/t)]2 = 0, which has equal roots.

Hence x + a = 0 is a tangent to the circle with diameter AB.

  


Find the locus of the centre of the circle described on any    of a   as diameter.

 


Let the equation of the parabola be y2 = 4ax.

Let t1, t2 be the extremities of the   . Then t1 . t2 = 1.

The equation of the circle on t1, t2 as diameter is

(x at22) (x at22) + (y 2at1) (y 2at2) = 0

or x2 + y2 ax (t12 + t22) 2ay (t1 + t2) + a2 t12 t12 + 4a2 t1t2 = 0


2 2 2 2 2
x + y ax (t1 + t2 ) 2ay (t1 + t2) 3a = 0. ( t1t2 = 1)

2 2
If (,) be the centre of the circle, then = a/2 (t1 +t2 ) If (, ) be the centre of the circle, then = a/2
(t12+t22 )

= a (t1 + t2) (t1 + t2)2 =2/a2 t12 + t22 + 2t1t2 =2/a2 2/a-2= 2/a2

2a 2a2 = 2 2 = 2a ( a).

Hence locus of (, ) is y2 = 2a(x a).

X  
 

The focal distance of a point P on the   

y2 = 4ax is the distance between the point P and the focus S, i.e. PS. Thus the  
of P = PS =
PM = ZN = ZA + AN = a + x.

or

2 2
PS = a + at = a(1 + t ).

 
 
      

2
Consider the    y = 4ax.

2 2
If (x1, y1) is a given point and y1 4ax1 = 0, then the point lies on the    . But when y1 4ax1 0,
we draw the ordinate PM meeting the curve in L. Then P will lie outside the    if PM > LM, i.e.,
2 2
PM LM > 0.
Now, PM2 = y12 and LM2 = 4ax1 by virtue of the coordinates of L satisfying the equation of the    .
2
Hence, the condition for P to lie outside the    becomes y1 4ax1 > 0.

Similarly, the condition for P to lie inside the    is y12 4ax1 < 0.

 c
c

 
    %
!    

The combined equation of straight line y = mx + c and   

y2 = 4ax gives us the co-ordinates of point(s) of their intersection. The combined equation m2x2 + 2x (mc
2a) + c2 = 0 will give those roots. The straight line therefore meets the    at two points.

2
Points of Intersection of a straight line with the parabola y = 4ax

Points of intersection of y2 = 4ax and y = mx + c are given by (mx+c)2=4ax

2 2 2
i.e. m x + 2x(mc 2a) + c = 0. (i)

Since (i) is a quadratic equation, the straight line meets the    in two points, real, coincident, or
imaginary. The roots of (i) are real or imaginary according as {2(mx 2a)}2 4m2c2 is positive or negative,
2
i.e. according as amc + a is positive or negative, i.e. according as mc is less than or greater than a,
(taking a as positive).

= 

When m is very small, one of the roots of equation (i) is very large; when m is equal to zero, this root is
infinitely large. Hence every straight line parallel to the axis of the    meets the curve in one point
at a finite distance and in another point at an infinite distance from the vertex. It means that a line parallel
to the axis of the   meets the    only in one point.


%
   

As in the preceding article, the abscissae of the points common to the straight line y = mx + c and the
parabola y2 = 4ax are given by the equation m2x2 + (2mx 4a) x + c2 = 0.

Hence, the required 


  

  


Find the Length of the chord intercepted by the parabola y2 = 4ax from the line y = mx + c. Also
find its mid-point. Solution:

Simply by applying the formula o length of the joining (x1, y1) and (x2, y2) we get,

Length of the chord = ((x1-x2 )2+(y1-y2 )2 )

2 2 2
= ((x1-x2 ) +m (x1-x2 ) )

= |x1 x2| (1+m2 ) = 4 (a(a-mc) ) (1+m2 )

2 2 2
[ x1+x2=(-2(m-2a) )/m and x1 x2=c /m ]

The midpoint of the  is ((2a-mc)/m2 ,2a/m)

@

     

2
Let P(x1, y1) and Q(x2, y2) be two neighbouring points on the    y 4ax. Then the equation of
the line joining P and Q is
y y1 = (y2-y1) / (x2-x1 ) (x x1) (1)

Since, points P and Q lies on the parabola, we have

y12 = 4ax1 (2)

y22 = 4ax2 (3)

(equation 3 and 2) give

2 2
y2 y1 = 4a(x2 x1)

(y2-y1)/(x2-x1 )=4a/(y1+y2 )

Equation of chord PQ (i.e. equation (1) becomes):

y y1 = 4a/(y1+y2 ) (x x1) (4)

Our aim is to in the equation of tangent at point P. For that, let point Q approach point P i.e. x2 ] x1 and
y2 ] y1.

y y1 = 4a/(2y1 ) (x x1)

y1y = 2a (x1 + x) (using equation (2))

This is the required equation of the



     y2 4ax at P(x1, y1).


= 

The angle between the



 drawn to the two    at the point of their intersection is
defined as the angle of intersection of two    .

@

  
#./.$

2
Let the equation of the    be y = 4ax.

Hence, value of dy/dx at P(x1, y1) is 2a/y1 and the equation of the

at P is

2
y y1 = 2a/y1 (x x1) i.e. yy1 = 2a(x x1) + y1 .
yy1 = 2a(x + x1).

Alternatively, we write the  


   joining the points P(x1, y1) and Q(x2, y2) on the
parabola y2 = 4ax. Equation of the chord is (x-x1)/(x2-x1 )=(y-y1)/(y2-y1 )

or (x-x1)/(x2-x1 ) = (y-y1 )(y2+y1 )/(y22-y12 )= (y-y1 )(y2+y1 )/((x2-x1 ) )

or 4a(x x) = (y y1) (y2 + y1).

When the two points P and Q tend to coincide, y2 ] y1 and the line PQ becomes

  
   . Its equation is 4a (x x1) = (y y1) (2y1) = 2yy1 2y12= 2yy1 8ax1 or yy1 = 2a(x + x1).

@


 

Suppose that the equation of a tangent to the    y2 = 4ax (i)

is y = mx + c. (ii)

The abscissae of the points of intersection of (i) and (ii) are given by the equation (mx + c)2 = 4ax. But
the condition that the straight line (ii) should touch the parabola is that it should meet the    in
coincident points

(mx 2a)2 = m2c2 (iii)

c = a/m. (iv)

Hence, y = mx + a/m is a tangent to the    y2 = 4ax, whatever be the value of m.

2 2
Equation (mx + c) = 4ax now becomes (mx a/m) = 0.

x = a/m2 and y2 = 4ax y = 2a/m.

2
Thus the point of contact of the

y = mx + a/m is (a/m ,2a/m).
  


2
Find the condition that the line y = mx + c may touch the   y = 4ax and also find its point
of contact.

 


Equation of   is yy2 = 4ax (1)

Slope of

at any point is

dy/dx=2a/y = m (say) (2)

y = 2a/m

from (1), x = a/my2

2
point of contact is (a/my ,2a/m)

Equation of

is

2
y 2a/m = m (x-a/my )

or y = mx + a/m

Therefore, the condition that y = mx + c touches the  

yy2 = 4ax is c = a/m.

  


Find the equation of normal to the   yy2 = 4ax, having slope m.

 


Slope of

at any point is

dy/dx=2a/y

Slope of normal at that point is


-y/2a = m (say)

Point of contact of a normal having slope m with the  

2 2
yy = 4ax is (amy , 2am)

So, equation of normal at this point is

2
y + 2am = m (x amy )

or y = mx 2am amy3.

  


If the line 2x + 3y = 1 touches the   yy2 = 4ax, find the length of its latus rectum.

 


Equation of any

to yy2 = 4ax is

2
y = mx + a/m my x my + a = 0.

Comparing it with the given



2x + 3y 1 = 0, we find

2
my /2=(-m)/3=a/(-1), m = (-2)/3, a = m/3 = -2/9.

Hence the length of the latus rectum.

= 4a = 8/9, ignoring the negative sign for length.

Alternative Solution:

Writing the given equation as

y = -2/3 x + 1/3=-2/x x-(2/9)/(-2/3), we find that a = 2/9.

Hence the length of the latus rectum = 4a = 8/9.



at the point t

2
Equation of the

at t is ty = x + aty . T
the point of intersection of the

s at t1 and t2 is (at1t2, a(t1 + t2)).

  


One the   yy2 = 4ax, three points E, F, G are taken so that their ordinates are in G.P. Prove
that the

 at E and G intersect on the ordinate of F.

 

Let the points E, F, G be (at1y2, 2at1), (at2y2, 2at2), (at3y2, 2at3) respectively. Since the ordinates of these
points are in G.P., t22 = t1t3.

 at E and G are t1y = x + at12 and t3y = x + at32. Eliminating y from
2
these equation, we get x = at1t3 = at2 . Hence the point lies on the ordinates of F.

  


Prove that the area of the triangle formed by three points on a    is twice the area of the
triangle formed by the

 at these points.

The intersection of the



, at these points, are the points

(at1, t2, a(t1 + t2)}, {at2t3, a(t2 + t3)}, {at3 t1, a(t3 + 1)}.

The area of the formed by these points=1/2 a2(t1 t2) (t2 t3) (t3 t1).


 
 @


 



2
Let y = mx + a/m be any tangent to y = 4ax passing through the point (x1, y1).

Then, we have

y1 = mx + a/m or m2x1 m1 + a = 0

If m1 and m2 are to roots of (i) then


m1 + m2 = y1/x1 and m1m2 = a/x1

Also the two tangents are y = m1x + a/m1 , and y = m2x + a/m2

Their combined equation is

(y m1x a/m1 ) (y m2x a/m2 ) = 0

On solving this we get

(y2 4ax) (y12 4ax1) = [yy1 2a (x + x1)]2

SS1 = t2.

Where S = y2 4ax, S1 = y12 4x1

2
Let y = 4ax be the equation of a    and (x1, y1) an external point P. Then, equation of
the

is given by

2 2
SS1 = t2, where S = y 4ax, S1 = y1 4ax1, T = yy1 2a(x + x1).

If the

 from the external point (x1, y1) touch the    at P and Q, then PQ is the chord of
contact of the tangents.


  


Prove that through any given point (x1, y1) there pass, in general, two

 to the    y2 =
4ax.

 


The equation to any



is y = mx + a/m. (1)

If this passes through the fixed point (x1, y1), we have

y1 = mx1 + a/m, i.e. m2x1 m y1 + a = 0. (2)

This is a quadratic equation and gives two values of m (real or imaginary). Corresponding to each value of
m we have, two different tangents. The roots of (2) are real and different if y12 4ax1 > 0, i.e. If the point
2
(x1, y1) lies outside the curve. The roots are equal, if y1 4ax1 = 0 i.e. if the point (x1, y1) lies on the curve.
2
In this case the two tangent merge into one. The two roots are imaginary if y1 4ax1 < 0, i.e. if the point
(x1, y1) lies within the curve.

 

The chord joining the points of contact of the tangents on the    from an external point is
called the chord of contact.

Let the tangent drawn from the point P(x1, y1) touch Parabola at Q(x2, y2) and R(x3, y3) then QR is
the  
of the point P(x1, y1) with respect to y2= 4ax.

The equation of tangents at Q and R are

yy2 = 2a(x + x2) (1)

yy3 = 2a(x + x3) (2)

Since (ii) and (iii) pass through (x1, y1) so we have

y1y2 = 2a(x1 + x2) (3)

y1y3 = 2a(x1 + x3) (4)

From (ii) and (iv) we find that the points Q(x2, y2) and R(x3, y3) lie on yy1 = 2a (x2+ x1), which being of first
degree in x and y represents a straight line. Hence the equation of the  
of P(x1, y1) is

yy1 = 2a (x + x1) and is of the form T = 0.

Equation of the chord of contact of the tangents drawn from a point (x1, y1) to the   y2 = 4ax is T =
0, i.e. yy1 2a(x + x1) = 0.


= 

The equation of the chord of the    y2 = 4ax with mid point

(x1, y1) is T = S1.


  


2
Find the equation of the chord of the    y = 12x which is bisected at the point (5, 7).


 


2
Here (x1, y1) = (5, 7), and y = 12x = 4ax a = 3.

The equation of the chord is S1 = T

2
or y1 4ax1 = yy1 2a(x + x1) or (7)2 12.5 = y(7) 6 (x + 5).

Or 6x + 7y + 19 = 0.

=      

=   
#./.$

The equation of the tangent at the point (x1, y1) is yy1 = 2a(x + x1). Since the slope of the tangent = 2a/y1,
slope of the normal is y1/2a. Also it passes through (x1, y1).

Hence, its equation is y y1 = -y1/2a (x x1). (i)

= 
 

In equation (i), put -y1/2a = m so that y1 = 2a and x1 = (y12)/4a = am2, then the equation becomes y = mx
3
2am am

2
where m is a parameter. Equation (ii) is the normal at the point (am , 2am) of the   


= 

3
If this normal passes through a point (h, k), then k = mh 2am am .

For a given    and a given point (h, k), this cubic in m has three roots say m1, m2, m3 i.e. from (h, k)
three
  can be drawn to the parabola whose slopes are m1, m2, m3. For this cubic, we have m1+
m2+ m3 = 0, m1 m2 + m2 m3 + m3 m1 = (2a h)/a, m1 m2 m3 = k/a.

If we have an extra condition about the


  drawn from a point (h, k) to a given    y2 = 4ax
then by eliminating m1, m2, m3 from these four relations between m1, m2, m3, we can get the locus of (h,
k).

Since the sum of the roots is equal to zero, the sum of the ordinates of the feet of the
  from a
given point is zero. These points are called = 
.

  


Find the locus of the point of intersection of two


      which are at right angles to
one another.

 


The equation of the


      y2 = 4ax is

3
y = mx 2am am .

It passes through the point (h, k) if

k = mh 2am am3 => am3 + m(2a h) + k = 0. (1)

Let the roots of the above equation be m1, m2and m3. Let the perpendicular
  correspond to the
values of m1 and m2 so that m1 m2 = 1.

From equation (1), m1 m2 m3 = -k/a. Since m1 m2 = 1, m3 = k/a.

Since m3 is a root of (1), we have a (k/a)3+k/a (2a h) + k = 0. k2 + a(2a h) + a2 = 0

2
k = a(h 3a).
Hence the locus of (h, k) is y2 = a(x 3a).

=   
 

The normal, being perpendicular to the tangent at (at, 2at) is given by y = tx + 2at + at3.

= 

If normal at the point at1 meets the parabola again at the point at2, then at2 = at1 2/at1 .

Point of intersection of the


  to the    y2 = 4ax (at12, 2at1) and (at2t2, 2at2) is
(2a+a(t12+t22+t1t2), a t1t2(t1+t2)).


  


Prove that the


       at the point whose ordinate is equal to the abscissa
subtends a right angle at the focus.

 


2
If the
      y = 4ax at P(at1t2, 2at1) meets it again at the point t2, then we have
t2 = t1 2/t1 .

If the abscissa and the ordinates of P be equal, then at12 = 2at1

t1 = 2 (rejecting t1 t= 0) 2 = 2 1 = 3

The co-ordinates of P and Q are therefore (4a, 4a) and (9a, 6a) respectively.

The focus is the point S (a, 0).

Slope of PS = and slope of QS = .

pPSQ = right angle. Hence the result.

  

2
Find the locus of the middle points of the normal chords of the    y = 4ax.

 


Equation of the normal chord at any point (at2, 2at) of the    is

y + tx = 2at + at3. (1)

Equation of the chord with mid point (x1, y1) is T = S1

or yy1 2a(x + x1) = y12 4ax1 or yy1 2ax = y12 2ax1. (2)

3 2
Since equations (1) and (2) are identical, 1/y1 =t/(-2a)=(2at+at )/t = 2a + ((-2a)/y1 )

or -(y12)/2a + x1 = 2a + 4a3/(y12 ) or x1 2a = (y12)/2a+4a3/(y12 )

Hence the locus of the middle point (x1, y1) is

x 2a = y2/2a+4a3/y2 .



  


P and Q are the points t1 and t2 on the    y2 = 4ax. If the


  to the    at P
and Q meet at R, (a point on the parabola), show that t1t2 = 2.

 


Let the
  at P and Q meet at R(at2, 2at).

Then t = t1 2/t1 and t = t2 2/t2 .

Therefore t1 + 2/t1 = t2 + 2/t2 (t1 t2) = 2(t1-t2 )/(t1 t2 ) t1t2 = 2.



  


Find the equations of the


      y2 = 4ax at the extremities of its latus
rectum. If the
  meet the    , again at P and Q, prove that PQ = 12a.
 


The ends of the latus rectum are (a, 2a) and (a, 2a). The equations of the
   
   at these points are (put t = 1 and 1)

y + x = 3a and y x = 3a.

These lines meet the    again at P(9a, 6a) and Q(9a, 6a) respectively.

PQ = 6a + 6a = 12a.

 

    

c

ccccccccccccccccccc c
c
c
c
(i) The tangent at any point P on a    bisects the angle between the focal chord through P and the
perpendicular from P on the directrix.

The tangent at P (at2, 2at) is ty = x + at2.

It meets the x-axis at T(at2, 0).

Hence ST = a (1 + t2).

2 2 2 2 2 2
Also, SP = (a (1+t ) +4a t ) = a(1 + t ) = ST, so that

pMPT = pPTS = pSPT TP bisects pSPM.


(ii) @ 
 

        !
    
   
 
  
   

2 2
Let P(at , 2a), be a point on the parabola y = 4ax.

The tangent at P is ty = x + at2.

Point of intersection of the tangent with the directrix x + a = 0 is (a, at a/t).

Now, slope of SP is (2at-0)/(at2-a)=2t/(t2-1)

2
and slope of SK is (at-a/t-0)/(-a-a)=-(t -1)/2t

(Slope of the SP).(Slope of SK) = 1.

Hence SP is perpendicular to SK i.e. pKSP = 90.

(iii) @

    
  
    

    

2
Let P(at , 2at) and P(at12, 2at1) be the end points of a focal part on the parabola. Then t.t1 = 1.
Equations of the tangents at the point P and the point P are ty = x + at2 and t1y = x + at12 respectively.

Let these tangents intersects at a point (h, k). Then h = att1 and k = a(t + t1).

Since the tangents are perpendicular, tt1 = 1 h a.

Hence the locus of the point (h, k) is x = a which is the equation of the directrix.

(iv) 


     
 
  
    
 

 
 

Equation of the perpendicular to the tangent ty = x + at2 (1)

From the focus (a, 0) is tx + y = at. (2)

and (2) intersect at x = 0 which is the equation of the tangent at the vertex.


   

The locus of the point of intersection of tangents drawn at the extremities of the chord of the conic
drawn through a point is called the polar of that point with respect to the conic. This point itself is called
the pole.

Equation of the polar of a point (x1, y1) with respect to the parabola y2 = 4ax.

Let us draw the chord QR from the point P(x1, y1) and if the tangents drawn from point Q and R meet
at the point T(h, k), required locus of (h, k) is polar. Since QR is the chord of contact of tangents from (h,
k), its equation is

ky = 2a(x + h)

This straight line passes through the point (x1, y1), we have

ky1 = 2a(x1 + h) (1)

Since the relation (1) is true, it follows that point (h, k) always lies on the line.

yy1 = 2a(x + x1) (2)

Hence (2) is the equation to the polar of pole (x1, y1)



 


The three points on the    , the normals at which pass through a common point, are called
the 
 
.


  

The locus of the middle point of a system of parallel chords of a    is called its diameter.cc

c
cc c
c
c
cccccccLet the    be

2
y = 4ax. (i)

let y = mx + c (ii)

be a system of parallel chords to (i) for different chords, only c varies, m remains constant.

y2 = 44a (y c)/m

my2 4ay + 4ac = 0

let y2 and y3 be the roots of (iii), then y2 and y3 are the ordinates of the points where (ii) cuts (i)

from (iii), y2 + y3 = 4a/m

Also, if (x1, y1) be the midpoint of the chord them

y1 = (y2-y3)/2=2a/m

Locus of (x1, y1) is y = 2a/m, which is the equation of the diameter.

= 

y = 2a/m is a straight line parallel to the axis of the    .c

 
c
 .

Find the equation of the    whose focus is (3, 4) and directrix is the line parallel to 6x 7y +
9 = 0 and     passes through point (3/2,2).

 


Let (x, y) be any point on the    . Then by definition, the distance between (x, y) and the focus
(3, 4) must be equal to the length of perpendicular from (x, y) on    . So first we will find the
equation of the directrix.

The line parallel to 6x 7y + 9 = 0 is

6x 7y + 6 = 0 (1)

Since     passes through (3/2,2), this point wil satisfy equation 91) hence

6 (3/2) 7 (2) + k = 0

K = 9 + 14 = 5.

Equation of directrix is 6x 7y + 5 = 0

Now by definition of    ,

2 2 2 2
({(x-3) +(y+4) } )= (6x-7y+5)/(6 +7 )

85 {(x 3)2 + (y + 4)2} = (6x 7y + 5)2

2 2
49x + 36y + 84xy 570x + 750y + 2100 = 0


 0

Find the equation of the    whose     makes an isosceles right angled triangle of area 4
square units with the axis in the 3rd quadrant and focus is on the line y = x, 2 units away from the origin.

 


First we find the equation of    . Let the directrix form the isosceles triangle OAB with OA = OB
= a.
Then according to the given condition,

( OAB) = 4

1/2 a2 = 4

a = 22 { the triangle in 3rd quadrant=-22}

Therefore the co-ordinate, of A and B are (22, 0) and (0, 22) respectively.

So, equation of directrix

(y 0) = ((0+22)/(-22-0)) (x + 22) x + y + 22 = 0

Now the focus S is on line y = x and 2 units away from the origin i.e.

OS = 2 point (2, 2) by definition of parabola, we have

((x-2)2+(y-2)2 )=|(x+y+22)/(12+12 )|

x2 + y2 82x 82y 2xy = 0c

R c cc
c
ccccccccccc
cc ccc c'c(cc
'cc
cc!)cc;"c*c cc!)cc;"c
cc#$ c *cccc'cc)*c;*ccc*cccc ccc'c
(!
*c "ccc' c%ccccc   cc 
cc
c
  cc
c
ccccccc+cc)c
cc!)cc;"cc,,c!#"cc
c
ccccccc cc!)cc;"ccc#$ c c,,c! "cc
c
ccccccc+c c ccc)cc *cc
c
ccccccc%c!#"c-cc
$!)cc;c"cc
c
ccccccc(cc) cc! "cc
c

c
c
cc%c.c!/"ccc ccccc c c 
cc)
ccc'c
cccc
c
!!) cc;cc;c"$*!) c c;c"$ "cc
c
c
c
 1

Find the equation of the common



 to the parabola y2 = 32x and x2 = 108y.

 

The equation of the tangent to the parabola y2 = 4ax, is

y = mx + a/m (1)

The equation of tangent to the parabola

y2 = 32x (2) is

y = mx + 8/m (3)

If this line given by (3) is also a tangent to the   

x2 = 108y, then (3) meets x2 = 108y (4)

in two coincident points

Substituting the value of y from (3) in (4) we get

x2 = 108 [mx+(8/m) ]

2 2
mx 108m x 864 = 0

The roots of this quadratic are equal provided b2 = 4ac.

2 2
i.e. ( 108 m ) = 4m (864)

m = (-2)/3 (m 0, from geometry of curves)

Substituting their value of m in (3), the required equation is

y = (-2)/3 x + 8/(2/3)

y = (-2)/3 x 12

2x + 3y + 36 = 0

 2

Find the equation of the



     y2 = 6x, which is parallel to the line cutting
intercepts 3 and 4 on x and y axis respectively.
 


The line cutting intercepts 3 and 4 on x and y-axis respectively is,

x/3+y/4 = 1

4x + 3y = 12 (1)

Slope of line (1) is 4/3

The equation of the parabola is y2 = 4 (3/2) x (2)

The equation of any tangent to (2) is

y = mx + (3/2m) (3)

for all value of m

If this tangent is parallel to line (1), then

m = (-4)/3

Hence from (3) required equation of the tangent is

y = (-4)/3 x + (3/2(-3)/4)

y = (-4)/3 x-9/8

32x + 24y + 27 = 0

 3

Find the angle of intersection of the    y2 = 8x and x2 = 27y.

 


The given parabolas are

y2 = 8x (1)
2
and x = 27y (2)

Solving (1) and (2) we get

2 2
(x /27) = 8x

x4 = 5832x

4
x 5832x = 0

x(x3 5832) = 0

x = 0, x = 18

Substituting these values of x in (2) we get y = 0, 12

The point of intersection are (0, 0), (18, 12)

At the point (0, 0)

2
y = 8x dy/dx=4/y dy/dx|(0,0) =

x2 = 27y dy/dx=2x/27 dy/dx|(0,0) = 0

the two curves intersect at the point (0, 0) at right angle.

At the point (18, 12)

2
y = 8x dy/dx|(18,12) =4/12=1/3 = m1 (say)

x2 = 27y dy/dx=2x/27 dy/dx|(18,12)= (218)/27=4/3 = m2 (say)

Let be the angle at which the two curves intersect at the point (18, 12)

Then tan acute = |(m2-m1)/(1+m1 m2 )|=3/13

acute = tan-1 (3/13)

 4

2 2
Prove that (x + a) = (y 4ax), is the   

 
of the tangents to the
2
parabola y /4.@= 4ax, which includes an angle

 


2
Let two tangent to the parabola y = 4ax (1)

be yt1 = x + at12 (2)

2
and yt2 = x + at2 (3)

Let the point of intersection of the tangent be (x1, y1) then solving equation (1) and (2) we get,

x1 = at1 t2

y1 = a(t1 + t2)

Also the slope of these



 are 1/t1 and 1/t2

If be the angle between these two tangents then

tan = + ((m1-m2)/(1+m1 m2 ))=((1/t1 -1/t2 ))/(1+1/t1 1/t2 ).

tan = ((t2-t1)/(1+t2 t1 ))

we are given = /4

tan /4 = 1 = ((t2-t1)/(1+t2 t1 ))

2
(1 + t1t2) = (t2 t1)

{1+(x1/a)2 } = (t1 t2)2 4t1t2 = (y1/a)2-4x1/a

(x1 + a)2 = y12 4ax1

Required locus of (x1, y1) is (x + a)2 = (y2 4ax)

 5

Prove that normal at one end of    of a    is parallel to the tangent at the other
end.
 


2
Let the parabola be y = 4ax (1)

The end points of latus rectum are (a, 2a) & (a, 2a) and

The equation of the normal to (1) at (a, 2a) is

(y 2a) = (x a)

y = x + 3a (2)

The equation of the tangent to (1) at (a, 2a)

y + 2a = (x a)

y=xa

and from (2) and (3), we find that the slope of normal to one end of the latus rectum is equal to the slope
of tangent at other end of tangent to the other end. Hence the required fact is proved.

 .6

Find the length of the    of parabola y2 = 4ax whose one end point is P(at2, 2at)

 


Let Q (at12, 2at) be the other end of this   .

The equation of the line joining (at2, 2at) and (at12, 2at1) is

(y 2at) = (2at1-2at)/(at12-at2 ) (x at2)

If this passes through the focus (a, 0), then

2at = 2/(t1+t) (a at2)

t (t + t1) = 1 t2

tt1 = 1
t1 = 1/t

2
Then Q is the point (a/t ,(-2a)/t)

The length of the focal chord PQ

= a |t+1/t| ((t-1/t)2+4)

2
a (t-1/t)

 ..

Prove that the locus of the point of intersection of two mutually perpendicular tangents one to each
of the    y2 = 4a (x + a) and y2 = 4b (x + b), is a line parallel to y-axis.

 


The given parabolas are

2
y = 4a(x + a) (1)

and y2 = 4b(x + b) (2)

Any tangent to (1) is y = m (x + a) + a/m (3)

Similarly any tangent to (2) is y = m (x + b) + b/m (4)

mm = 1

or m = -1/m

(4) becomes y = {(x + b)/m} bm (5)

The required locus is obtained by eliminating m between (3) and (5).

For this subtracting (5) from (3), we get

0 = x (m+1/m)+a(m+1/m)+b(m+1/m)

x+a+b=0
This is the required locus which is parallel to y axis.

 .0

If two

     intercept a constant length on any fixed tangent, find the locus of their
point of intersection.

 


Let yt = x + at2 (1)

2
be a fixed

     y = 4ax (2)

Let the other two tangent to (2) be

2
yt1 = x + at1 (3)

And yt2 = x + at22 (4)

The point of intersection of (1) with (3) and (4) are P {att1, a(t1 + t2)} and Q{at t2, a (t + t2)}

Given PQ is constant or PQ2 = constant

2 2 2 2
a t2 (t1 t2) + a (t1 t2) = constant

a2(t2 + 1) (t1 t2)2 = constant

2
(t1 t2) = constant since t is constant as (1) is a fixed tangent.

(t1 t2)2 = c (say) (5)

Let (x1, y1) be the point of intersection of (3) and (4),

then x1 = at1t2 and y1 = a(t1 + t2) (6)

We know that

(t2 t1)2 = (t1 + t2)2

From equation (5), (6) and 97) we get


2
c = (y1/a) - 4x1/a

2 2
or y1 = 4x1 a a c = 4a (x1+1/4 ac)

the locus of (x1, y1) is

y2 = 4a (x+1/4 ac)

2
The required locus is a parabola whose    is 4a i.e. equal to latus rectum of y = 4ax.

 .7

Find the locus of the poles of normals to    y2 = 4ax.

 


Any
      y2 = 4ax is (1)

y = mx 2am am3 (2)

Let (x1, y1) be the pole of (2) with respect to (1), then (2) is the polar of (x1, y1) w.r.t (1)

i.e.

yy1 = 2a (x + x1)

comparing (2) & (3), we get

2a/m=y1/1=(2ax1)/(-2am-am3 )

Hence we get

x1 = 2a am2 (4)

and y1 = 2a/m (5)

Eliminating m between (4) & (5) we get

y12 (x1 + 2a) + 4a3 = 0

The required locus of (x1, y1) is


(x + 2a) y2 + 4a3 = 0

 .8

Find the locus of the point P if the perpendicular from that point P upon its polar with respect
to    , is of constant length.

 


2
Polar of P (x1, y1) with respect to the parabola y = 4ax is

y1y = 2a (x1 + x)

or y1y = 2ax 2ax1 = 0 (1)

We are given that the distance of P (x1, y1) from line (1) is constant, say

2 2 2
|y1 -2ax1-2ax1 |/(y1 +(2a) ) = (constant)

Locus of (x1, y1) is

2 2 2 2 2
(y 4ax) = (y + 4a )

 .1

Find the locus of the point, from which the three normals to the parabola y2 = 4ax cut the axis at
points whose distance from the vertex are in A.P.

 


Any normal to the parabola y2 = 4ax is y = mx 2am am3 (1)

If (1) passes through (x1, y1) then

y1 = mx1 2am am3

3
am + m (2a x1) + y1 = 0 (2)
m1 + m2 + m3 = 0 (3)

m1m2 + m2m3 + m3m1 = ((2a-x1 ))/a (4)

m1 m2 m3 = (-y1)/a (5)

Also, the normal with slope m1, i.e. y = m1x 2am1 am13 cuts axis of the   at the point A(2a
2 2 2
+ am1 , 0) similarly, normals with slope m2 and m3 cut the axis at B(2a + am2 , 0) and c(2a + am3 , 0).

OA, OB and OC are in A.P. (given) where O is origin or vertex of the parabola.

2OB = OA + OC (6)

2m22 = m12 + m32

2
(m1 + m2 + m3) 2 (m1 m2 + m2 m3 + m3 m1)

= 0 2 ((2a-x1)/a)

2
m2 = 2/3 ((x1-2a)/a) (7)

From equation (3), we have

2 2
m2 = (m1 + m3)

= m12 + m32 + 2 m1 m3

2
= 2m2 + 2 (-y1/am2 ) (using 6 and 5)

m32 = -2y1/a (8)

Cubing equation (7) and squaring equation (8) we get

27 ay12 = 2(x1 2a)3

Locus of point (x1, y1) is

27 ay2 = 2(x 2a)3


 .2

Show that the locus of the mid point of all   , of a    is also a parabola.

 


9 .

Let the    be given by

y2 = 4ax

Then its focus is (a, 0). Let (x1, y1) be the mid point of a chord of the given parabola. Then it equation is

S1 = T

2
or y1 4ax1 = y.y1 2a (x + x1)

It passes through the focus (a, 0) of (1), then

2
y1 4ax1 = 0.y1 2a (a + x1)

y12 = 2a(x1 a)

The required locus of (x1, y1) is

y2 = 2a (x a), which is a    .

9 0

Equation of chord AB is

y (t1 + t2) = 2(x + at1t2) (1)

It passes through the focus (a, 0) (2)

0 = 2(a + at1 + t22) t1t2 = 1

2h = a(t12 + t22) and k = a (t1 + t2) (3)

2h = a((t1 + t2)2 2t1t2)


2
= a ((k/a) +2) using (2) and (3)

2
k = 2a (h a)

Locus of M (h, k) is

y2 = 2a (x a)



 .3

2 2
A tangent to the parabola y + 12x = 0 cuts the    y = 4ax at P and Q. Find the locus of
middle points of PQ.

 


2
Any tangent to the    y = 4bx is y = mx b/m

y = mx 3/m

2
Let (x1, y1) be the mid point of PQ, where P and Q are point of intersection o line (1) and y = 4ax

Equation of chord PQ is

S1 = T

y12 4ax1 = y1y 2a(x + x1)

2
y1y 2ax y1 + 2 ax1 = 0 (2)

Equation (1) can be written as

2
my m x + 3 = 0 (3)

equation (2) and (3) represent the same line

2 2
m/y1 =m /2a=3/(2ax1-y1 ) (4)

m = 2a/y1 (from 4)

Again, from (4), we get


m/y1 =3/(2ax1-y12 )

2a/y1 =3/(2ax1-y12 )

Locus of (x1, y1) is

4a2x = y2 (3 + 2a)



 .4

The normal at any point P of the y2 = 4ax meets the axis in G and to the

at the   at H.
If A be the vertex and the rectangle AGQH be completed, prove that the locus of Q is x3 = 2ax2 + ay2.

 


Any normal to the    y2 = 4ax (1)

3
is y = mx 2am am (2)

This normal (2) meets the axis y = 0 of (1) in G and the



at the vertex i.e. x = 0 in H

2 3
The coordinates of G and H are (2a + am , 0) and (0, 2am am ) respectively. Also the vertex A is (0,
0). Let Q be (x1, y1)

Given that AGQH is a rectangle. AQ and GH are its diagonals and therefore there mid points are same.
Now the mid point of AQ is (1/2 x1,1/2 y1 ) and that of GH is

[1/2 (2a+am2+0) 1/2 (0-2am-am3 ) ]

i.e. [1/2 (2a+am2 )-1/2 (2am+am3 ) ]

The mid points coincide so we have

1/2 x1 = 1/2 (2a + am2), 1/2 y1 = -1/2 (2am + am3)

or x1 = 2a + am2, y1 = (2am + am3)

The required locus of Q is obtained by eliminating m between these.


2 2 2 2 2
Now y1 = (m a ) (2 + m )

2 2 2
= a (m a) [2+am /a]

2
= a (x1 2a) (x1/a)

ay12 = (x1 2a) x12

So the locus of Q is

ay2 + 2ax2 = x3

 
  is one of the easiest topics in the 
 
 of Co-ordinate Geometry in Mathematics.

: :is defined as the locus of a point which moves such that the ratio of its distance (Eccentricity)
from a fixed point (Locus) and a fixed line (Directrix) is less than one i.e. a point moves such that its
distance from a fixed point is always less than the distance from a fixed line, we get a different types of
curve for one value of eccentricity, which are similar for all values of eccentricity less than one. Thus curve
looks like a circle but it is not exactly a circle. Rather it is more like the edges of an egg. And if we plot the
movement of the Earth and other planets around the Sun, it is the same curve satisfying the above
condition of eccentricity less than one. This beautiful curve has been named as " ".

In this chapter we will discuss in detail the nature/properties of this beautiful and important curve. As you
will see, the curve is symmetrical about two axes. We will study the standard form of   where the X
and y-axes will be taken as these axes. The main emphasis in this chapter should be on learning the
properties of . The judgement of using parametric co-ordinates, which can reduce the complexity of
the problem, should also be learnt.

@  
  

1.c -   
 
2.c @


= 
3.c  

 
4.c   
-   
 
If we slice an egg obliquely there will appear a typical curve by its edge. We find the similar but larger
curve if we trace the curve of the movement of earth around the sun. Our mathematicians and scientist,
named this curve as the  .   one of the conic sections is obtained by cutting one nappe of
cone with a plane that does not pass through the vertex.




An   is locus of a point, which moves in a plane such that the ratio of its distances from a fixed point
and a fixed line is constant and always less than one.

In other words " " is a conic for which the  


 
e < 1. Let S be the focus of  , P any point on the   and PM perpendicular distance of
the     from P, then

SP/PM = e < 1

Let S be the focus and ZM be the     of the  . Let be its 
 .

We draw SZ perpendicular to the     and divide SZ internally and externally in the ratio e : 1 and let
A and A' be the internal and external point of division.

Then we have SA = e AZ ...... (1)

And SA' = e A'Z ...... (2)

. . A and A' lie on the  .

Let AA' = 2a and take O the midpoint of AA' as origin. Let P(x, y) be any point on the  referred to
OA and OB as co-ordinate axis.

Then from figure it is evident that


AS = AO - OS = a - OS

AZ = OZ - OA = OZ - a

A'S = A'O + OS = a + OS

A'Z = OZ + OA' = OZ + a

Substituting these values in (1) and (2), we have

a - OS = e (OZ - a) ...... (3)

a + OS = e (OZ + a) ...... (4)

Adding (3) and (4), we get

2a = 2 e OZ

Or OZ = a/e ...... (5)

Subtracting (3) from (4), we get

2 OS = 2ae => OS = ae ...... (6)

.. The     MZ is x = OZ = a/e and the co-ordinate of the focus S are (OS, 0) i.e. (ae, 0). Now
as P(x, y) lies on the  .

So we get

2 2 2
SP = e PM or SP = e PM

(x - ae)2 + y2 = e2 [OZ - x co-ordinate of P]2

2 2 2 2 2
=> (x - ae) + y = e [a/e - x] = (a - ex) ...... (7)

=> x2 + a2e2 - 2axe + y2 = a2 + e2x2 - 2aex

2 2 2 2 2 2 2
or x /a + y /a (1-e ) = 1 [Dividing each term by a (1 - e )]

or x2/a2 + y2/b2 = 1 where b2 = a2 (1 - e2)

This is the standard equation of an  , O is called the centre of the  , AA' and BB' are called
the major and minor axes of   (where b < a).

There exists a second focus and second     for the curve. On the negative side of the origin take
a point S', which is such that SO = S'O = ae and another point Z' such that ZO = OZ' = a/e.
Draw Z'K' perpendicular to ZZ' and PM' perpendicular to Z'K'

The equation (7) may also be written in the form

(x + ae)2 + y2 = (a + ex)2

2 2 2
=> S'P = e (PM')

Hence, any point P on the curve is such that its distance from S' is e times to its distance from Z'K' so
we should have obtained the same curve, if we had started with S' as focus, a Z'K' as     and the
same  
 .



We have considered a > b, now if we consider b > a, what will be the shape of
the   x2/a2 +y2/b2? In this case the major axis AA' of the   is along the y-axis and is of length
2b. See figure.

The minor axis of BB' = 2a. The foci S and S' are (0, be) and (0, -be) respectively. The     are MZ
2 2 2
and M'Z' given by y = + b/e, respectively. Also here a = b (1 - e ).

= 

Let P(x1, y1) be any point. This point lies outside, on or inside the   (8) according as x21/a2 + y21/b2 =
1 > 0 or = 0 or < 0.


  
A curve is said to be a central curve if there is a point, called the centre, such that every chord passing
through it is bisected at it.

%  

The length of a chord through the focus and at right angle to the major axis of the   is known
as the latus rectum of the  .

There being two foci of an  , there are two rectum, which are of equal length.

2
yL = b /a

.. The length of latus rectum LSL' = 2b2/a

= 


c The major axis AA' is of length 2a and the minor axis BB' is of length 2b.

c The foci are (-ae, 0) and (ae, 0).

c The equations of the directrices are x = a/e and x = -a/e.

c The length of the semi latus rectum = b2/a.

c Circle is a particular case of an   with e = 0.

X  
 

Since S'P = ePN', SP = ePN,

S'P + SP = e(PN + PN')


= e (NN') = e(2a/e) = 2a

=> the sum of the focal distances of any point on the   is equal to its major axis.


 


 

2 2 2 2
Let   be x /a +y /b = 1 ...... (i)

Its foci S and S' are (ae, 0) and (-ae, 0). The equation of its directrices MZ and M'Z' are x = a/e
and x = -a/e respectively. Let P(x1, y1) be any point on (i)

Now SP = e PM = e NZ = e (OZ - ON) = e[(a/e)-x1] = a - ex1

and S'P = ePM' = e (Z'N) = e (OZ' + ON) = e[(a/e) + x1] = a + ex1

.. SP + S'P = 2a = AA'

So by this property an   can also be defined as "the locus of a point which moves such that the sum
of its distances from two fixed point is always constant.
, X


c If in the equation x2/a2 +y2/b2 = 1, a2 < b2, then the major and minor axis of the  lie along the y and
the x-axis and are of lengths 2b and 2a respectively. The foci become (0, + be), and the directrices
become y = + b/e where e = (1-a2/b2 ). The length of the semi-lactus rectum becomes a2/b2.

c If the centre of the   be taken at (h, k) and axes parallel to the x and the y-axes, then the equation of
the   is (x-h)2/a2 +(y-k)2/b2 = 1.

c Let eh equation of the     of an   be ax + by + c = 0 and the focus be (h, k).

Let the  
  of the   be e(e < 1).

If P(x, y) is any point on the  , then

2 2 2
PS = e PM

2 2 2 2 2 2
=> (x - h) + (y - k) = e (ax+by+c) /(a + b ), , which is of the form

ax2 + 2hxy + by2 + 2gx + 2fy + c = 0 ... (*) where

= abc + 2fgh + af2 - bg2 - ch2 0, h2 < ab.

These are the necessary and sufficient conditions for a general quadratic equation given by (*) to
represent an  .

 
 
&  
 

The point P(x1, y1) is outside or inside the  x2/a2 + y2/b2 = 1, according as the quantity
((x12)/a2 +(y12)/b2 -1) is positive or negative.
    

 

2 2 2 2
Clearly, x = a cos, y = b sin satisfy the equation x /a +y /b = 1 for all real values of .

Hence, the parametric equations of the   x2/a2 +y2/b2 = 1 are x = a cos,
y = b sinq where is the parameter.

2 2 2 2
Also (a cos , b sin ) is a point on the   x /a +y /b = 1 for all values of
(0 < < 2).

The point (a cos, b sin) is also called the point . The angle is called the eccentric angle of the point (a
cos, b sin) on the  .

Draw a circle with AA' (the major axis) as the diameter. This circle is called the auxiliary circle of the  .
The equation of the circle is x2 + y2 = a2. Any point Q on the circle is (a cos, a sin). Draw QM as
perpendicular to AA' cutting the  at P. The x-co-ordinate of P = CM = a cos.

=> y-co-ordinate of P is b sin

=> P (a cos, b sin).

`   

2 2
Find the centre, the length of the axes and the  
  of the   2x + 3y - 4x - 12y + 13 = 0.

i  

2 2
The given equation can be written as 2(x - 1) + 3(y - 2) = 1

=>(x-1)2/(1/2)+(y-2)2/(1/3) = 1 => The centre of the   is (1, 2).

The major axis = 2. 12 = 2.


The minor axis = 2.1/3=2/3 => e2 = 1 1/2 = 1/3 => e = 1/3.

`   

Find the  


   whose foci are (2, 3), (-2, 3) and whose semi minor axis is of length 5.

i  

Here S is (2, 3), S' is (-2, 3) and b = 5.

=> SS' = 4 = 2ae ] ae = 2.

But b2 = a2 (1 - e2) => 5 = a2 - 4 => a = 3.

Centre C of the   is (0, 3).

Hence the  


   is (x-0)2/9+(y-3)2/5 = 1

2 2
=> 5x + 9y - 54y + 36 = 0.

`   

Find the  


    having centre at (1, 2), one focus at (6, 2) and passing through the point
(4, 6).

i  

With centre at (1, 2) the  


   is (x-1)2/a2 +(y-2)2/b2 = 1. It passes through the point (4,
6).

=> 9/a2 +16/b2 = 1. ...... (1)

Distance between the focus and the centre = (6 - 1) = 5 = ae

2 2 2 2 2
=> b = a - a e = a - 25. ...... (2)

Solving for a2 and b2 from the equations (1) and (2), we get a2 = 45 and b2 = 20.

Hence the  


   is (x-1)2/45+(y-2)2/20 = 1.

`   
Find the  
    (in standard form) having latus rectum 5 and 
  2/3.

i  

2 2 2 2
Let the   be x /a +y /b = 1 with a > b.

Latus rectum = 5 = 2b2/a => 2b2 = 5a. ...... (1)

2 2 2 2 2
Also b = a (1 - e ) = a (1-4/9) = 5a /9

=> 5a/2 = 5a2/9 => a = 9/2 and hence b2 = 5/2a = 45/4.

2 2
The  
   , in the standard form, is thus x /(81/4)+y /(45/4) = 1.

`   

Find the equation of the ellipse, which cuts the intercept of length 3 and 2 on positive x and y-axis.
Centre of the ellipse is origin and major and minor axes are along the positive x-axis and along positive y-
axi

i  

x2/a2 +y2/b2 = 1 ...... (1)

According to the given condition the ellipse (1) passes through (3, 0) and (0, 2), so we have.

2 2
9/a = 1 => a = 9

2 2
And 4/b = 1 => b = 4

Therefore, the equation of the ellipse is x2/9 + y2/4 = 1

`   

Obtain the equation of an ellipse whose focus is the point (-1, 1) whose directrix is the line passing
through (2, 5) having the unit gradient and whose eccentricity is .

i  

Let P(x, y) be any point on ellipse.

Its focus is S (-1, 1).


Let the directrix be y = x + c ...... (1)

( . gradient m = 1)

Line (1) passes through (2, 5) so,

5 = 2 + c => c = 3

The directrix is y = x + 3

=> x - y + 3 = 0 ...... (2)

Now let PM be the perpendicular from P, drawn to its directrix

(2). By definition of ellipse SP = e PM

2 2 2
or SP = e PM

=> (x + 1)2 + (y - 1)2 = (1/2)2 [(x-y+3)/((12+12 ))]2

2 2 2
=> 8[(x + 1) + (y - 1) ] = (x - y + 3)

=> 7x2 + 7y2 + 2xy + 10x - 10y + 7 = 0,

This is the required equation of ellipse.

@


= 
ccccccccThe   is x2/a2 +y2/b2 = 1 ...... (1)

Let P(x1, y1) and Q(x2, y2) be two points on the  . The equation of the line PQ is,

y - y1 = ((y2-y1)/(x2-x1 )) (x - x1) ...... (2)

since points P and Q lies on (1), we get

(y2-y1)/(x2-x1 )=(-b2 (x2-x1 ))/(a2 (y2-y1 ) )

So (2) becomes

y - y1 =(-b2 (x2-x1 ))/(a2 (y2-y1)) (x - x1)


As point Q approaches towards point P along the  , the line PQ tends to the

at P. So, by
substituting x1 and y1 for x2 and y2 in the above equation, we have the equation of the

at P as

y - y1 = (-b2 (2x1 ))/(a2 (2y1)) (x - x1)

2 2 2 2 1 2
=> (xx1)/a + yy1/b =(x1 )/a +(y` )/b = 1 [as P lies on (1)]

Hence the equation of the



to x2/a2+y2/b2= 1 at P(x1,y1) is xx1/a2 + yy1/b2= 1. ...... (3)

Equation of the

in terms of 'm'

Let the line y = mx + c ....... (4)

2 2 2 2
Touch the   x /a +y /b = 1 ...... (5)

Eliminating y between (4) and (5), we get;

2 2 2 2 2 2 2 2
x (b + a m ) + 2a mcx + a (c - b ) = 0 ...... (6)

If (4) touches (5) then the roots f (6) must be coincident i.e. D = 0

2 2 2 2 2 2 2 2
i.e. (2a mc) = 4(b + a m ) a (c - b )

solving this we get c = +(a2 m2+b2)

2 2 2
So the equation of

is y = mx + (a m +b ) for all real m ......(7)

From the equation (6) and (7) we get the point of contact as ((+_a2 m)/(a2

2 2 2 2 2
m +b ),(b )/(a m +b ))

 
 @

 
 

; Let the equation of the   be x2/a2 +y2/b2 = 1 . Slope of



at

2 2
(x1, y1) = dy/dx(x1,y1)=-b /a x2/y1

Hence the equation of the



at (x1, y1) is y - y1 = (-b2 x1)/(a2 y1)(x - x1)

2 2
=> (xx1)/a +(yy1)/b = 1.

; Equation of

at the point q i.e. (a cos, b sin) is obtained by putting x1= a cos, y1 = b sin

=> (x cos )/a+(y sin )/b = 1.




 
 @


@ < 
 
@



To find the condition that the line y=mx+ c may touch the   x2/a2 +y2/b2 = 1.

@

to the   at (a cos, b sin) is (x cos)/a+(y sin)/b = 1.

If y - mx = c is also a

to the given   at 'q', then comparing coefficients, we get

2 2 2 2 2 2 2
(cos)/am = (sin)/b=1/c or 1/c =(sin)/b=(cos)/(-am)=((sin +cos )/(b +(-am) ))=1/(a m +b )

=> c = (a2 m2+b2 ).

2 2 2 2
Therefore, the equation of a

to the   x /a +y /b = 1 is

y = mx (a2 m2+b2 ) for all values of m.

`   

Find the locus of the point of intersection of the



 to the   x2/a2+y2/b2 = 1 (a > b) which
meet at right angles.

i  

The line y = mx (a2 m2+b2) is a



to the given   for all m. Let is passes through (h, k).

2 2 2 2 2 2 2 2 2
=> k - mh = (a m +b ) => k + m h - 2hkm = a m + b

=> m2 (h2 - a2) - 2hkm + k2 - b2 = 0.

If the

 are at right angles, then m1m2 = -1.

=> (k2-b2)/(h2-a2 ) = - 1 => h2 + k2 = a2 + b2.

2 2 2 2
Hence the locus of the point (h, k) is x + y = a + b which is a circle. This circle is called the Director
Circle of the  .

= 
The locus of the point of the intersection of two perpendicular

 to an  is a circle known as
the director circle.

`   

Prove that the locus of the mid-points of the intercepts of the



 to the  x2/a2 + y2/b2 = 1 =
1, intercepted between the axes, is a2/x2 +b2/y2 = 4.

i  

The

to the   at any point (a cos, b sin)(x cos)/a+(y sin )/b = 1.

Let it meet the axes in P and Q, so that P is (a sec, 0)

and Q is (0, b cosec). If (h, k) is the mid-point of PQ, then h = (a sec)/2.

=> cos = a/2h and k = (b cosec)/2 => sin = b/2k.

2 2 2 2
Squaring and adding, we get a /4h +b /(4k ) = 1

2 2 2 2
Hence the locus of (h, k) is a /x +b /y = 4.

`   

Prove that the product of the lengths of the perpendiculars drawn from the foci to any

to
2 2
the   x /16+y /9 = 1 is equal to 9.

i  

2
For the given   a = 4, b = 3 and hence 9 = 16 (1 - e )

=> e = 7/4. The foci are thus located at (7,0) and (-7,0).

Equation of a

to the given   is

y = mx + (16m2+9) (as a = 4, b = 3).

Lengths p1 and p2 of the perpendiculars drawn from the foci are

p1 = ((16m2+9)+7 m)/(1+m2 ) and p2 = ((16m2+9)-7 m)/(1+m2)

2 2 2 2 2
=> p1p2 = (16m +9-7m )/(1+m ) = 9(1+m )/(1+m ) = 9.
= 

The product of lengths of the perpendiculars drawn from the foci to any

to
the   x2/a2 +y2/b2 = 1 is b2.

 
 =  
 

The
  to a curve is a line perpendicular to the

to curve through eh point of contact.

. . The slope of
  at point (x1, y1) = a2y1/b2x1 and so its equation is (x-x1)/((x1/a2))=(y-y1)/((y1/b2 ) ).

; To find the equation of the


  to the   x2/a2 +y2/b2 = 1 at (x1, y1):

2 2
Equation of the

at (x1, y1) is (xx1)/a +(yy1)/b = 1

=> Slope of the


  is a2/b2 +y1/x1; => equation of the
  is

2 2 2 2
y - y1 = a /b +y1/x1 (x - x1) =>(x-x1)/(x1/a )=(y-y1)/(y1/b ).

; Equation of the
  at (a cos, b sin) is (x-a cos)/((a cos)/a2) = (y-b sin )/((b sin)/b2).

2 2
=> ax sec - by cosec = a - b .

ax1 sec - by1 cosec = a2 - b2.

2 2 2 2 2
or ax1((1+t )/(1-t )) - by1 ((1+t )/2t) = a - b , where t = tan /2.

On simplification, this equation gives

4 2 2 3 2 2
by1t + 2(ax1 + a -b )t + 2(ax1 - a + b )t - by1 = 0.

This is a 4th degree equation in t which gives, in general, four values fo t. Hence from a fixed point
four
  can be drawn to the given  .

`   

If the
  to the   x2/a2 + y2/b2 = 1 at the points (x1, y1), (x2, y2) and (x3, y3) are concurrent, prove

that = 0.

i  
The equation of the
  to the given   at (x1, y1) is

a2xy1 - b2yx1 - (a2 - b2)x1y1 = 0. ...... (1)

Similarly the
  at (x2, y2) and (x3, y3) are

a2xy2 - b2yx2 - (a2 - b2)x2y2 = 0. ...... (2)

2 2 2 2
a xy3 - b yx3 - (a - b )x3y3 = 0. ...... (3)

Eliminating a2x, b2y and (a2 - b2) from (1), (2) and (3), we find that the three lines are concurrent

if = 0.

@


= 

`   

2 2 2 2
If the
  at the end of a latus rectum of the   x /a +y /b = 1 passes through the extremity of
a minor axis, prove that e4+e2-1=0.

i  

Equation the
  to te given   at

(ae,b2/a) is (x-ae)/(ae/a2) = (y-b2/a)/(b2/ab2).

Since it passes through (0, - b).

2 3
- a = - ab - b

2 2 2
=> a = ab + a (1 - e )

or b = ae2 => b2 = a2e4

or a2 (1 - e2) = a2e2 a4 + e2 - 1 = 0.

Rcccc c c c cc c c c c c c 

We know that y = mx + (a2 m2+b2) is a



to the   x2/a2 +y2/b2 = 1 for real values of m. If
2 2 2
this

passes through a point (x1, y1) we have y1 = mx1 + ((a m +b )).

or m2(x12 - a2) - 2x1y1m + (y12 - b2) = 0, which being a quadratic equation in m gives two values of m. Thus
from a point two

 corresponding two values of m) can be drawn to an  .


@


  

From a fixed point (x1, y1) in general two



 can be drawn to an  . The equation of the pair
2 2 2 2 2 2 2 2 2 2 2 2
of

 drawn to the   x /a +y /b = 1 is given by (x /a +y /b -1)((x1 )/a +(y1 )/b -1) =
2 2 2
(xx1/a + yy1/b -1) .

@


= 

In symbols we write SS1 = T2, where

2 2 2 2 2 2 2 2 2 2
S x /a +y /b = 1, S1 (x1 )/a +(y1 )/b 1 and T xx1/a + yy1/b 1

2 2 2 2
; If from the point P(x1, y1)

 PQ and PR be drawn to the   x /a +y /b = 1, then the line
joining the points of contact Q and R is called the chord of contact. Equation of the chord of contact is
xx1/a2 + yy1/b2 - 1 = 0 or T = 0.

; Equation of the Chord Joining the Points ( cos, sin),


( cos, sin) is

x/a cos((+)/2)+ y/b sin((+)/2) = cos((-)/2)

; Equation of a chord which is bisected at the point (x1, y1) is xx1/a2 + yy1/b2 -1 = (x12)/a2 +(y12)/b2 - 1 or =
S1

; Length of the chord ... (from package).

0 0 0 0
@
 
   
     = > = ?.
    
?
> 
2 2 2 2
Points of intersection of the   and the line are given by x /a +(mx+c) /b = 1

i.e. (a2 m2 + b2)x2 + 2a2 cmx + a2(c2 - b2) = 0 ...... (1)

Therefore the straight line meets the   in two points (real, coincident or imaginary).

If (x1, y1) and (x2, y2) be the points of intersection, the length of the chord is

2 2 2
((x1-x2 ) +(y1-y2) )=(1+m )= |x1 - x2| ...... (2)

(since y1 - y2 = m (x1 - x2))

where x1 and x2 are the roots of the equation (1), and

x1 + x2 = -(2a2cm)/(a2 m2 + b2), x1 x2 (a2 (c2 - b2))/(a2 m2 + b2) so that

2 2 2 2 2 2 2 2 2 2 2 2 2 2
(x1 - x2) = (x1 + x2) - 4x1 x2 = (4a c m )/(a m + b ) -(4a (c - b ))/((a m

+ b2))=(4a2 b2 (a2 m2 + b2 - c2))/(a2 m2 + b2)2.

2 2 2 2 2 2 2 2 2 2 2
Hence the length of the chord is (((1+m )4a b (a m + b - c ))/(a m + b ) ).

i.e. 2ab/(a2 m2 + b2)2) ((1+m2))(a2) m2)+b2)-c2))).

=> e cos(+)/2 = cos(-)/2

=> e [cos /2 cos /2 + sin /2 sin /2] = cos /2 cos /2 + sin /2 sin /2

=> e [1 - tan /2 tan /2] = 1 + tan /2 tan /2 => tan /2 . tan/2 = (e-1)/(e+1).

If the chord passes through (-ae, 0) then tan/2 . tan/2 = (e+1)/(e-1).

@


= 

`   

The

and
  at a point P on an   meet the minor axis at A and B. Prove that AB
subtends a right angle at each of foci.

i  

The equations of the



 and
  at a point P(x1, y1) on the   x2/a2+ y2/b2 = 1 are

2 2
xx1/a + yy1/b = 1 ...... (1)
2 2
and (x-x1)/((x1/a ))+(y-y2)/((y1/b )) = 1 ...... (2)

Also the minor axis is y-axis i.e. x = 0

2
Solving (1) and x = 0, we have A (0, b /y1)

Solving (2) and x = 0, we have B (0, y1 - a2y1/b2)

2 2
Let S(ae, 0) be one of the foci of the  . Then the slope of SA = ((b /y1) - 0)/(0-ae)= b /aey1 =
m1 (say)

And the slope of SB = ([y1 (a2/b2 ) y1 ]-0)/(0-ae)

2 2 2 2 2 2 2 2 2
= y1/ae ((a -b ))/b = (y1 a e )/(aeb ) [. b = a (1 - e )]

= aey1/b2 = m2 (say)

Evidently m1m2 = -1 => SA | SB

i.e. AB subtends a right angle at S(ae, 0). Similarly we can show that AB subtends a right angle at the other
focus S'(-ae, 0)

`   

Prove that the locus of the middle points of the portion of



to the  x2/a2 +y2/b2 = (a+b)
between the axis is the curve a/x2 + b/y2 = 4/((a+b)).

i  

Any

to the  x2/a2 +y2/b2 = 1 , is

2 2 2
y = mx + (a m + b )

Therefore the

to given   is

2
y = x + (a(a+b) m +b(a+b)) ...... (1)

(1) Meets x-axis at {-(a(a+b) m2+ b(a+b) )/m,0}

2
(2) Meets y-axis at {0,(a(a+b) m +b(a+b))}

Let (x1, y1) be the mid points of the portion of the



intercepted between the axes, then

2
x1 = 1/2 [{-(a(a+b) m + b(a+b))/m} + 0]
2
y1 = 1/2 [0+{(a(a+b) m +b(a+b))} + 0]

=> 4x12 = (a(a+b) m2+ b(a+b))/m2

2 2
and 4y1 = a(a + )m + b(a + b)

Eliminating m, we get

2 2 2 2 2
a(a+b)/(4x1 ) - b(a+b)/(4y1 ) = am /(am +b) + b/(am +b) = 1

.. The locus of (x1, y1) is a/x2 + b/y2 = 4/(a+b) = 1

`   

If PS1Q and PS2R be two focal chord of the   whose two foci are S1 and S2and the eccentric angle of
2 2
P is '' then show that the equation of chord QR is x/a cos + y/b.(1+e )/(1-e ) sin + 1 = 0.

i  

Let Q be (a cos , b sin ) and R be (a cos , b sin ) then the equation of the chord QR is

x/ cos(+)/2 + y/b sin(+)/2 = cos(-)/2

which on simplifying becomes

x/a (1-tan /2 tan /2)+y/b (tan /2+tan /2) = 1 + tan /2 tan /2 ..... (1)

Also, PQ and PR are focal chord thus

tan /2 tan /2 = (e-1)/(e+1) and tan /2 tan /2 = (e+1)/(e-1)

(From previous illustration)

On substituting the values of tan /2 and /2 in (1), we get


@


= 

`   

Show the locus of middle points chord of the  x2/a2 +y2/b2 = 1 which subtend right angle at the
centre is x2/a4 + y2/b4 =(1/a2 +1/b2 ) (x2/a2 + y2/b2)2.

i  

Let (x1, y1) be the middle point of chord PQ, then its equation is

T = S1 or 1/a2 + yy1/b2 =(x1/b2)/a2 +(y1/b2)/b2 ...... (1)

Since the origin 'O' is the centre so the equation of pair of lines OP and OQ can be obtained by
homogenizing the equation of the   x2/a2 +y2/b2 = 1, with the help of (1), thus

2 2 2 2 2
x /a +y /b = (1) =

2 2 2 2 2 2 2 2 2 2 2 4 2 2 4 2 2
or ((x1 )/a +(y1 )/b ) (x /a +y /b ) = (x x1 )/a +(y y1 )/b + (2xyx1 y1)/(a b ).

It represents a pair of perpendicular lines if

1/a2 ((x12)/a2 +(y12)/b2 )2 - (x12)/a4 + 1/b2 ((x12)/a2 +(y12)/b2)2-(y12)/b4 = 0.

2 4 2 4 2 2 2 4 2 4 2
or, (x1 )/a +(y1 )/b = (1/a +1/b )(x /a +y /b ) .

So the locus of (x1, y1) is

x2/a4 + y2/b4 = (1/a2 + 1/b2 ) (x2/a2 +y2/b2 )2.




 

 
c

    


 

     of an   which is a circle described on the major axis of an  as its diameter.

Let the   be

x2/a2 + y2/b2 =1 ...... (1)

Then the equation of its auxiliary circle is x2 + y2 = a2 ...... (2)

Take a point P(x1, y1) on (1).

Through P, draw a line perpendicular to major axis intersecting major axis in N and auxiliary circle in P'.

Let P' be (x1, y2). Then we have (x12)/a2 + (12)/b2 = 1 and 12 + y12 = a2

From these two relations, we get:

2 2 2 2 2 2 2 2 2
(y2 )/(y1 )=(a -1 )/(b {1-(x1 /a )}) = a /b

=> y2/1 = a/b

Now, let OP' make an angle f with the major axis of the   (1), P' being the corresponding point of P
on the      of the  . Then is called the eccentric angle of the point P. From the figure it
is evident that if (x1, y1) are the co-ordinates of P, then x1 = ON = OP' cos = a cos. Also P(x1, y1) is a
point on (1).

So solving(x12)/a2 +(12)/b2 =1 for y1, we get y1 = b sin


Therefore (a cos , b sin ) are the parametric co-ordinates of any point P on the , where is the
eccentric angle of P.

 

 

= 

.@

     


The equation of the line PQ reduces to that of tangent when x/a cos + y/b sin = 1
...... (a)

0=      


@@

Equation of normal can be derived by using the formula of equation of straight line passing through point
(a cos, b sin) and perpendicular to tangent (a) i.e. (y - b sin ) = (a sin /b cos ) (x - a cos ).

2 2
=> a x sec - b y cosec = a - b

  
 

The locus of the middle points of a system of parallel chords of an   is called the   
  .

Let y = mx + c ...... (1)

be the equation of a system of parallel chords of the  x2/a2 + y2/b2 = 1


...... (2)
In (1) m is constant and c varies from chord to chord. Let K (x1, y1) be the midpoint of a chord PQ of this
system.

Eliminating y between (1) and (2) we get

(a2m2 + b2)x2 + 2a2mcx + a2(c2 - b2) = 0 ...... (3)

2 2 2 2
=> x2 + x3 = (-2a mc)/(a m +b )

But x1 = (x2+x3)/2 =(-a2 mc)/(a2 m2+b2)

2 2 2 2 2
Or c = (-x(a m +b ))/(a m )

Also K (x1, y1) is a point on (1) so

2 2
y1 = mx1 + c => y1= -b x1/a m

2 2 2 2 2 2
.. The locus of K (x1, y1) is y = -b x/a m which is a diameter of the   x /a + y /b = 1.

= 

1. Conjugate Diameters: Two   


  which bisects chords parallel to each other are
called conjugated diameters. Therefore the diameters y = mx and y = m1x of the   x2/a2 +y2/b2 = 1
are conjugate if mm1 = -b2/a2.

2. In an  , the major axes bisects all chords parallel to the minor axes and vice-versa, therefore
major axes and minor axes of an   are conjugate diameters but they do not satisfy the condition
mm1 = -b2/a2 and are the only perpendicular conjugate diameters.

7  
A   

If the length of two conjugate      be equal then they are called equi conjugate
diameters.

2 2 2 2
The equation of equi conjugated diameters are x /a y /b = 1.
4. The eccentric angles of the ends of a pair of conjugate   
 differ by a right angle
i.e., if one end of a diameter (PQ) is
P(a sin , b cos ).

5. The sum of the squares of any two conjugate    


  is constant and equal to
the sum of the squares of the
2 2 2 2
semi-axis of the   i.e. OP + OP = a + b .

6. The produce of the focal distances of a point on an   is equal to the square of the semi-
diameters, which is conjugate to the diameter through the point.

7. The tangents at the ends of a pair of conjugate diameters of an   form a parallelogram and the
area of the parallelogram is constant and is equal to the product of the axis i.e. equal to 4ab.

    
 

The director circle is the locus of the point of intersection of pair of perpendicular tangents to an  .

Two perpendicular tangents of   x2/a2 + y2/b2 = 1 are

2 2 2
y - mx = (a m +b ) ...... (1)

and my + x = (a2+b2 m2) ...... (2)

To obtain the locus of the point of intersection (1) and (2) we have to eliminate m squaring and adding (1)
and (2), we get

(y - mx)2 + (my + x)2 = (a2m2 + b2) + (a2 + b2m)

=> x2 + y2 = a2 + b2, which is the equation of the director circle.

Rcccc c cc  c c c c c c c cc cc


c

2 2 2 2
Let the   be x /a + y /b = 1 ...... (1)

Any line through (x1, y1) is y - y1 = m(x - x1) ...... (2)

Eliminating y between (1) and (2) we get

x2 (b2 + a2m2) + 2ma2 (y1 - mx1) x + a2 [(y1 - mx1)2 - b2] = 0

If x2 and x3 are the roots of this equation then

x2 + x3 = -2ma2 (y1 - mx1)/(b2 + a2m2)

But (x1, y1) the mid point of the chord.

.. x1 = (x2+x3)/2=(-ma2 (y1-mx1 ))/(b2+a2 m2)

2 2
=> m = (-b x1)/(a y1)

.. From (2) and (3) the equation of the chord where mid point is (x1, y1) is

2 2
y - y1 = ((-b x1)/(a y1)) (x - x1)

=> xx1/a2 + yy1/b2 =(x12)/a2 +(y12)/b2

2 2
Or T = S1 where T = xx1/a + yy1/b -1

and S1 = (x12)/a2 + (y12)/b2 -1


 
 

The locus of the points of intersection of tangents drawn at the point extremities of the chords passing
through a fixed point is called the polar of that fixed point and the fixed point is called the pole.
Equation of the polar of P(x1, y1) with respect to the  

x2/a2 + y2/b2 = 1 is

2 2
xx1/a + yy1/b =1

`   

How to find out pole of the line lx + my + n = 0 w.r.t. the   x2/a2+y2/b2 = 1.

i  

Let (x1, y1) be the pole of line lx + my + n = 0. ...... (1)

2 2 2 2
w.r.t. the   x /a + y /b = 1 ...... (2)

Now the polar of (x1, y1) w.r.t. (2) is

2 2
xx1/a + yy1/b = 1 ...... (3)

Since (1) and (3) represents the same polar, so comparing them we have

2 2
(x1/a )/l+(y1/b )/b2 = (-1)/n

or x1 = (-a2 l)/n y1 = (-b2 m)/n

2 2
.. The required pole is ((-a l)/n,(-b m)/n)

  cR c cRc


R  c


2 2
Find the points on the   x + 3y = 6 where the

are equally inclined to the axes. Prove also
that the length of the perpendicular from the centre on either of these

 is 2.

i  

2 2
The given   is x + 3y = 6

Or x2/6+y2/2 = 1 ...... (1)

If the coordinates of the required point on the   (1) be (6 cos ,2 sin ) then the

at the
point is x/6 cos + y/2 sin = 1 ...... (2)
Slope of (2) = (-cos )/62/(sin )=(-2)/6 cot

As the

are equally inclined to the axes so we have

o
-1/3 cot = + tan 45 = + 1

.. tan = + 1/3

The coordinates of the required points are

(63/2, 21/2) and (63/2, +-21/2)

-
= ((32)/2,1/2) and ((32)/2,+ 1/2)

Again the length of perpendicular from (0, 0) and (2),

2 2
= (6.2)/(2 cos + 6 sin )=(23)/((2,3/4)+(6.1/4) )=(23)/3 = 2

R  c 

If P be a point on the   x2/a2 + y2/b2 = 2/c whose ordinate is 2/c, prove that the angle between
the

at P and SP is tan-1 (b2/ac), where S is the focus.

i  

The given   x2/a2 + y2/b2 = 2/c ...... (1)

If (x',(2/c)) be the coordinates of the given point P on the   (1).

Then the

at P will be:(xx')/a2 +(yy')/b2 = 1

2 2
(xx')/(a (2/c) )+(yy' ((2/c)))/(b (2/c))= 1

The slope of

at P is (-b2 (2/c) x')/(a (2/c)(2(2/c)))= m1 (say)

If S be the focus, then slope of PS = = y'/(x'+ae)=(2/c)/(x'+a(2/c)e)= m2 (say)

If angle between the focal distance SP and



at P is , then

tan = (m2-m1)/(1+m2 m1).

(a2 (2/c)+b2 x'2+ aeb2 x' (2/c))/((a2 x'+a3 e(2/c)-b2 x')(2/c))

point (x',(2/c)) lies on   (1), we have


2 2 2 2 2
b x' + a (2/c)=(2a b )/c

and . a2 - b2 = a2 e2 so we have

2 2 2 2 2 2 2
(-(2a b )/c + aeb x' (2/c))/((a e x'+a e(2/c)) (2/c)) = ((2/c) ab ((2/c)

a+ex'))/(a2 e((2/c a+ex')) (2/c)) = b2/ae.

-1 2
.. = tan b /ae. Hence proved.

R  c

2 2 2 2
If P, Q, R are three points on the  x /a +y /b = 1 whose eccentric angles are , and then find
the area of PQR.

i  

2 2 2 2
The coordinates of the gives points P, Q, R on the   x /a +y /b = 1, will be (a cos , b sin ), (a cos
, b sin ) and (a cos , b sin ). Area of triangle PQR formed by these points

= 1/2 [x1y2 - x2y1 + x2y3 - x3y2 + x3y1 - x1y3]

= 1/2 [ab cos sin - ab sin cos + ab cos sin - ab sin cos + ab in cos - ab cos sin
]

= 1/2 ab [2 sin(-)/2 cos (-)/2 + 2 sin (-)/2 cos (-)/2 + 2 sin (-)/2 cos (-)/2]

= ab sin(-)/2 (cos(++2)/2 cos(-)/2)

= 2ab sin (-)/2 cos (-)/2 cos (-)/2.

R  c

Find the locus of the extremities of the latus recta of all  having a given major axis 6a.

i  

Let LSI be the latus rectum, C be the centre of the   and the coordinates of L be (x, y) then x = CS =
3 ae ...... (1)

2 2 2 2
And y = SL = b /3a =(9a (1-e ))/3a = 3a (1 - e ) ...... (2)

Eliminating the variable 'e' from (1) and (2) we get eh locus of L.

Hence putting the value of e from (1) and (2), we get


2 2
y = 3a(1-x /9a )

=> x2 = 9a2 - 3ay

2 2
=> x = 3a(3a - y), which is clearly a parabola. Similarly we can show that the locus of L' is x = 3ay(y + 3a)
which is again a parabola.

R  c

Find the equation of the


  at the end of the latus rectum of the  x2/a2 +y2/b2 = c2 and find
the condition when each normal through one end of the minor axis.

i  

The   x2/a2 + y2/b2 = c2

=> x2/(a2 c2+y2/(b2 c2) = 1 ...... (1)

2
Then the end point of the latus rectum is (ace,(b c)/a)

The normal at this point will be

(x-ace)/(ace/(a2 c2)) = (y-b2 c/a)/((b2 c/a)/(b2 c2))

=> ((x-ace))/e ac = (y-(b2 c)/a) ac

2
=> (x-ace)/e = y-(b c)/a

If this normal passes through (0, - bc), then, we have

(-ace)/e = -bc-b2/a c

=> a = b + a(1 - e2)

2
=> b - ae = 0

2 2 2 4
=> b/1-e => b /a = e

=> e4 + e2 = 1. This is required condition.

R  c
2 y2 2 2
The circle x + = 4 is concentric with the   x /7 + y /3 = 1; prove that the common

is
o
inclined to the major axis at an angle 30 and find its length.

i  

2 2
The   x /7+y /3=1 ...... (1)

The equation to the circle is

2 2
x +y =4 ...... (2)

As the line y = mx + (a2 m2+b2)

2
i.e. y = mx + (7m +3) ...... (3)

is always the

on the  . If this is also a

on the circle (2) then length of perpendicular
from the centre (0, 0) on the line (1) must be equal to radius of circle i.e. 2.

Hence, (7m2+3)/(1+m2) = 2

2 2 2
=> 7m + 3 = 2 (1 + m )

2 2
=> 7m - 4m = 4 - 3

=> m2 = 1/3

=> m = + 1/3

-1 o
Hence the common

to the two curves is inclined at an angle of tan (+1/3) i.e. 30 to the axis.

= 

We can also prove the above result by using the fact that the line = mx + (7m2+3) will be

to
2 2 2 2 2
x + y = 4 if discriminent of x + (m + (7m +3)) = 4 is zero.

Let P and Q be the points of contact of the common



with   and circle respectively and O be
the common centre of the two, then PQ = (OP2-OQ2)[ CPQ = 90o]

2 2 2 2 2 2 2 2
The coordinates of P are [(-a m)/(a m + b ), b /(a m + b )]

i.e. [(-7/3)/(16/3),3/(16/3)]

i.e.[(-7)/4,(33)/4]

and coordinate of O are (0, 0)

2 2
So, OP = (((-7)/4) + ((33)/4) ) = (19/4)
As OQ = r = 2

.. PQ = (OP2-OQ2 ) = (19/4-4) = 3/2

R  c

2 2 2 2
If q be the angle between CP and normal at point P, on the   a x + b y = 1, then find out tan
2 2
and prove that its greatest value is (b -a )/2ab. C is centre of   and P is any point on  .

i  

The equation of the   be

x2/(1/a2)+y2/(1/b2) = 1 ..... (1)

If be the angle between the normal at P = (1/a cos ,1/b sin and PC where C is the centre of
the   given by (1) equation to the normal PG is

2 2
x/a sec - y/b cosec = 1/a -1/b

2 2
=> bx sec - ay cosec = (b - a )/ab

Its slope = (b sec )/(a cosec ) = b/a tan = m1 (say)

The slope of PC = (1/b sin )/(1/a cos ) = a/b tan = m2 (say)

2 2
tan = (m1-m2)/(1+m1 m2) = (a/b tan -a/b tan )/(1+(b/a) tan (a/b) tan ) = ((b -a )tan )/ab(1-
2
tan )

(b2-a2)/2ab.(2tan)/(1-tan2 )

2 2 2
tan = (b - a )/2ab sin

The value of tan will be maximum when sin 2 is maximum, sin 2 is maximum i.e. sin 2 is 1.
Therefore the greatest value of tan is (b2-a2)/2ab.

R  c

Find the locus of the point of intersection of the two straight lines (x tan )/a - y/b + tan x = 0 and x/a+(y
tan )/b where a is fixed angle. Also find the eccentric angle of the point of intersection.

i  
Equation of the lines are given as

(x tan )/a-y/b + tan = 0 ...... (1)

x/a+(y tan )/b - 1 = 0 ...... (2)

To find the locus of the point of intersection, we have to eliminate the variable 'tan a' from (1) and (2), so
by (2),

y/b=1/(tan ) (1-x/a) and by (1)

y/b = tan (1+x/a)

Multiplying we get

y/b)2-(1-x/a)(1+x/a)

2 2 2 2
=> x /a +y /b = 1

This is the equation of an  

Again solving (1) and (2), we get

x = a(1-tan2 )/((1+tan2 ))

2 2
x = a(1-tan )/(sec )

Let the abscissa of the point of intersection be a cos f, then

2 2
x = a cos = a(1-tan )/(sec )

=> cos = (1-tan2 )/(sec2 )

2 2 2 2
=> (1-cos )/(1+cos )=(sec -(1-tan ))/(sec +(1-tan ) )

(By components & dividendo)

2 2
= (2 tan )/2 = tan

=> (2 sin2/2)/(2 cos2 ) = tan2

2 2
=> tan /2 = tan

=> tan /2 = tan

Hence = 2
R  c


If TP and TQ are perpendiculars upon the axes from any point T on the  x2/a2+ y2/b2 = 4. Prove that
PQ is always normal to fixed concentric  .

i  

The   is given by x2/(4a2 )+ y2/(4b2 ) = 1 ...... (1)

If the co-ordinates of T on the   be (2a cos f, 2b sin f) an TP and TQ perpendiculars on x-axis and y-
axis respectively, the co-ordinates of P and Q will be (2a cos f, 0) and (0, 2b sin f) respectively.

Now equation to PQ is

y - 0 = (2b sin -0)/(0-2a cos )(x - 2a cos )

=> x/(2a cos )+y/(2b sin ) = 1

=> x/a sec + y/b cosec = 2 ...... (2)

Now equation to the normal at point (A cos , B sin ) with respect to any other concentric  

2 2 2 2
x /A +y /B = 1 is

Ax sec - By cosec = A2 - B2 ...... (3)

As (2) and (3) are similar, on comparing then we have,

2 2
A/(12a)+B/(12b) = A - B ...... (4)

Solving the two equations given by (4), we get

B = (2a2b)/(a2-b2) and A = (-2 ab2)/(a2-b2 )

So the line (2) i.e. x/2a sec + y/2 cosec = 1 is a normal to the fixed  x2/A2 +y2/B2.

2 2 2 2 2 2
Where A = (-ab )/(a -b )and B =(a b)/(a -b ).

R  c

If the straight line y = 2x + 2 meet the ellipse 2x2 + 3y2 - 6, prove that equation to the circle, described on
the line joining the points of intersection as diameters, is 7x2+ 7y2 + 12x - 4y - 5 = 0.

i  
The line is given as

y = 2x + 2 ...... (1)

and the ellipse is given as

x2/a2 +y2/b2 = 1 ...... (2)

Solving (1) and (2), we gets

x2/3+(2x+2)2/2 = 1

2 2
=> x /3 + 2(x + 2x + 1) = 1

=> 7x2 + 12x + 6 = 3

2
=> 7x + 12x + 6 = 3

Let x1 and x2 be the two roots of this equation.

x1 + x2 = -12/7 ...... (3)

and x1x2 = 3/7 ...... (4)

Let y1 and y2 be the corresponding ordinates for the abscissa x1 and x2; so the co-ordinates of the points
of intersection will be (x1, y1) and (x2, y2). As these lie on line

y = 2(x + 1)

We have y1 = 2(x1 + 1) and y2 = 2(x2 + 1)

Where y1 + y2 = 2(x1 + x2) + 4 ...... (5)

y1y2 = 4(x1 + 1) (x2 + 1) ...... (6)

The equation to the circle drawn with the line joining (x1, y1) and
(x2, y2) as diameter is

(x - x1) (x - x2) + (y - y1) (y - y2) = 0

=> x2 + y2 - x(x1 + x2) - y(y1 + y2) + x1x2 + y1y2 = 0

2 2
=> x + y - x(x1 + x2) y[2(x1 + x2) + 4] + x1x2 +......+ 4[x1x2 + (x1 + x2) + 1] = 0

(By (5) and (6))


Putting the values from (3) and (4)

x2 + y2 - x (-12/7) - y[2(-12/7)+4] + 3/7 + 4[3/7+(-12/7)+1] = 0

2 2
= 7x + 7y + 12x - 4y - 5 = 0. Hence proved.

R  c


If the product of the perpendiculars from the foci upon the polar of P be constant and equal to c2. Find
the locus of P.

i  

Suppose the equation to the ellipse x2/a2 + y2/b2 = 1. The co-ordinate of foci are (ae, 0) and (-ae, 0).

Let the co-ordinates of P be (h, k). Then polar of P is

xh/a2 +yk/b2 = 1

2 2 2 2
Or b xh + a yk - a b = 0 ...... (1)

If P1 and P2 be the lengths of the perpendiculars on the line (1) from (ae, 0) respectively are

2 2 2 2 2 4 2
P1 = (b hae-a b )/(b h + a k )

And P2 = (-a2 b2-b2 hea)/(b2 h2+a4 x2)

.. P1P2 = (-b2 h2 a2 e2+a4 b4)/(b4 h2+a4) = c2 (By hypothesis)

=> a4b4 - b4h4a2e2 = c2b4h2 - c2a4k2

4 2 2 2 2 2 4 2 4 4
=> b h (c + a e ) + c a k = a b

Generalizing the locus of the point P(h, x) is

4 2 2 2 2 2 4 2 4 4
b x (c + a e ) + c a y = a b

R  c


2 2 2 2 2 2 2 2
Chords of ellipse x /a +y /b = 1 always touch another concentric ellipse x / + y / = 1, show that
the locus of their poles is (2 x2)/a2 + (2 y2)/b2 = 1.

i  

Let (x1, y1) be the pole of a chord of the ellipse


2 2 2 2
x /a +y /b = 1 ...... (1)

Then the equation of this cord is the same as the polar of (x1, y1) with respect to (1)

2 2
i.e. xx1/a + yy1/b = 1 ...... (2)

If (2) touches the ellipse x2/2 + y2/2 = 1, then

2 2 2 2 2 2 2
(b /y1) = {-{b x1/a y1}} +

=> b4/y21 = (2b4x21/a4y21) + 2

2 2 4 2 2 4
=> ( x1 )/a +( y1 )/b = 1

.. The locus of (x1, y1) is (2 x2)/a2 +(2 y2)/b4 = 1

Hence proved.

R  c


If the straight line y = x tan + ((a2 tan2 +b2)/2), being the angle of inclination, intersects the ellipse
x2/a2 +y2/b2 = 1. Then prove that the straight lines joining the centre to their point of intersection are
conjugate diameters.

i  

The equation of the ellipse be

2 2 2 2
x /a +y /b = 1 ...... (1)

and equation to the line is given as

y = x tan + ((a2 tan2 +b2)/2), being angle of inclination.

We can write this equation as,

2 2 2
y = mx + ((a m +b )/2)

=> ((y-mx)/2)/(a2 m2 + b2 ) = 1 ...... (2)

To get the equation to the lines joining the point of intersection to the origin, making (1) homogeneous
with the help of (2), we have

2 2 2 2 2 2 2 2
x /a + y /b = [((y-mx)/2)/(a m + b )]
2 2 2 2 2 2
= 2((y + m x - 2mxy))/(a m + b )

=> (b2 x2 + a2 y2) (a2m2 + b2) = 2a2 b2(y2 + m2x2 - 2mxy)

2 2 2 2 2 2 2 2 2 2 2 2
y a (a m - b ) + 4m - b xy - b x (a m - b ) = 0

=> y2 + (4 mb2)/((a2 m2+b2)) xy -b2/a2 x2 = 0 ...... (3)

2
This equation represents two straight lines y = m1x and y = m2x then the combined equation will be y -
2
(m1 + m2)xy + m1m2x = 0.

Comparing (3) and (4); we get

2 2
m1m2 = -b /a

which is the condition of diameter to be conjugate. Hence the lines are the conjugate diameters.

R  c


The eccentric angles of two points P and Q on the ellipse 1, 2. Find the area of the parallelogram
formed by the tangents at the ends of diameters through P and Q.

i  

The ellipse is

x2/a2 +y2/b2 = 1

Equation to the tangent at the points P and Q are

x/a cos 1 + y/b sin 1 = 1 ...... (1)

and

x/a cos 2 + y/b sin 2 = 1 ...... (2)

Solving (1) and (2), we will have the coordinates of the point of intersection. Multiplying (1) by sin 2 and
(2) by sin 1 and subtracting, we get

x/a (sin 2 cos 1 - cos 2 sin 1) = sin 2 - sin 1

=> x/a sin (2 - 1) = 2 sin (2 - 1)/2cos (2 + 1)/2

.. x = a (cos((1 - 2)/2))/(cos(1 - 2)/2); y = b (sin(1 + 2)/2)/(cos (1 - 2)/2)

Above are co-ordinates of the point of intersection L of tangents at p and Q, i.e. at 1 and 2. Putting
1 = p + 1 in above we get the co-ordinates of the point of intersection M of tangent at Q and P' as
Area of the parallelogram LMNO = 4CLM

= 4 . 1/2 (x1 y1 - x2 y2)

= 2 ab/(sin (1 - 2)/2 cos (1 - 2)/2) [ - cos2 (1 + 2)/2-sin (1 + 2)/2]

= (-4ab)/(sin(1 - 2))

= - 4ab cosec (1 - 2) Area can't be (-) ve

So the area = 4ab cosec (1 - 2)

Hyperbola
s    

 
         
  
 
     
       

 
  
        
      

Hyperbola              
    
     
  
     
      

  
    
                        
 

    
 
 
s     

    
   
  

     
  
  

!    properties of hyperbola


   
           s    
   
     "     
 
 

 # 


 rectangular hyperbola  
   

     
 
 
     
        
   
 
  
 




s   $%  & '


 

 Hyperbola    

Topics Covered: 

Basic Concepts of Hyperbola



Relation between Focal Distances

Parametric Coordinates

Important Properties of Hyperbola

Examples Based on Hyperbola

Ellipse Vs Hyperbola

Propositions of a Hyperbola

Examples on Propositions of a Hyperbola

Rectangular hyperbola

Intersection of a Circle and a Rectangular Hyperbola

Conjugate hyperbola

Examples on finding locus of point

Solved Examples on Hyperbola Part-I

Solved Examples of Hyperbola (Part-II)

Solved Examples of Hyperbola Part-III



-   
 
As we have studied earlier that by slicing a cone with a plane in different orientations, we obtain conic
sections. The hyperbola is obtained by cutting a right circular cone at both the nappes by a plane. The
hyperbola is to be thought of a single curve consisting of two branches, one on each nappe.





A hyperbola is the locus of a point which moves in a plane such that its distance from a fixed point (called
the focus) is e ( >1) times its distance from a fixed straight line (called the directrix).

The hyperbola is a conic section for which the eccentricity (e) is greater than unity.


  

Let S be the focus and ZM the directrix of a hyperbola.


Since e > 1, we can divide SZ internally and externally in the ratio e : 1; let the points of division be A and
A as in the figure. Let AA = 2a and be bisected at C. Then, SA = e.AZ, SA = e.ZA

SA + SA = e(AZ + ZA) = 2ae

i.e., 2SC = 2ae or SC = ae.

Similarly by subtraction, SA SA

= e(ZA ZA) = 2e.ZC 2a = 2eSC SC = a/e.

Now, take C as the origin, CS as the x-axis, and the perpendicular line CY as the y-axis. Then, S is the point
(ae, 0) and ZM the line x = a/e. Let P(x, y) be any point on the hyperbola. Then the condition
2 2 2 2 2 2 2 2 2 2 2 2
PS = e .(distance of P from ZM) gives (x ae) + y = e (x a/e) or x (1 e) + y = a (1 e )

2 2 2 2 2
i.e. x /a y /a (e 1) = 1. (i)

Since e > 1, e2 1 is positive. Let a2 (e2 1) = b2. Then the equation (i) becomes x2/a2 y2/a2 = 1.

The eccentricity e of the hyperbola x2/a2 y2/a2 = 1 is given by the relation e2 = (1 + b2/a2).

Since the curve is symmetrical about the y-axis, it is clear that there exists another focus S at (ae, 0) and
a corresponding directrix ZM with the equation x = a/e, such that the same hyperbola is described if a
point moves so that its distance from S is e times its distance from ZM.

The points A and A where the straight line joining the two foci cuts the hyperbola are called the vertices
of the hyperbola.

The straight line joining the vertices is called the transverse axis of the hyperbola, its length AA is 2a.

The middle point C of AA possesses the property that it bisects every chord of the hyperbola passing
through it. It can be proved by taking P(x1, y1) as any point on the hyperbola. If (x1, y1) lies on the
hyperbola then so does P(x1, y1) because the hyperbola is symmetrical about the x and the y axes.
Therefore PP is a chord whose middle point (0, 0), i.e. the origin C. On account of this property the middle
point of the straight line joining the vertices of the hyperbola is called the centre of the hyperbola.
The straight line through the centre of a hyperbola which is perpendicular to the transverse axis does
not meet the hyperbola in real points. If B and B be the points on this line such that BC=CB=b, the line
BB is called the conjugate axis.

A latus rectum is the chord through a focus at right angle to the transverse axis.

The length of the semi-latus rectum can be obtained by putting x = ae in the equation of the hyperbola.
2 2 2 2 2
Thus y = b a e /a 1 = be 1 = b.b/a = b /a.

X 
   
Since the curve is symmetrical about x-axis, therefore there exists another focus at point (ae, 0) of the
hyperbola. (Similar to ellipse).

Corresponding to these foci, there are two directrices whose equations are x = a/e and x = a/e.

& 
 !
X  

cc
The difference of the focal distances of a point on the hyperbola is constant. PM and PM are
perpendiculars to the directrices MZ and MZ and PS PS = e(PM PM) = eMM = e(2a/e) = 2a =
constant.



 

 

A hyperbola can be defined in another way; Locus of a moving point such that the difference of its
distances from two fixed points is constant, would be a hyperbola.

@

 
A  
The points A(a, 0) & A(a, 0) are called the vertices of the hyperbola and the line AA joining the vertices is
called the transverse axis and the line perpendicular to it, through the centre (0, 0) of the hyperbola is
called conjugate axis.




Any chord of the hyperbola through C will be bisected at C (by symmetry), therefore C is called the centre
of hyperbola.

Thus hyperbola is a central curve.

%  

The chord of a hyperbola through one of the foci and at right angle to the transverse axis is called the
latus rectum:

If 2l be the length of the latus rectum, then the co-ordinates of one of its extremities is (ae, l).

2 2 2 2
The point (ae, l) lies on the hyperbola x /a y /b , so we have

2 2 2
e (l /b ) = 1

l2 = b2(e21) = b2a2(e21)/a2 = b4/a2 (? b2 = a2(e2 1)

2 2
l = b /a

The length of the latus rectum = 2 b2/a2

&  
 
!     

2 2 2 2
The quantity x1 /a y1 /b = 1 is positive, zero or negative, according as the point (x1, y1) lies within, upon
or without the curve.

    

2 2 2 2
We can express the coordinate of a point of the hyperbola x /a y /b = 1 in terms of a single parameter,
say .

In the adjacent figure OM = a sec and PM = b tan . Thus any point on the curve, in parametric form is x
= a secq, y = b tan.

In other words, (a sec , b tan ) is a point on the hyperbola for all values of . The point (a sec, b tan)
is briefly called the point .


  
cc
Since the fundamental equation of the hyperbola only differs from that of the ellipse in having b2 instead
of b2, it will be found that many propositions for the hyperbola are derived from those for the ellipse by
changing sign of b2. Some results for the hyperbola x2/a2y2/b2 = 1 are

The tangent at any point (x1, y1) on the curve is xx1/a2 yy1/b2 = 1.

The tangent at point is x sec/a y tan/b = 1.

The straight line y = mx + c is a tangent to the curve, if c2 = a2 m2 b2. In other words, y = mx + a2m2
2
b touches the curve for all those values of m when m > b/a or m < b/a.

2 2
Equation of the normal at any point (x1, y1) to the curve is xx1/x1/a = yy1/y1(b) .

The equation of the chord through the points 1 and 2 is

The equation of the normal at is ax cos + by cot = a2 + b2.


Through a given point, four normals (real or imaginary) can be drawn to a hyperbola.

The tangent drawn at any point bisects the angle between the lines, joining the point to the foci,
whereas the normal bisects the supplementary angle between the lines.

Equation of the director circle is x2 + y2 = a2 b2. That means if a2 > b2, there would exist several points
2 2
such that tangents drawn from them would be mutually perpendicular. If a < b , no such point exist. For
2 2
a = b , centre is the only point from which two perpendicular tangents (asymptotes) to the hyperbola can
be drawn.

From any point (x1, y1) in general two tangents can be drawn to hyperbola. The equation of the pair of
tangents is

1 2 2 2 2 2 2 2 2 2 2 2
(x /a y /b 1) (x1 /a y1 /b 1) (xx1/a yy1/b 1) or SS1 = T

The equation of the chord of contact is xx1/a2 yy1/b2 1 = 0 or T = 0.

The equation of the chord bisected at the point (x1, y1) is

2 2 2 2 2 2
xx1/a yy1/b 1 = x1 /a y1 /b 1 or T = S1.

Equation of the chord the points (a sec , b tan ) and (a sec ?, b tan ?) is

x/a cos ?/2 y/b sin +?/2 = cos +?/2.

 - 
 
  


Find the equation of the hyperbola whose directrix is 2x + y = 1, focus is (1, 1) and
eccentricity is 3.

 


Let S(1, 1) be focus and P(x, y) be any point on the hyperbola. From P draw PM
perpendicular to the directrix then PM =2x+y1/22+12 = 2x+y1/5

Also from the definition of the hyperbola, we have

SP/PM = e SP = ePM

(x1)2+(y1)2 = 3 (2x+y1/5)
(x 1)2 + (y 1)2 = 3 (2x+y1)2/5

5[(x2+12x)+(y2 + 1 2y)]=3(4x2 + y2 + 1 + 4xy 4x 2y)

7x2 2y2 + 12xy 2x 4y 7 = 0

  


Find the directrix, foci and eccentricity of the hyperbola

ax2 y2 = 1

 


The given hyperbola is

ax2 y2 = 1

or x2/(1/a) y2/1 = 1 (1)

which is of the form(x2/a2) (y2/b2) = 1

Here a2 = 1/a, b2 = 1

If e be the eccentricity of the hyperbola, then


b2 = a2(e2 1)

1 = (e2 1)

a = (e2 1)

or e2 = a + 1 or e =a+1

Also the foci are given by (+ ae, 0)

The required foci are

(+ 1/a (a+1), 0)

or (+ a+1/a, 0)

And the directricies are given by x = + (a/e)

x = + [1/a/a+1] (? a = 1/a, e = 1/a+1)

x = + 1/a(a+1)

  


Find the locus of a point, the difference of whose distances from two fixed points is
constant.

 

Let two fixed points be S (ae, 0) and S (ae, 0). Let P(x, y) be a moving point such that

SP SP = Constant = 2a (say).

Then [(xae)2 + (y0)2] [x+ae]2+(y0)2 = 2a

[(xae)2+y2] = + 2a + [(x+ae)2+y2

(x ae)2 + y2 = 4a2 + (x + a2)2 + y2 + 4a [(xae)2+y2]

(x ae)2 (x + ae)2 4a2 = + 4a [(xae)2+y2]

4aex 4a2 = + 4a [(xae)2+y2]

(ex + a)2 = (x + ae)2 + y2

(e2 1)x2 y2 = a2(e2 1)

x2/a2y2/b2 = 1 taking b2 = a2(e2 1)

This is a hyperbola.

  


If A, B, C are three points on the hyperbola xy = c2 and AC is perpendicular to BC, prove


that AB is parallel to the normal to the curve at C.

 

Let the three points A, B, C respectively be (ct1, c/t1), (ct2, c/t2) and (ct3, c/t3). Since AC is
perpendicular to BC,

(c/t3c/t1/ct3ct1) = 1 t1t2 = 1 (1)

Normal to the curve at C (ct3, c/t3) is

y = xt32 + 2/t3 (1t34) and its slope is t32 = 1/t1t2 (2)

Slope of AB = (c/t2c/t1/ct2ct1) = 1/t1t2 = t32

AB is parallel to the normal at C.

  


Find the equation of the hyperbola the distance between whose foci is 16, whose
eccentricity is 2 and whose axis is along the x-axis centre being the origin.

 


We have b2 = a2(e2 1) = a2 b = a.

Also 2ae = 16 ae = 8 a = 42.

Hence the equation of the required hyperbola is


x2/32 y2/32 = 1 x2 y2 = 32.

  


The perpendiculars drawn from the centre of a hyperbola x2/a2 y2/b2 = 1 upon the
tangent and normal at any point of the hyperbola meet them in Q and R. Find the locus
of Q and R.

 


Tangent at any point P(a sec , b tan ) is sec y/b tan = 1. (1)

Equation of the line through centre (origin) perpendicular to (1) is y = a sin /b x

sin = by/ax

Eliminating from (1), we get x/a cos y/b cos (by/ax) = 1.

x2 + y2 = ax cos (x2 + y2)2 = a2x2(1 b2y2/a2x2)

Or (x2 + y2)2= a2x2 b2y2, which is the locus of Q.

Normal at the point P (a sec , b tan ) is ax cos + by


cot = a2 + b2 (2)

Equation of the line perpendicular to (2) drawn from the centre is

y = bx/a sin (3)

Form (2) and (3),

sin = bx/ay and ax 1b2y2/a2x2 + by 1b2x2/a2y2 . ay/bx = a2 + b2

(x2 + y2)2 (a2y2 b2x2) = (a2 + b2)2 x2y2, which is the locus of R.

 B 



  !!    !C
!  
 "

Most of the results obtained in the case of the ellipse x2/a2 y2/b2 = 1 hold good for the
hyperbola x2/a2 y2/b2 = 1, when only the sign of b2 is changed. The proofs of these
results can be derived exactly in the same manner as they were derived for ellipse. So let
us see some final results.

1. Tangent at (x1, y1) to the hyperbola x2/a2 y2/b2 = 1 is xx1/a2 yy1/b2= 1 i.e. T = 0.

2. Equation of tangent in terms of m is y = mx + (a2m2b2).

3. Equation of the normal at (x1, y1) to the hyperbola is xx1/(x1/a2)yy1/(y1/b2).

4. Equation of pair of tangents drawn from point (x1, y1) to the hyperbolax2/a2 y2/b2 =
1 is given by SS1 = T2

Where S = x2/a2 y2/b2 = 1


S1 = x12/a2 y12/b2 = 1

T = xx1/a2 yy1/b2 = 1

5. The Chord of Contact of tangents from (x1, y1) to the hyperbola x2/a2 y2/b2 = 1 is
given by T = 0 i.e. xx1/a2 yy1/b2 = 1.

6. The Polar of Pole (x1, y1) to the hyperbola x2/a2 y2/b2 = 1 is given by T = 0
i.e. xx1/a2 yy1/b2 = 1.

7. The equation of Chord of hyperbola x2/a2 y2/b2 = 1 whose middle point is (x1, y1) is
given by T = S1 i.e. x12/a2 y12/b2 = xx1/a2 yy1/b2 = 1.



Try to get above results yourself using traditional methods similar to the ellipse.


 
!
    

   
  

   

      C 
 

  "

Yes. Before that let us understand the concept of the auxiliary circle of a hyperbola. The
circle described on the transverse axis of hyperbola as its diameter is called its auxiliary
circle.

We know that the line AA joining the vertices A(a, 0) and A(a, 0) of the
hyperbola x2/a2 y2/b2 = 1 is called the transverse axis.

The equation of the auxiliary circle, described on AA as diameter, is

(x a) [x (a)] + (y 0)] + (y 0)(y 0) = 0


or x2 + y2 = a2

Now let us draw the foot N of any ordinate NP of the hyperbola draw a tangent NU to
this circle, and join CU. The

CU = CN cos NCU

i.e. x = CN = a sec NCU

The angle NCU is therefore the angle ?.

Also NU = CU tan ? = a tan ?

So that NP : NU = b : a

So the ordinate of the hyperbola is therefore in a constant ratio to the length of the
tangent drawn from its foot to the auxiliary circle.

When it is desirable to express the co-ordiantes of any point of the curve in terms of
one parameter than we use

x = a sec ?, and y = b tan ?

= 

This angle ? is not so important an angle for the hyperbolas the eccentric angle is for the
ellipse.

 
   
   

The locus of the middle point of a system of parallel chords of a hyperbola is called its
diameter.

The equation of the diameter is y = b2x/(a2m), where m is the slope of the system of
parallel chords.
= 

c

A   

Two diameters of a hyperbola which bisect chords parallel to each other are called
conjugate diameters.

The diameters y = mx and y = m1x of the hyperbola x2/a2 y2/b2 = 1 are conjugate if
mm1 = b2/a2.

     

The director circle is the locus of the point of intersection of a pair of perpendicular
tangents to a hyperbola.

Equation of the director circle of the hyperbola x2/a2 y2/b2 = 1 is x2 + y2 = a2 b2 i.e. a


circle whose centre is origin and radius is (a2b2).

= 
If b2 < a2, this circle is real.

If b2 = a2, the radius of the circle is zero, and it reduces to a point circle at the origin. In
this case the centre is only point from where tangents at right angle can be drawn to the
hyperbola.

If b2 > a2, the radius of the circle is imaginary, so that there is no such circle, and so no
tangents at right angles can be drawn to the circles.

  

A line, which is tangent to the hyperbola at infinity, but which is not itself at infinity, is
called the asymptote of the curve.

To find the equation of the asymptotes of the hyperbola x2/a2 y2/b2 = 1.


Let y = mx + c be an asymptotic to the given hyperbola. Then eliminating y, the
abscissas of the points of intersection of y = mx + c

or x2(b2 a2m2) 2a2mcx a2(b2 + c2) = 0 (1)

If the line y = mx + c is an asymptote of the hyperbola then it touches the hyperbola at


infinity i.e. both the roots of the equation (1) are infinite and for this we must have b2
a2m2 = 0 and 2a2mc = 0. Hence we get m = + (b/a) and c = 0.

The asymptotes are y = + (b/a) x

Or x2/a2 + y2/b2 = 0

Their combined equation is (x/a+y/b)(x/ay/b) = 0 or xx2/a2 y2/b2 = 0which shows


that the equation of the asymptote differs from that of the hyperbola in the constant
term only. Also the angle between the asymptotes is 2 tan1 (b/a).

The lines x2/a2+y2/b2=0 are also asymptotes to the conjugate hyperbolax2/a2y2/b2=1.

& C

The equation of the hyperbola and that of its pair of asymptotes differ by a constant.
For example, if S = 0 is the equation of the hyperbola, then the combined equation of
the asymptotes is given by S + K = 0. The constant K is obtained from the condition that
the equation S + K = 0 represents a pair of lines. Finally the equation of the
corresponding conjugate hyperbola is S + 2K = 0.

Any line drawn parallel to the asymptote of the hyperbola would meet the curve only
at one point.
  


Find the hyperbola whose asymptotes are 2x y = 3 and 3x + y 7 = 0 and which


passes through the point (1, 1).

 


The equation of the hyperbola differs from the equation of the asymptotes by a
constant.

The equation of the hyperbola with asymptotes 3x + y 7 = 0 and 2x y = 3 is (3x +


y 7) (2x y 3) + k = 0. It passes through
(1, 1) k = 6.

Hence the equation of the hyperbola is (2x y 3)(3x + y 7) = 6.

  


Find the angle between the asymptotes of the hyperbola x2/a2y2/b2 = 1, then length of
whose latus rectum is 4/3 and which passes through the point (4, 2).

 


We have 2b2/a = length of the latus rectum = 4/3 3b2 = 2a

Also, the hyperbola passes through the point (4, 2).

Hence 16/a2 4/b2 = 1 16/a2 6/a = 1

Or a2 + 6a 16 = 0 (a 2)(a + 8) = 0 a = 2 b2 = 4/3.
The asymptotes of the given hyperbola are y = + b/a x or y + 1/3 x.

If 1 and 2 are the angles which the asymptotes make with the positive x-axis, then

tan 1 = 1 = /6 and tan 2 = 1/3 2 = /6.

Hence the angle between the asymptotes = /3.

  


Prove that the chords of the hyperbola x2/a2y2/b2 = 1, which touch its conjugate
hyperbola are bisected at the point of contact.

 


Let P(x1, y1) be the mid-point of the chord of the given hyperbola, so that the equation
of the chords is xx1/a2yy1/b2 = x12/a2y12/b2. (1)

If touches the conjugate hyperbola x2/a2y2/b2 = 1, then

x2/a21/b2 [xx1/a2x12/a2+y12/b2]2.b4/y12 + 1 = 0 will have equal roots. Simplifying, we


find that

x2/a2 [y12/b2x12/a2]+2xx1/a2[x12/a2y12/b2][y12/b2x12/a2]2+y12/b2= 0 has equal roots so


that

4x12/a2 [x12/b2y12/a2]+4[y12/b2x12/a2][[y12/b2x12/a2]2y12/b2]= 0

or, x12/a2 [x12/a2y12/b2][x12/a2y12/b2]2 + y12/b2 = 0 or (x12/a2y12/b2)(x12/a2


x12/a2+y12/b2)+y12/b2= 0

or x12/a2 y12/b2 + 1= 0 (x1, y1) lies on the conjugate hyperbola.


Hence the chord (1) touches the conjugate hyperbola at its midpoint (x1, y1).

 
 


Any tangent to the conjugate hyperbola x2/a2y2/b2 = 1 is

x = my + b2m2a2. (2)

If this is same as the chord (1), then m = a2y1/b2x1 and hence

a4/x12 [x12/a2 y12/b2]2 = b2m2 a2 = b2a4y12/b4x12 a2

Or [x12/a2 y12/b2]2 = y12/b2 x12/a2 or x12/a2 y12/b2 = 1

(x1, y1) lies on the conjugate hyperbola.

the chord (1) touches conjugate hyperbola and is bisected at the point of contact.

&
 
cc
If the asymptotes of a hyperbola are at right angles to each other, it is called a
rectangular hyperbola.
A hyperbola whose asymptotes are at right angles to each other is called a rectangular
hyperbola. The angle between asymptotes of the hyperbola x2/a2 y2/b2 = 1, is 2 tan
1
(b/a).

This is a right angle if tan1 b/a = /4, i.e., if b/a = 1 b = a.

The equation of the rectangular hyperbola referred to its transverse and conjugate
axes as axes of coordinates is therefore:

x2 y2 = a2. (1)
Equation referred to asymptotes as coordinate axes: To obtain this equation we rotate
the axes of reference through 45o. Thus we have to write

x/2 + y/2 for x and x/2 + y2 for y.

The equation (i) becomes

(1/2) (x + y)2 (1/2)(x y)2 = a2 i.e. xy = a2 or xy = c2 where c2 = a2/2.

 


We know that the asymptote of hyperbola

x2/a2 y2/b2 = 1 (1)

are given by y = + (b/a) x (2)

If be the angle between the asymptotes, then

= tan1 (m1m2/1+m1m2) = tan1 ((b/a)(b/a)/1+(b/ab/a))

tan1 (2(b/a)/1(b2/a2)) = 2 tan1 (b/a)

But if the hyperbola is rectangular, then =/2

i.e., /2 = 2 tan1 (b/a)

or tan (/2) = b/a b = a

From (1) the equation of the rectangular hyperbola is x2 y2 = a2

 
   

(i) In a hyperbola b2 = a2 (e2 1). In the case of rectangular hyperbola (i.e., when b =
a) result become a2 = a2(e2 1) or e2 = 2 or e =2

i.e. the eccentricity of a rectangular hyperbola =2


(ii) In case of rectangular hyperbola a = b i.e., the length of transverse axis = length of
conjugate axis.

Hence it is also called an equilateral hyperbola.


 
    
  &
 

A rectangular hyperbola and a circle meet in four points. The mean of these four points
is the middle point of the centres of the hyperbola and that of the circle.

Let the rectangular hyperbola be xy = c2 and the equation of the circle be x2 + y2 + 2gcp
+ 2fy + k = 0. Any point on the hyperbola is (cp, c/p). If it lies on the circle, then c2p2 +
c2/p2 + 2gcp + 2fc/p + k = 0.

c2p4 + 2gcp3 + kp2 + 2fcp + c2 = 0.

This is fourth degree equation in p, which has four roots. Hence the circle and the
hyperbola intersect in four points. If p1, p2, p3, p4 are the roots of this equation, then

p1 + p2 + p3 + p4 = 2gc/c2 = 2g/c cp1 + cp2 + cp3 + cp4 = 2g

x1+x2+x3+x4/4 = g/2

Also 1/p1 + 1/p2 + 1/p3 + 1/p4 = p1p2p3/p1p2p3p4 = 2fc/c2/c2/c2 = 2f/c

c/p1 + c/p2 + c/p3 + c/p4 = 2f y1+y2+y3+y4/4 = f/2.

Hence the mean of the four points is (g/2, f/2) which is the mid-point of the centre of
the hyperbola and that of the circle.

  


A circle and a rectangular hyperbola meet in four points A, B, C and D. If the line AB
passes through the centre of the circle, prove that the centre of the hyperbola lies at the
mid-point of CD.
 


The line AB passes through the centre of the circle. Hence AB is the diameter of the
circle and the mid-point of AB is the centre of the circle. Let the co-ordinates of A, B, C,
D be respectively (x1, y1) (x2, y2), (x3, y3) and (x4, y4). Let the centres of the hyperbola and
the circle be (h, k) and (g, f).

Then x1+x2x3+x4/4 = h+g/2. But g = x1+x2/2

2g+x3/x4/4 h+g/2 x3+x4/2 = h

Similarly y3+y4/2 = k.

Hence (h, k) is the mid-point of CD.


      
     
  /
!

  
  !      
 "

Let transverse and conjugate axes as axes of co-ordinates (XOX and YOY in the above
figure), the equation of rectangular hyperbola is

X2 Y2 = a2 (1)

Also we know that the asymptotes of a rectangular hyperbola are at right angles to each
other.

Let Ox and Oy be the asymptotes, each making as angle of /4 with the co-ordinate
axes. Rotate the axes through as angle of /4 with the co-ordinate axes. To find the
equation of the rectangular hyperbola referred to asymptotes as axes. We have to
substitute for X and Y in (1) the xcos(/4) ysin(/4) and ycos(/4) + xsin(/4)
respectively.

i.e. x+y/2 and yx/2 respectively.

So from (1) equation of the rectangular hyperbola referred to asymptotes as axes is

(x+y/2)2 = (yx/2)2 = a2 xy = a2/2

xy = c2 where 2c2 = a2

The shape of the above rectangular hyperbola referred to asymptotes as co-ordinate


axes is as shown is the adjacent figure.

= 

1. Parametric co-ordinates of any point on the rectangular hyperbola xy = c2 is (ct, c/t)


where t is the parameter.

2. Equations of tangent and normal at any point (ct, c/t) on the rectangular hyperbola xy
= c2 are x + yt2 = 2ct and xt3 yt ct4 + c = 0 respectively.


A 
A hyperbola whose transverse and conjugate axes respectively are the conjugate and
transverse axes of a given hyperbola is called the conjugate hyperbola of the given
hyperbola. The asymptotes of these two hyperbolas are also the same.
Equation of a conjugate hyperbola is x2/a2 y2/b2 = 1 (1)

of the given hyperbola x2/a2 y2/b2 = 1

Its transverse and conjugate axes are along y and x axes respectively.

= 

1.Any point on conjugate hyperbola (2) is (a tan q, b sec q)

2.The equation of the conjugate hyperbola to xy = c2 is xy = c2.

3.By comparing the equations of hyperbola x2/a2 y2/b2 = 1.

asymptotes x2/a2 y2/b2 = 0

and conjugate hyperbola x2/a2 y2/b2 = 1 we find that:

(a) The equation of the hyperbola and asymptotes differ by the same constant by which
the equations of the asymptotes and the conjugate hyperbola differ.

(b) Hyperbola + Conjugate hyperbola = 2 (Asymptotes). (check yourself).

4.The tangents drawn at the points, where a pair of conjugate diameters meets a
hyperbola and its conjugates form a parallelogram, whose vertices lie on the asymptotes
and whose area is constant. (prove yourself).

5.If a pair of conjugate diameters of hyperbola meet the hyperbola and its conjugate in
P, P and D, D respectively, then the asymptotes bisect PD and PD. (Prove yourself).

  

If e1 and e2 are the eccentricities of the hyperbola x2/a2 y2/b2 = 1 and its conjugate
hyperbola, prove that e12 + e22 = 1.

 


The eccentricity e1 of the given hyperbola is obtained from

b2 = a2(e12 1). (1)

The eccentricity of e2 of the conjugate hyperbola is given by

a2 = b2(e22 1). (2)

Multiply (1) and (2), we get,

1 = (e12 1)(e22 1) 0 = e12 e22 e12 e22

e12 + e22 = 1.

 

C is a centre of the hyperbola x2/a2y2/b2 = 1 and the tangent at any point P meets asymptotes in the
point Q and R. Find the equation to locus of the centre of the circle circumscribing the triangle CQR.

 


This equation to the hyperbola is given as


x2/a2y2/b2 = 1 (1)

Let P any point on it as (a sec ?, b tan ?), then the equation of tangent at P is

x/a y/b sin ? = cos ? (2)

The equation to the asymptotes to (1) are

x/a = y/b (3)

and x/a = y/b (4)

Solving (2) and (3), we get the coordinates of Q as


(acos?/1sin?, bcos?/1sin?)

Solving (2) and (4), we get the coordinates of R as

(acos?/1+sin?, bcos?/1+sin?)

Let O be the centre of the circle passing through C, Q and R having its coordinates as (h, k). Then
clearly OC = OQ

h2 + k2 = (hacos?/1sin?)2 + (kbcos?/1sin?)2

h2 + k2 = (hacos?/1sin?)2 + (kbcos?/1sin?)2

h2 + k2 = h2 + k2 + (a2 + b2) cos2?/(1sin?)2 (2ah + 2bk) cos?/1sin?

2(ah + bk) = (a2 + b2) cos?/1sin? (5)


Similarly OC = OR

Hence h2 + k2 = (hacos?/1+sin?)2 + (k+bcos?/1+sin?)2

Which on simplification as in the last case, given

2(ah bk) = (a2 + b2) cos?/1sin? (6)

to get the locus of the point O we have to eliminate f from (5) and (6), so multiplying the two we get

4(a2h2 b2k2) = (a2 + b2) cos2?/1sin2? = (a2 + b2)2

for (h, k), we get the required locus as

4(a2x2 b2y2) = (a2 + b2)2


 

A straight line is drawn parallel to the conjugate axis of a hyperbola meets it and the conjugate
hyperbola in the points P and Q. Find the locus of point of intersection of tangents at P and Q.

 


Let the equation to the hyperbola be

x2/a2y2/b2 = 1 (1)

and its conjugate hyperbola be

y2/b2x2/a2 = 1 (2)

Let p be any point (a sec ?, b tan ?) on P. The equation of the line parallel to the conjugate axis (1) i.e.
y-axis passes through P will be
x = a sec ? (3)
The line (3) will cut the conjugate hyperbola (2) at Q where x = a sec ? and hence y = b (1+sec2?),
therefore the coordinates of Q will be {asec?, b(1+sec2?)}

Now the equation to the tangent to (1) at P is

x/ay/b sin ? = cos ?

or x/a cos ? = y/b sin ? (4)

and the equation to the tangent to (2) at Q is

y/b (1+sec2?) sec ? = 1

x/a + cos ? = y/b (1+cos2?) (5)

on squaring and adding (4) and (5), we have


2x2/a2 + 2 cos2 ? = y2/b2 [(1 + cos2 ?) + sin2 ?] = 2 y2/b2

cos2 ? = y2/b2 x2/a2

Putting the value of cos ? in (5) we get

x/a + (y2/b2x2/a2) = y/b (1+y2/b2x2/a2)

Squaring we have,

x2/a2 + y2/b2 x2/a2 + 2 x/a (y2/b2 x2/a2) = y2/b2 (1+y2/b2x2/a2)

2x/a (y2/b2x2/a2) = y2/b2(y2/b2x2/a2)


y2/b2 (y2/b2x2/a2) = 2x/a

y4/b4 (y2/b2x2/a2) = 4x2/a2

There is the required locus.

 

From a point A, perpendiculars AB and AC are drawn to two straight lines OB and OC. If the area
OBAC is constant, find the locus of A.

 


Let the bisectors of the angles BOC be taken as axis. So the equations of OB and OC are respectively.

x cos + y sin = 0

and x cos y sin = 0 where = 1/2 pBOC


Take any point A as (h, k); then

AB = Perpendicular from A on OB

= hcos+ksin/cos2+sin2 = h cos + k sin (1)

and similarly

AC = Perpendicular from A on C

= h cos k sin (2)

The equation to AB will be

(h x) sin + (y k) cos = 0
y cos x sin + h sin k cos = 0 (3)

Similarly the equation AC will be

(h x) sin (y k) cos = 0

Now OB = Perpendicular distance of (3) from (0, 0).

= 00+hcos+ksin/cos2+sin2

= h sin k cos
Similarly OC = perpendicular distance of (0, 0) from (4)

= h sin + k cos .

Now the area of quad. OBAC = OAB + OAC

= 1/2 OB AB + 1/2 OC AC

= 1/2 [h sin kcos][h cos + k sin] + 1/2 [h cos k sin] [h sin + k cos]

= (h2 k2) sin.cos = constant = S (say)

h2 k2 = {s/sina, cosa} which is again constant = a2 (say)

Therefore the locus of the point (h, k) will be x2 y2 = a2, which is hyperbola.


 
 

 

Show that the line 4x 3y = 9 touches the hyperbola 4x2 9y2 = 27.

 


We know that if the line y = mx + c touches the hyperbola x2/a2y2/b2 = 1, then c2 =


a2m2 b2

Here the hyperbola is

x2/(27/4)y2/(27/9) = 1

i.e. here a2 = 27/4 b2 = 27/9 = 3

And comparing 4x 3y = 9 with y = mx + c, we get

m = 4/3, c = 3

a2m2 b2 = (27/4)(4/3)3 3 = 12 3 = 9 = (3)2

or a2m2 b2 = c2

Hence the given line touches the given hyperbola.

 

Prove that the mid points of chords of the hyperbola x2/a2y2/b2 = 1 parallel to the
diameter y = mx lie on the diameter a2my = b2x

 


The hyperbola is x2/a2y2/b2 = 1 (1)


The equation of any chord parallel to the diameter

y = mx is y = mx + c (2)

Eliminating y between (1) and (2), we get

x2/a2(mx+c)2/b2 = 1

x2(b2 a2m2) 2a2m cx a2(b2 + c2) = 0

b2x3 = a2my3

The locus of (x3, y3) is

b2x = a2my Hence proved.

 

Prove that the angle subtended b any chord of a rectangular hyperbola at the centre is
the supplement of the angle between the tangents at the end of the chord.

 


Let the equation of the hyperbola be x2 y2 = a2 and P and Q be any two points on it
such that their coordinates are respectively (a sec ?1, a tan ?1) and (a sec ?2, a tan ?2) and
C be the centre of the hyperbola.

Equation of the line PC is y 0 = atan?10/asec?10 (x 0)

y = x sin ?1 (1)

Similarly equation to QC will be y = x sin ?2 (2)

If a be the angle between PC and QC, then

tan = sin?1sin?2/1+sin?1sin?2 (3)

Again the equation to the tangent at P is

x a sec ?1 y a tan ?1 = a2
y = x/sin?1 acos?1/sin?1 (4)

Similarly the equation to the tangent at Q2 is

y = x/sin?2 a cos?2/sin?2

If b be the angle between the tangents at f1 and f2, then

tan = 1/sin?2 1/sin?2/1+1/sin?1 1/sin?2 = sin?2sin?1/1+sin?1sin?2

= (sin?1sin?2)/1+(sin?1sin?2)

tan = tan

tan = tan ( ) (By (3))

= Hence proved.

 

The normal to the hyperbola 16x2 9y2 = 144 meets the axes in M and N. MP and NP
and are drawn at right angles to the axes. Find the locus of P.

 


The equation to the hyperbola is

x2/a2 y2/b2 = 1 (1)

Let L be any point on it having the coordinates (3 sec ?, 4 tan ?) then the equation to the
normal at this point will be given by

3x sin ? + 4y = (9 + 16) tan ? (2)

Let this normal cut the axis of x at M whose coordinates are (x, 0) and the axis of y at N
whose coordinates are (0, y) solving (2) with y = 0, we get

x = 9+16/3cos? = 25/3cos? (3)


Similarly solving (2) with x = 0, we get

y = (9+16)tan?/4 = 25tan?/4 (4)

If PM and PN be the lines parallel to the axes, the coordinates of


P = (x, y) will be clearly given by (3) and (4). The required locus of P will be obtained by
eliminating ? from (3) and (4).

Using the fact sec2 ? tan2 ? = 1, we get

9x2 16y2 = 625

 

Prove that a circle can be drawn through the foci of a hyperbola and the points at which
any tangent meets the tangents at the vertices of the hyperbola.

 


& 

If four points (x1, y1), (x2, y2), (x3, y3) and (x4, y4) are concyclic then
These points are (ae, 0), (ae, 0), (a, b tan ?/2) and (a, b cot f/2)

apply : R1 ] R1 R2 and expand along R1, we get

= (2ae)

= (2aeb) [(a2 e2 a2 b2) cot ?/2 + 9a2e2 b2 a2) tan ?/2] = 0

points S, S, R and Q are conclycic.

 

On a level plain the crack of the rifle and the thud of the ball striking the target are
heared at the same instant, prove that the locus of the hearer is a hyperbola.

 


Suppose A to be the target and B to be the firing point and let the hearer be at a point
P. If V1 and V2 be the velocity of the sound and bullet respectively, then the time taken
by the bullet in reaching from B to the target A = BA/v2 and the time taken by the sound
in reaching from A to P = AP/v1.
Again the time taken by the sound in reaching from B to P = BP/v1.

As by hypothesis the sound reaches the hearer at P, simultaneously, the time taken by
the sound to reach upto the position P from then target together with the time taken by
the bullet to reach the target must be same as the time taken by the sound in reaching
from the rifle upto P.

Hence BA/v2 + AP/v1 = BP/v1

BA/v2 = BP/v1 AP/v1 = 1/v1 (BP AP)

BP AP = v1/v2 AB

As v1, v2 and AB are constants, hence

BP AP = Constant

Therefore the locus of the point P is the hyperbola having foci at A and B and transverse
axis equal to v1/v2 AB

 

Prove that the locus of the pole of a chord of the hyperbola which subtends a right
angles at the vertex, is, x = a2b2/a2+b2.

 


The coordinates of the vertex are (a, 0). Transferring the origin to this point, the
equation of the hyperbola x2/a2y2/b2 becomes;

(x+a)2/a2 y2/b2 = 1

x2/a2y2/b2 = 2x/a

b2x2 a2y2 = 2ab2x (1)

The equation to the polar of (h, k) w.r.t hyperbola is given by


b2hx a2hky = a2b2 (2)

After transformation the equation (2) becomes

b2(x + a)h a2yk = a2b2

or b2hx a2yk = a2b2 ab2h (3)

The equation of the lines joining the points of intersection of the hyperbola and the
chord to the origin is obtained by making (1) homogeneous with the help of (3). Hence
on simplification, this equation becomes

(a2b2 ab2h) (b2x2 a2y2) = 2ab2x(b2xh a2yk)

If they are at right angles, the sum of the coefficients of x2 and y2 must be zero; hence

b2 a2 + 2b2h/ah = 0

generalizing for (h, k), we get the required locus as

x = a a2b2/a2+b2

 

Find the locus of intersection of tangent to a hyperbola, which meet at a constant angle
.

 


Let the equation to the hyperbola be

x2/a2y2/b2 = 1 (1)

Equation to any tangent to (1) is


y = mx + a2m2b2

If the tangent passes through a point (h, k) when we must have

k = mh + (a2m2b2)

or m2(h2 a2) 2mhk + (k2 + b2) = 0 (2)

Let m1 and m2 be the two roots of this equation.

m1 = tan1 and m2 = tan2, we have

tan1 + tan2 = 2hk/h2a2

tan1 tan2 = k2+b2/h2a2

and as (tan1 tan2)2 = 4h2k24(k2+b2)(h2a2)/(h2a2)2.

= 4(a2k2b2h2+a2b2)/(h2a2)2

If the two tangents met at an angle , clearly = (1 2).

Hence cot = cot(1 2)

= 1/tan(12) = 1+tan1tan2/tan1tan2

cot2 = (1+tan1tan2)2/(tan1tan2)2 = (h2+k2+b2a2)2/4(a2k2b2h2+a2b2)2

Simplifying, the required locus is

(x2 + y2 + b2 a2)2 = 4cot2b(a2y2 b2x2 + a2b2)2

 

Find the equation to the hyperbola whose asymptotes are the straight lines x + 3y 1=
0 and 2x y + 7 = 0, and which passes through the point (1, 2).
 


Equation to the asymptotes are given as

x + 3y 1 = 0 and (1)

2x y + 7 = 0 (2)

(1) and (2) may be given by

(x + 3y 1)(2x y + 7) = 0 (3)

As the equation to the hyperbola will differ from (3) only by a constant, it may be given
by

(x + 3y 1)(2x y + 7) = (4)

(where is a constant)

(1, 2) lies on the curve given by (4), we have

(1 + 6 1)(2 2 + 7) =

= 42

Hence the equation to the hyperbola will be

(x + 3y 1)(2x y + 7) = 42

2x2 xy + 6xy + 7x 3y2 + 21y 2x + y 7 = 42

2x2 3y2 + 5xy + 5x + 22y 49 = 0

7) 
@
!  


 
  

   

D
9,&)=

We constantly describe both the shapes and positions of three dimensional objects. For example,
one simple way to describe a 7A is to approximate its shape as a mesh of triangles. Each
triangle is de?ned by three vertices, and the positions of each of these vertices will have to be
described by three coordinates [x,y,z]t.

In this chapter, we shall study the   


 
 and   
  of a line joining two points
and also discuss about the equations of lines and planes in space under different conditions,

 !
 !
/ 

 
/   
 !
 !C!

 

 
 
.

Lower level geometry of planes and graphical two dimensional coordinates moves into a three
dimensional space in  
 
 .

@  

E&
  
 
 

E  
 
 


E  
& 

E  %


EA 
 %


E@7


E@7   %


E   
 !
@!=


%


E@

E7)   

E7)   

E7)   

E7)   B

E7)   B

E7)   B


& @=)+%& ,,&=@F@9= 

Let O be any point in space

and be three
lines perpendicular to each other. These
lines are known as coordinate axes and
O is called origin. The planes XY, YZ, ZX
are known as the coordinate planes.


 


 

Consider a point P in space. The


position of the point P is given by triad
(x, y, z) where x, y, z are perpendicular
distance from YZ-plane, ZX-plane and
XY-plane respectively.

If We assume i, j, k unit vectors along


OX, OY, OZ respectively, then position
vector of point P is xi + yj + zk or
simply (x, y, z).


x-axis = {( x, y, z) | y = z = 0}

y-axis = {(x, y, z) | x = z = 0}

z-axis = {(x, y, z) | x = y = 0}
xy plane = {(x, y, z) | z = 0}

yz plane = {(x, y, z) | x = 0}

zx plane = {(x, y, z) | y = 0}

2 2 2
OP = x + y + z

 
 , 


Shifting the origin to another point


without changing the directions of
the axes is called the translation of
axes.

Let the origin O be shifted to


another point
O' (x', y', z') without changing the
direction of axes. Let the new
coordinate frame be O'X'Y'Z'. Let P
(x, y, z) be a point with respect to
the coordinate frame OXYZ.

Then, coordinate of point P w.r.t.


new coordinate frame O'X'Y'Z' is (x1,
y1, z1), where

x1 = x x', y1 = y y', z1 = z z'

ccccccccccccccccccccccccccccccccccccccccccccccccccccccccccccccccccccccccccccccccccccccccccccccccccccccccc

R  c
cccccc`cc ccc
c c c c cc c cc cc
ccc  c cc c c c ccc cc 

i   x' = x x1, where (x1, y1, z1) is the shifted origin

y' = y y1

z' = z z1

x' = 0 1 = 1

y' = 4 2 = 2

z' = 5 + 3 = 8

The coordinates of the point w.r.t. to new coordinate frame is (-1, 2, 8).

= 
Distance between the points P(x1, y1, z1) and Q (x2, y2, z2) is

(x1 x2)2 + (y1 y2)2 + (z1 z2)2

The point dividing the line joining P(x1, y1, z1) and Q(x2, y2, z2) in m : n ratio is

(mx2 nx1 / m + n, my2 ny1 / m + n, mz2 nz1 / m + n) where m + n 0 .,

The coordinates of centroid of a triangle having vertices A (x1, y1, z1), B (x2, y2, z2) and C (x3, y3, z3) is
G (x1 + x2 + x3 / 3, y1 + y2 + y3 / 3, z1 + z2 + z3 / 3).

R  c cccccccc  c cc cc cc c c c c c c c
 c c cc  ccc

i  cccccccLet the coordinates of the required point be (x, y, z), then

x = 2(3) + 3(5) / 3 + 5 = 21/8

y = 3(3) 4(5) / 3 + 5 = 11/8

z = 4(3) + 7(5) / 3 + 5 = 47/8

Hence the required point is (21/8, 11/8, 47/8).


R  ccccccc c ccc cc c c cc
c
c
c cc
c c c c  

i   The general coordinates of a point R which divides the line joining A (3, 2, 4) and B (1, 1, 1)
in the ratio : 1 are ( + 3 / + 1, 2 / + 1, + 4 / + 1) (1)

If C (1, 4, 2) lies on the line AB, then for some value of m the coordinates of the point R
will be the same as those of C.

Let x-coordinate of point R = x-coordinate of point C.

Then, + 3 / + 1 = 1 => = 2

Putting = 2 in (1) the coordinates of R are (1, 4, 2) which are also the coordinates of C.

Hence the points A, B, C are collinear.

& @,= ,=,X%=


If , , be the angles which a given directed line makes with the
positive directions of the co-ordinate axes, then cos, cos, cos are
called the direction cosines of the given line and are generally
denoted by l, m, n respectively.

Thus, l = cos, m = cos and n = cos

By the definition it follows that the direction cosine of the axis of x


are respectively cos0, cos90, cos 90i.e. (1, 0, 0).

Similarly direction cosines of the axes of y and z are respectively (0, 1,


0) and (0, 0, 1).


& 
 !
   
 


Let OP be any line through the origin O which has direction


cosines l, m, n.

Let P (x, y, z) and OP = r

Then OP2 = x2 + y2 + z2 = r2 . (1)

From P draw PA, PB, PC perpendicular on the coordinate


axes, so that

OA = x, OB = y, OC = z.

Also, pPOA = , pPOB = and pPOC = .


 & 
 !
  
 

From triangle AOP, l = cos = x/r => x = lr

Similarly y = mr and z = nr

Hence from (1)

2 2 2 2 2 2 2 2 2 2 2
r (l + m + n ) = x + y + z = r => l + m + n = 1

= 

If the coordinates of any point P be (x, y, z) and l, m, n be the direction cosines of the line OP, O being the
origin, then (lr, mr, nr) will give us the co-ordinates of a point on the line OP which is at a distance r from
(0, 0, 0).

  
& 
If a, b, c are three numbers proportional to the direction cosine l, m, n of a straight line, then a,
b, c are called its direction ratios. They are also called direction numbers or direction
components.

Hence by definition, we have

1/a = m/b = n/c = k (say)


=> l = ak, m = bk, n = ck => k2(a2 + b2 + c2) = l2 + m2 + n2 = 1

=> k = 1 / a2 + b2 + c2 = 1/a2

l = a/a2. Similarly m = b/a2 and n = n/a2

where the same sign either positive or negative is to be chosen throughout.

 If 2, 3, 6 be the direction ratios, then the actual direction cosines are 2/7, 3/7, 6/7.

= 

Direction cosines of a line are unique but direction ratios of a line in no way unique but can
be infinite.

  cc
i cc c 
c c c  c cc  c c  c cc cc cc
   c cc c cic ccc c  c c cc  c c  ccc cc
c c c  c c  c c  c cc  c ccc c

V 
 

 
 

    
 
    c

@ c
c cc

cc

cccccccccccccccccccccccc cccccccc cccccccccccccccccccccccccccccccccccc c

cc

cccccccccccccccccccccccc c!"cc "cc"cccccccccccccccccccccccccccccccccccccccccccc" c

cc

cccccccccccccccccccccccc# c c
ccc c c c" cc cc

cc

cccccccccccccccccccccccc! cc "cc "cc"ccc


cc

ccccccccccccccccccccccccc$ "cc" cc%"cccc" cc  cc" ccc

cc

ccccccccccccccccccccccccc &cc&"ccc"&c

cc

cccccccccccccccccccccccc c &cc&"c ccc" c

cc

ccccccccccccccccccccccccccc cccc cc& ccc

cc

cccccccccccccccccccccccc cc cccc


c

cc

ccccccccccccccccccccccccc&cc &cc&"cc'"cc "cc" c&c'( "cc"cc"")cc&


%c

cc

ccccccccccccccccccccccccic  c ccc cc&


%c&
%c"&
%c

cc

cccccccccccccccccccccccc* c c &cc"&cccc &"c

cc

ccccccccccccccccccccccccccc cc &"cc &"cc& cc&"c

cc

cccccccccccccccccccccccc@ c &cc"&c c & cc&"c


c &cc &"cc&cc&
"cc""cc"cc&
+c

cc

ccccccccccccccccccccccccc@ c  c cc c c cc&


+c"&
+c&
+c

(
          cc
cc
cc

@ c  c cc c#,c-  c#c.c/c0 c c,."c


/"c0" cc."cc.cc/ c/"cc/ccc/ c c0"cc0ccc
/ c

cc

@ c  c cc

cc
cc."cc. c&c'1."cc. "c cc/"cc/ c&c'1."cc. "ccc0"cc
0 c&c'1."cc. "c
c

cc

!  " 

 


c

cc

V 
 

 
 

 
 
#

$ %

!&'(()* +&(()* c

cc

ccccc2  c cc*3cc+cc+cc" cc"cc"cc c

cc

cccccccccccc"cc"cc"cc4c

cc
cc

cccccccccccc2  c ccc"&cc"&cc& ccc

cc

    c

cc
cc

 c5cc cc c c   c c*3c c*6c c  c


 cc
c c  c ccc cc c"c "c
"c
/c c
c/c7cc"cc  "cc"c

cc
c

8c  c cc c ccccc c"c"c"cc


c cc
 c c c c
c/c
cc

c5cc ccccccccccccccccccccccccccccccccccccccccccccccc c . 9cc c c c

cc

 
   c

" "

c :c 
c  5cccc 5c

ccccccccccccc "cc "cc"  ""cc ""cc"" cc"cc  "cc" "c

cc
" " "
ccccccccccccc "cc"  cc "cc " cc"cc" c

cc
"
ccccccccccccc 5ccc'; "cc"  cc

cc


c    
   c

cc

8c c
c cc c c5cc4c cc5ccc

cc

c"cc  "cc"cccc"cc"cc"cccc

cc


c       c

cc

8c c
c ccc c7cc c c c5ccc

cc

<c "cc"  "cc "cc " "cc"cc" "cc


cc

  c c c/c c

cc

 "cc" ccc "cc "ccc c"cc"cccc8&"cc & "cc&"c

cc

i /c&"cc&"cc&"c

cc

V ',   
-
$

 
)(( ((%%
. c

cc

ccccc"cc"cc"cc " cc " cc" " cc"cc%cc+ccc

cc

ccccccccccccc cc c

A 
 %

c

#- cc c c-  c c  c#c.c


/c0 c c,c."c/"c0" cc c c
 c  c ccc cc c
c

*3cc."cc. cc /"cc/ cc0"cc0 c


cc
cc

 ( 
       c

cc

 c*3c c   c c c  c  c*ccc c c



c  c cc cc

*=cc- cc c*#cc   c c*3c

c
ccccccc.cc cc /cc cc0cc c

" " "


 c*#cc'. cc/ cc0 c

c c  cc  c#c

ccccccc#=cc'*#"cc*="c
cc

cc

V 
. %%
.
  %
/&()(* 
$ 
%
$
.$!&)((* -
$

 
)(( c

cc

ccccccccccccccccc2  c cc c c c&'"cc""cc""c"&'"cc""cc""c"&'"cc""cc""c

cc

 c&c"&c"&c

cc

ccccccccccccccccccccccccc#=cc.cc cc /cc cc0cc cccccccc&cccc cc"&ccc cc"&cc" cc&c

cc
"c " "
cccccccccccccccccccccccc*#cc' ccc cc" cc'%c

cc
" "
ccccccccccccccccccccccccc# c  c#=cc'*# cc#= cc'%"&4cc'"4&c

cc
cc
    c cc

cc cc

cc
c

c
cc

iccc>*36c c
c/c c c>"cc>."cc>/"cc>0"c

cc

@ %=cc
c
(   c6 c cccc  c#.c/c0 c8c.c/c0cc c c
/c  c/c   c
c  c  c   cc 
cc cc  c ?c#c@ ccc  c c  c c c
 c ccc#c8 c /ccccc/c
 cc8c,cc/c c  cc @cc cc
  cc c   c c#,c cc c c ccc8c c c c c   c c#,c 
c c
 c c  
c  c c /cc c  /cc cc c ccc  c/c
 c
c

R     (   c


c Ac7 cccc c.cc/cc0cc ccc

c B7 cc cc c= c c c.cc /cc0ccc cc c c cc c c c c
  c c cc cc c cc c  c cc c c

c @ c7 c c cc c  c#.c/c0 c c 
c  c c

cc cc c c c.cc. cc/cc/ ccc0cc0 ccc


c @ c7 cc cc c  c cC c  c.c/c0 c

."c/"c0" c c.c/cc0 c c ccc


c @ c7 cc cc c   cccccc c.c/c0c.c
/c c.&cc/&cc0&cc
ccccDc c
!
c B7 ccEFcc c.ccccccccccccc7 cccc cEFcc c.cc c

cccccccccccB7 ccFGcc c/ccccccccccccc7 cccc cFGcc c/cc c

cccccccccccB7 ccGEcc c0ccccccccccccc7 cccc cGEcc c0cc c

"
c Hc  c /c*c.c/c0 c3c."c/"c0" c6c.c/c0 c c2c.+c/+c0+ c ccc cc  c
 cc cc c  c c c  c

cc

6G IcC$9cH  c c7 c c cc c  c c  c"cCc c  c c c cc c
 c c c c  c c cc

c

@ c  c cc c c c ccc"cCcc

cc

cccccccccccccccccccccccc@ c7 cc7  cc c".c" cc/cc ccc0c ccc

cc

ccccccccccccccccccccccccc".cc/cc0c+ccc

cc

   c

cc

*c c cc c   ccc c cc cc c c/c  c c c


c

cc

*c c cc.cc/cc0cc ccc c".cc"/cc"0cc "ccc

c c

cc

=  c
cc

ccccccccc8c"c"c"ccc c ccc c c c c

cc

ccccccccc8c&"cc&"cc&"c c cccc c c c

cc

6G IcC49cH  cc c cc".cc/cc0ccc c.cc/cc"0ccc

cc

cccc c

cc

cc

cc

6G IcC9cccccccccccH  c c7 cc cc c  c"cc+ ccc c cc c.C
. c

cc

@ c7 cc cc c  c"cc+ c c

cc

cccccccccccccccccccccccc.cc" cc/cc cc0cc+ cccccccccccccccccccccccccccccccc c

cc

cccccccccccccccccccccccc ccc c cc cc 


c

cc

cccccccccccccccccccccccccc+cc!cccccccccccccccccccccccccccccccccccccccccccccccccccccccccccc" c

cc
cccccccccccccccccccccccc c7  cc cc c.C. c c c cEFcc c

cc

ccccccccccccccccccccccccccccccc<cccc<c+cc!cccc&!cc&+c

cc

cccccccccccccccccccccccccB7 cc7  cc c!/cc+0ccc

cc


 ( 
 c

@ c cc c c c c  c#.c/c0 c c cc.cc/cc0cc ccc cJ.cc/cc


0cc c&c'"cc"cc"Jc

    c

B7 ccc c  c c cc   cc ccccc c


ccc cc
ccK
ccc

` 
      c

8c7 cccc c.cc/cc0cc ccc c  c cc c c ccccccic
c c c c cc cc   c c 
c  c cc cc c
c/c
c5cccc ccc&c'"cc"cc"c

@ cc c cc c c   c c cL&"cc5c

#
c #c c   c c ccc ccc ccccc
$
c #c c   c c cc c c&cc& cc&c

cc

6G IcC9cccccccccccH  c c7 ccc c  c c   ccc".cc+/cc0cc


cc.cc/cc0cc%c cc c   c c 
c  c ccc c

B7 cc7  ccc

cccccccccccccccccccccccc".cc+/cc0cc ccK.cc/cc0cc% ccc

cc

cccccccccccccccccccccccc c"ccK .cc+ccK /cc0ccK cccc%K ccccccc

cc

cccccccccccccccccccccccc@ cc cc cc   c ciccc ccccc


cc

cccccccccccccccccccccccc"ccK cc +ccK cc ccK ccc cKccc

cc

cccccccccccccccccccccccccB7 cc7  cc c.cc!/cc%0cc"ccc

cc

cccccccccccccccccccccccc c.cc!/cc%0cc"ccc


       cc

@ c7 cc cc  c cc c c


cc.c/c0c ccc c".c
c"/cc"0c "ccc c

cc

cc

@ @&) @%=
i   c c c c  c  /c c   cc   cc ccicc7 c
c   c c c   cc c7 cc cc  c cc7 cc   c c c.c
c/cc0cc ccc".cc"/cc"0cc "cccccccccccccccc c

c
ic7 c c c   c c  c c   c/c   cc cc

R       (   c

 
  cc
cc

ccccccccccB7 cc   c c c  c  c#c.c/c0 c c c  c ccc cc
c..cc/cc/& cc0cc0c&cc

%
c B7 cc   c c c  c c  c#c.c/c0 c c,c."c/"c0" c ..c&c."cc.cc//c&c
/"cc/cc0cc/c&c0"cc0c

cc
=  cccccc@ cc  ccc  ccc c c
c/c.ccc/cc c0cc c cc c
 c c  c.c/c0 c c  c c  c c cc c

6G IccccH  c c7 cc c   c c  c c  ccc c  cc

cccccccccccc cc c0C. c

cccccccccccc c c c0C. c

  
c cB7 cc   c cc c0C. c 
cMcc4 cNcc4 cOcc c

cc

cccccccccccccccc cccccc

cc

cccccccccccc@ c7 cc   c c cc c0C. c c c  ccc c c.ccc&ccc/c
cc&ccc0ccc&cc

cc

 cccccccc7 cc   c c c c0C. c

cc

cccccccccccc c /c ?cMcNcc c.c c/c.c


/c

cc

cccccccccccc@ c7 cc   c c c c0c. c c c  ccc c c.c&ccMc
c/ccc&c cMcc0ccc&cc

cc

ccccccccccccc.ccc&ccc/ccc&c cc0ccc&cc

cc
cc

6G IcC"9cccccccccccH  c c  cc c  c c c c-  c c  c"cc c cc
+c c c cc".cc/cc0cc!c

cc
cc

@ c  c cc c cccc"c+cc cccc ccc%c

cc
ccccccccccccccccccccccccPc7 cc c c-  c c
c  c c

cc

cccccccccccccccccccccccc."c&ccc/ccc&ccc0ccc&cc%ccc/ cccccccccccccccccccccccccccccccccccccccccccccccc

cc

cccccccccccccccccccccccc6  cc/c  cc c cccc"cccc%cc c

cc

cccccccccccccccccccccccc8c c  c cc c


cc".cc/cc0cc!c c

cc

cccccccccccccccccccccccc"cc" cccc cc%cc cc!ccccc

cc

cccccccccccccccccccccccc6  cc c  cccc"c ccc% cc c cc"c! c

cc

=  c

&
c 8c7 cc   c c c
c cc c ccc  c  c/   c c@ c   c
c   c c cB. c

cc

6G IcC9cccccccccccH  c c/   c c c7 cc c c

cc

cccccccccccccccccccccccc.cc"/cc0cc+ccc+.cc/cc"0ccc

cc
cc

ccccc@ c7 cc c c cc cc

cc

cccccccccccccccccccccccc.cc"/cc0cc+ccc+.cc/cc"0cccccccccccccccccccccc c

cc
cccccccccccccccccccccccc cc ccc c  c cc c ci c c c c c c c cc
c c c c c c c c cc

cc

ccccccccccccccccccccccccPccc" ccccc+cc cc"ccc

cc

cccccccccccccccccccccccci
c cc c

cc

cccccccccccccccccccccccc&+cc +%cc&$c c&cc &"cc&cc&' "cc""cc "cc&'$c

cc

ccccccccccccccccccccccccic  c cc c cc&'$c"&'$c&'$c

cc

cccccccccccccccccccccccc=c c  c c  ccc  ccc c cc  c c c  c c c  c c
c0ccc# c0ccc c c7 c
c/c cc 
c

cc

cccccccccccccccccccccccc.cc"/cc+ccc+.cc/ccccc

cc

cccccccccccccccccccccccc
c cc c.cc"c/ccc

cc

ccccccccccccccccccccccccicc  cc c c c"cc c

cc

ccccccccccccccccccccccccc7 cc c c c/   c c c

cc
cc

ccccccccccccccccccccccccc c c

c
   
 !
 !




%

c
@c cc cc   c c c /c c c c c c cc@ c   c c  c c
 c c ccC   c c c c c cc   c  c* c cc
   c  c c    c c c c c c cc   c  cc
cc

    c c7 cc cC   c cc

cc

cccccccccccc..c&ccc//& cc00&ccc/ ccccccccccccccccccccccccccccccccccccccccc c

cc

cccccccccccc* c.."c&c"cc//"& "cc00"&"cc"c/ cccccccccccccccccccccccccccccccccc" c

cc

*/c  cc c c c#c.ccc/cc c0cc c cc c" c c

cc

,c."cc""c/"cc ""c0"cc"" c

cc

 c#,cc c cc   c  c8 c  c c cc

cc

Qcc.c."c"" c cc/c/"c "" ccc0c0"c"" Rc

cc

@ c c c c c c


c c3/c c cc  /cc  c"c7 c
cc c"c

cc

ic/c
c c
ccc c"ccc c* c7 /c  c#c c,ccc c@ c
 c#,c c   c  c

cc
@ c   c  ccc c/c#,cc c

cc

=  c

'
c 8c/c   c c c
c cc c c ccc   c  c/   c c ccc
 c c

cc

V )
   
     
("010)20)("0
02'0 !
 3.
 
  
   c

cc

cccccA
c cc.&cc/$&cc0&ccc/ ccccccccccccccccc c

cc

cccccccccccccccccccccccccc.c&ccc/cc!c&c"cc0cc%c&c+cc"c/ cccccccccccccccc" c

cc

cccccccccccccccccccccccc*/c  cc c c c#cccc$ccccc c cc c" c c

cc

cccccccccccccccccccccccc,ccc"c""cc!c+"cc% c

cc

cccccccccccccccccccccccc8c#,c c cc   c  c c  c cc#,c

cc

ccccccccccccccccccccccccccc cccc" c$cc cc""cc! cc cc+"cc% c

cc

cccccccccccccccccccccccc ccc"cc%ccc"ccccc+"ccc

cc

cccccccccccccccccccccccc*c#,c c c c c c c" c


cc

ccccccccccccccccccccccccccc"cc% cccc""cc cccc+"cc ccc

cc

ccccccccccccccccccccccccccc!"cccccccccccccccccccccccccccccccccccccccccccccccccccccccccccccccccccccc c

cc

cccccccccccccccccccccccc ccc"cc% cc"cc""cc cc+cc+"cc ccc

cc

cccccccccccccccccccccccc c!cc"ccccccccccccccccccccccccccccccccccccccccccccccccccccccccccccccccccccc+ c

cc

ccccccccccccccccccccccccSc
c7 c c c+ cc ccc"cc

cc

ccccccccccccccccccccccccic  c#cc$ c c,cc!c% c

cc

ccccccccccccccccccccccccc cc   c  c#,cc'( "cc!$ "cc% ")cc'c

cc

cccccccccccccccccccccccc2  c cc   c  c c c"ccc

cc

cccccccccccccccccccccccccB7 cc   c  c c.&"cc/$&cc0&c

@  &
c
*c c ccccc  c  c 
c cc c  c c  c cc . c  c c  c
H . c  c c c cc c c  c  c c c cc c

R    (   c

(
c 8c cc c ccc c c c cc c7 cc c c

cccccc.cc "cc/cc "cc0cc "cc"c


)
c 8c cc c c  c c c cc c."cc/"cc0"cc"c
*
c u   c@ cc7 ccc c c."cc/"cc0"cc".cc"
/cc"0cc ccc

" "c "


6 cc ccc
c c cc'T cc
c cc c

+
c (    cB7 ccc c c.  cc   cc*c.c/c0 c c3c."c/"c0" c c
.cc. c.cc." cc/cc/ c/cc/" cc0cc0 c0cc0" ccc

6G IccH  c c7 cc c c  cc  c c  ccc+ ccc" c cc
c c c c c cc cc.cc/cc0cccc
cc

ccccc c7 cc c cc

" " "


cccccccccccc. cc/ cc0 cc".cc"
/cc"0cc cccc

cccccccccccc c c cCccCc


cCc c  c cc.cc/cc0ccc

ccccccccccccccc
cccccccccccccccccccccccccccccccccccccccccccccccccccccccccccccccccccccccccc c

cccccccccccc cc  ccCcc+ cc"cCc%


cc$cc ccCc"%cccccccccc" c

cccccccccccccCcc" cc"ccCcc
cc+ccc ccCcccccccccccccccccccccccccccccccccc c

cccccccccccc c cc  ccCcc cc"ccCc%c


cc ccCcccccccccc+ c

cccccccccccc
c cc7 cc c

ccccccccccccccCcc
cccccCc"c c ccc

cccccccccccc@ c7  c7 cc c c c

" " "


cccccccccccc. cc/ cc0 cCc"c.cc%c/cCc+c0cccccc
c
cc

6G IcC9cccccccccccH  c c7 cc c c c c c"cc+ c c  cc  c
c  cc"c c

" "
cccccU cc cc'(" "cc" cc+ )cc'c

" " " "


cccccccccccccccccccccccccB7 cc c c c.cc" cc/cc cc0cc+ cc' c

" " "


cccccccccccccccccccccccc c. cc/ cc0 cc+.cc%/cc$0cc""ccc

,%B&,-%9
 R `Rc

ccccccccccccccccccccc@ cc c c c c  c cc


c/c c7 ccc cccc
c"cc "cc"ccc c

cc

&!*L&&+*L&c

cc

&V*L&+&4*5  c

cc

cccccccccccccccccB  cc c c c cc 


c

cc

cccccccccccccccccccccccc"cc "cccc "cccc" cccc ccccc ccc

cc

cccccccccccccccccccccccc cccc c ccccc c cccc

cc

ccccccccccccccccccccccccc&cc &cc&c
c c  c ccc c

cc

cccccccccccccccccccccccc8c ccc cccccc ccccc

cc

ccccccccccccccccccccccccc&cc &cc&c
c  c cc c c c

cc

cccccccccccccccccccccccc@ cc c c c c

cc

cccccccccccccccccccccccc c

cc
"ccccccccccccccccccccc@ c7 cc cc  c  c c cc   cc cc.cc/cc0cc%c
cc c".cc/cc0ccccc c c c c./cc 9c

cc

&!*.cc"/cccc&+*.cc"/ccccc

cc

&V*.cc"/cccc&4*.cc"/ccccc

cc

B7 cc c7  cc c.cc/cc0cc% ccK".cc/cc0cc ccc

cc

cccccccccccccccccccccccc ccc"K .ccccK /ccccK 0cc%ccK ccc

cc

cccccccccccccccccccccccc@ cc c c c./cc c7 c c0ccc

cc

cccccccccccccccccccccccc ccc.cccc/cc0ccc

cc

ccccccccccccccccccccccccc3/c cc  /c

cc

cccccccccccccccccccccccccc"K cccccK ccccK cccc cKccc

cc

cccccccccccccccccccccccccB7 cc7  cc c

cc

cccccccccccccccccccccccccc" .cccc /cccc 0cc%cc cccc.cc"/ccccc

cc

ccccccccccccccccccccc@ c  cc c cc c c c c c  c c cc.cc+/c
c%0cccccc9c

cc
&!*&%c+&%c%&% &+*&%c+&%c%&% c

cc

&V*&%c+&%c%&% &4*&%c+&%c%&% c

cc

@ c7 cc cc c.cc+/cc%0cccccccccccccccccccccccccc c

cc

@ c  c cc c c c cc ccc+c%cic7 cc c c
 ccc c c c c cc cc

cc

cccccccccccccccccccccccc.&cc/&+cc0&%ccc/ cccccccccccccccccccccccccccccccccccccccccccccccccccccccccccccccccccccccc" c

cc

cccccccccccccccccccccccc@ c  cc/c  c#cc" ccc+c% c8c c  c cc cc c c

cc

cccccccccccccccccccccccc cc++ cc%% ccccc ccc&%c

cc

cccccccccccccccccccccccc# c c
ccc  cc c cc c c#cc&%c+&%c
%&% c

cc

+ccccccccccccccccccccc@ c  cc c  cc"c c c cc.cc/cc0ccc  cc c c


 c.&"cc/&cc0&%c 9c

cc

&!*!c cccccccc&+*+c c

cc

&V*c cccccccc&4*"c c

cc

Pccc c c  c c  cc c  c c cc c  c


 cc c c
c c@ c   c cc9c
cc

@ c7 cc c c  c c  cc"c c cc c
c c c

cc

cccccccccccccccccccccccc.&"cc/"&ccc0&%ccc/ c

cc

cccccccccccccccccccccccc@ c  cc/c  cc c c"ccccc"c%cc c

cc

cccccccccccccccccccccccc8c c  c c c c


cc c

cc

cccccccccccccccccccccccc"cccccc" cc%cc ccc<c!cccccc&!c

cc

ccccccccccccccccccccccccc  cc   c c4&!c&!c&! c

cc

ccccccccccccccccccccccccc@ c7  c  c

cc

ccccccccccccccccccccccccc c  c c c  cc"c c cc4&!c&!c&! c

cc

ccccccccccccccccccccccccc'4&! "cc"&! "cc&! "cc&!cc'+4ccc c

cc

ccccccccccccccccccccc@ c cc c  ccc+ c c cc".cc/cc0ccccc c9c

cc

&!*cc" ccc&+*c"c c

cc

&V*cc" c&4*"cc c

cc
*c c cc c c c  c#,c c
c c c cc c cc c
 c c c  cc#,c c=c

ic  c cc c#,c c"cc


c

cccccccccccccccccccccccccB7 cc c#,cc.&"cc/&


cc0+&cc/ c

cccccccccccccccccccccccc*/c  cc c#,c c"ccccccc


c+ c c

cccccccccccccccccccccccc8c c  c cc cc c

cccccccccccccccccccccccc""cc cccc cccc+ cccc


ccccc

ccccccccccccccccccccccccc  cc cc c=c


cc+c c

cccccccccccccccccccccccc*c=c c c  cc#,c


cc

ccccccccccccccccccccccccccc.&"c+cc/&"ccc+0&"cc

cc

ccccccccccccccccccccccccc.ccc/ccc0cc"c

cc

ccccccccccccccccccccccccc8 cc  c#ccc+ c c c  c,ccc" c

cc

%ccccccccccccccccccccc@ c7 cc c c  cc  c c  ccc ccc c ccc c
 c c c cc cc c c9c

cc

&!*."cc/"cc0" cc".cc/cc0 cccc

cc

&+*."cc/"cc0" cc".cc/cc0 cccc

cc
" " "
&V*. cc/ cc0 cc".cc/cc0 cccc
cc
" " "
&4*. cc/ cc0 cc".cc/cc0 cccc

cc

 c7 cc cc


c/c

cc

."cc/"cc0"cc".cc"
/cc"0cc ccccccccccccccccccccc c

cc

cccccccccccccccccccccccc*c cc  c  ccc ccc c ccc cicc 


c

cc

cccccccccccccccccccccccccc"cc ccccc"
cc ccccc"cc ccc

cc

ccccccccccccccccccccccccSc
ccc
cccc&"c cc c

cc

cccccccccccccccccccccccc8cc c c cc c c c

cc

cccccccccccccccccccccccccc'"cc
"cc"cc c

cc

cccccccccccccccccccccccc"cc&+c cc "cc cc c/ c

cc

cccccccccccccccccccccccccc cc   c

cc

cccccccccccccccccccccccc T& cccc&+" cc cccccc cc&c

cc

cccccccccccccccccccccccc*c "T& "cc&"c


c c cc&c

cc
ccccccccccccccccccccccccPc c c   c c cc&c

cc

ccccccccccccccccccccccccic  c
cc cc 
ccc
cccc&c

cc

ccccccccccccccccccccccccc7 cc c c

cc

cccccccccccccccccccccccc."cc/"cc0"cc.cc/cc0 cc&cccc."cc/"cc0" cc".cc/cc0 cccc

cc

!ccccccccccccccccccccc*c  c 
cc  c c cc c  c c c . c  c c  c8 cc c
9c

cc

&!*cccccccc&+* c c

cc

&V* ccccccccc&4* c

cc
cc

 c c  cc 


c  c#cc.c/c0 c c*ccc c c3ccc cc c
 . c  c* c c
c c

cc

cccccccccccccccccccccccc*#&3#cc  cc?c/ cc*#"cc?"3#"c

cc
" " " " " " "
ccccccccccccccccccccccccc.cc cc/cc cc0cc cc? (.cc cc/cc cc0cc )c

cc
" " " " " " "
ccccccccccccccccccccccccccc? . cc/ cc0 cc".cc? cc cc? ccc

cc
" " " " " "
cccccccccccccccccccccc7  cc c. cc/ cc0 cc"? &? cc ccc:  c cc c
cc

1 6


2% 
-
 
#

$&((* &((*c

cc

&!*&+*c

cc

&V*&4*c

cc

cccc c c ccKc9cc

cc

ccccccccccccc.C  c cK"c&cKccccKcc"&cc

cc

ccccccccccccccccccccccccPc* c c c c c

cc

!
7 $8(9(:(;
 .
$ .  8
9:;c

cc

&!*)&+*0c

cc

&V*0&4*0c

cc
@ c  c cc c cc c
ccc c

cccccccccccccccccccccccc2  c cc&'c&'c&' c

cccccccccccccccccccccccci /c  c cc*icc&'c


&'c&' c

cccccccccccccccccccccccc3#cc&'c&'c&' c
cc
cccccccccccccccccccccccc6,cc&'c&'c&' c

cc

cccccccccccccccccccccccc cc ccc  c cc c c

cc

ccccccccccccccccccccccccMcc &'cNc &'cOcc &'cVcc &'c

cc

cccccccccccccccccccccccc"cc"Ncc"Occ"Vcc+"cc "cc" c&ccc+&cc c"cc "c"cc c

cc

ccccccccccccccccccccccccPc3 c c c cc

) 6%
&()()*(&((*(&(()* &((*c

cc

&!*
&+*% c

cc

&V*
$3.&4*   c

cc

ccccB7 cc cc c  c c  ccCcC cC+c+c+ c c+cc c

c ccccccccccccccccccccccccc c

cc

ccccccccccc@ c  cc4c+ c  c c7 c c

cc

ccccccccccccPc3 c c c c c

cc
)) !-
% % .$
" %
&((*   
 "
!(
+(V 6. %
  %  .$!(+(V% 
 % 
c

cc

&!*22"""2&+*"22""2c

cc

&V*"22"&4*"2c

cc

cccc c c7 c c ccc.&Mcc/&Ncc0&Occc

cc

ccccccccccccccccccccccccc.&Mcc/&Ncc0&Occc ccc  ccc c

cc

cccccccccccccccccccccccc=c c  cc   cc cc c c  c.cc*Mcc c3cNc


 cWc6ccO c

cc

cccccccccccccccccccccccccB7 c ccc c c  cc c c  c*c3cWc6cc


.ccMc/ccNc c0ccOc

cc

ccccccccccccccccccccccccc@ ccc c c  c c

cc

cccccccccccccccccccccccc&.cc&/cc&0ccc/c  cMcNcOc c


c7  c

cc

ccccccccccccccccccccccccPc* c c c c c

cc

) V 
 
$   c

cc

!
 &!* % 2)
% ""
 c
cc

< &<*   % 


% ""
 c

cc

=    c

cc

&!* ! < . <


  "% 
 !c

cc

&+* ! < . <


   "% 
 !c

cc

&V*!
 .. <
c

cc

&4*!
. <
 .c

cc

A
cc/cc0ccccc cc c.C. ccccccccc

cc

cccccccccccccccccccccccc c c c cc cc c c.C. c

cc

ccccccccccccccccccccccccPc6 c c c cc

cc

) 63.
  %  

$ 
"80(90 2:0 
&"8*
&9*&2:*( 
3. c

cc

&!*)&+*)c

cc

&V*&4*c
cc

i c   c c c c ccc c cc c c c c c c
c
ci c  c cc   c ccc cc c  c cc c c cccc
cciccc ccccc

cc

ccccccccccccccccccccccccPc2 c c c cc

cc

) 6  
       
$ 
"00'00200
 '0120"0
c

cc

&!*>&+*c

cc

&V*&4*'>)c

cc

i c c c cc   cc   c  c c c c ccc

cc

ccccccccccccccccccccccccPc6 c c c cc

cc

) ! 
$ 
% .$ %
&()()* ? 
%  % "
2 
%%
.  
"0)020) 63.
   
$ 
c

cc

&!*"0)0)2)0)&+*"0)0)2)0)c

cc

&V*"0))0)2)0&4*"0))0)2)0c

cc

 c  c cc   c ccc cc

cc
c+cc ccccc

cc

cccccccccccccccccccccccccc" ccccc

cc

ccccccccccccccccccccccccc&cc &cc&4cc&cc &cc&c

cc

cccccccccccccccccccccccccB7 cc   c c c."&cc/&cc0&c

cc

ccccccccccccccccccccccccPc6 c c c c  c

cc

)' ? %


&)((* 
.
.
(   3.
  
%
c

cc
     
&!*"  2 "1'2)&+*&"  2 *"1'2)
c

cc
     
&V*"  2 ")')2)&4*"  2 "1'2)c

cc

B7 cc c cc.cc "cc/cc+ "cc0cc "cc4c

cc

ccccccccccccccccccccccccPc* c c c c c

ccc

) ?3.
 %
&"2*"1)2  " 



 
&()()*(    " 

  %
c

cc

&!*&(()*&+*&((*c
cc

&V*&((*&4*&(()*c

cc

@ c cc c c cc"c cc c c.  /cc   c c.c/c0 c c

cc

cccccccccccccccccccccccc."&"ccc/&"cc"c0&"ccc

cc

cccccccccccccccccccccccccU7  c  c ccc! c

cc

ccccccccccccccccccccccccPc6 c c c cc

cc

$ccccccc@ c  c cc c c  c c c c c c c  c c
 c ccc"cc" cc"c c c

cc

cccccccccccc* c"&c&c"& cccccccccccccccccccccccccccccccccccccccccccccccc3 c"&c&c"& c

cc

cccccccccccc6 c"&c&c"& ccccccccccccccccccccccccccccccccccccccccccccc2 c"&c&c"& c

cc

ccccc c  c cc c7  c cc

cc

cccccccccccc@ ccCc"ccCc"cccc

cc

cccccccccccc* c"cccccc

cc

cccccccccccccccCc"cc
cc

cccccccccccccCc"ccc+ccccccCc"cc

cc

cccccccccccc@ c  c cc c7  c ccXCc"ccCc"ccX"cCcc"c

cc

cccccccccccc  c cc c7  c c

cc

cccccccccccccc c

cc

4ccccccc@ c  c+c!c$ c"cc+ cc"c c cc"c cc9c

cc

cccccccccccc* c c
 ccc cccccccccccc3 c c

cc

cccccccccccc6 c c
 ccc 0  cccccccccccccccccc2 c/ c

cc

ccccc c*cYc+c!c$ c3cYc"cc+ c6cYcCccCc"c c2cYcc"c c

cc

cccccccccccc2  c cc*3cYc"&%c+&%c+&% cc&c"&c"& c

cc

cccccccccccc2  c cc62cYc"&%c+&%c+&% c

cc

ccccccccccccc&c"&c"& c

cc

ccccccccccccic*3cc c62c
cc

cccccccccccc2  c cc*2cYc!/$0/*c1$0/*c/$0/"c

cc

cccccccccccc2  c cc36cYc!/$0/*c1$0/*c/$0/"ccc

cc

ccccccccccccc!/$0/*c1$0/*c/$0/"c

cc

ccccccccccccc*2c cc c36c

cc

cccccccccccc@ c*362c cc c

cc

 63.
  % %  % "2) %
$ .$ 
%
&()('*
c

cc

cccccccccccc* c+.cCc/cc"0cCccccccccccccccccccccccccccccccccccc3 c.cCc+/cc"0cCcccc

cc

cccccccccccc6 c+.cCc/cc"0ccccccccccccccccccccccccccccccccccc2 c.cCc+/cc"0ccccc

cc
cc

cccccB7 cc ccc c cc+.cCc/cc"0ccccc cc c cccccccccccccccc

cc

cccccccccccc+.cc/cc"0cc?ccc c cc  cccCc% c

cc

ccccccccccccc"cCccCc"cc?ccc<c?ccCcc

cc
cc
cccccccccccc@ c7  c7 c c+.cCc/cc"0cCccccc

cc

"ccccccc*cc c c  c c c  cc c  c*c"cc c3c%cc c c c


 c c c c8 c7 c c c

cc

cccccccccccc* c".cCc/cc0cc+ccccccccccccccccccccccccccccccccccccccccccc3 c".cCc/cc0cc+c

cc

cccccccccccc6 c.cCc/cc0ccccccccccccccccccccccccccccccccccccccccccc2 c.cCc+/cc"0ccc

cc

cccccZ C  cc*cCc"cc c c3c%cc c c"cc c

cc

cccccccccccc  c cc c c-  c*c c3c cX"cCccc

cc

cccccccccccc@ c7 cc c c c c*3c c c  c"cc c c

cc

cccccccccccc".cc" cc/cc cc0cc cccc".cCc/cc0cc+c

cc
cc

""ccccccc*cc  c cC  c.c c*c3c6c c  c c  cc  c*36c ccc c


@ c7 cc cc c c

cc

cccccccccccc* c.&cc/&cc0&ccccccccccccccccccccccccccccccccccccccccccccc3 c.&cc/&cc0&ccc

cc

cccccccccccc6 c.&cc/&cc0&cc"ccccccccccccccccccccccccccccccccccccccccccc2 c=cc c

cc
cc

ccccc@ cc  c cC  c.c c*c3c6c c  c  cc c  c*36c ccc c


cc

ccccccccccccc cc cGC. c cccc c

cc

ccccccccccccicGC   ccccc

cc

cccccccccccc@ cc cEC. c cccc c

cc

cccccccccccccEC   cccc

cc

cccccccccccc cc cFC. c cccc c

cc

cccccccccccccFC   cccc

cc

cccccccccccc@ c7  c7 c c.&cc/&cc0&ccc

cc

ccccccccccccc.&cc/&cc0&ccc

cc
cc

"ccccccc@ c cc c c.cc .cc cc/cc" /cc+ cc0cc 0cc ccc  c

cc

cccccccccccc* c'"cccccccccccccccccccccccccccccccccccccccccccccccccccccccccccc3 c"c

cc

cccccccccccc6 c'cccccccccccccccccccccccccccccccccccccccccccccccccccccccccccc2 cc

cc
cc

ccccc.cc c.cc cc/cCc" c/cCc+ cc0cc 0cc ccc c c


c7 cc c
cc

ccccccccccccic c  cc c   cc

cc

ccccccccccccCcc"cCc c cCcc+cCc c

cc
" " "
cccccccccccc cc'" cc" cc" cc'c

cc
cc

"+ccccccc@ c cc c  c ccc   c c c

cc

cccccccccccc* c0cccccccccccccccccccccccccccccccccccccccccccccccccccccccccc3 cc

cc

cccccccccccc6 c  cccccccccccccccccccccccccccccccccccccccccccccccccc2 ccc c

cc

ccccccMccccNcc ccOccc

cc

cccccccccccc cc  c ccMcccNcccOc

cc

ccccccccccccccc ccc

cc

cccccccccccc  c c  c

cc
cc
cc

"ccccccc@ cc c   c c c  c cc&"c&"c&'" c &'c&'c
&' c c

cc
 
cccccccccccc* c c"&' ccccccccccccccccccccccccccccccccc3 c c!#$'%"c

cc

cccccccccccc6 c c&'% ccccccccccccccccccccccccccccccc2 ccc c

cc

ccccc c

cc

"%ccccccc:  ccc c c c c cc cc.cc/cc0cc ccc c   c c.c
 c/c.c c7cc

cc

cccccccccccc* cJJccJJccccccccccccccccccccccccccccccccccccccccccccccccccccc3 cccc

cc
" "
cccccccccccc6 cccccccccccccccccccccccccccccccccccccccccccccccccccccccccccc2 c cc ccc

cc

ccccc@ ccc.ccc/ccc0cc ccc   c.c c/c.c c7cc

cc

cccccccccccc@ cJcMJccJcNJc

cc

cccccccccccccJJccJ Jc

cc

cccccccccccccJJccJJc

cc

"!ccccccc@ c7 ccc   c cc c c.C. c c


c/c

cc
cccccccccccc* c.&cc/&cc0&cccccccccccccccccccccccccccccccc3 c.&cc/&cc0&c

cc

cccccccccccc6 c.&cc/&cc0&cccccccccccccccccccccccccccccccc2 c.&cc/&cc0&c

cc

cccccB7 cc   c cc c.C. c c

cc

ccccccccccc.&cc/&cc0&c

cc

ccccccccccccccccMcc cccNcccL&"ccccccOcccc

cc

"$ccccccc8c#c"cc% c c,cc+c cc c  c c  c cc c#,cc

cc

cccccccccccc* c&'!c!&'!c&'!ccccccccccccccccccccccccc3 c&'!c!&'!c&'!c

cc

cccccccccccc6 c&'!c!&'!c&'!ccccccccccccccccccccccccccccc2 c!&'!c&'!c&'!c

cc

ccccc#cc"ccCc% c,cccCc+c c

cc

cccccccccccc  c ccXcCc!cc

cc

cccccccccccc  c cc&'+4"c!&'+4"c&'+4" c

cc

ccccccccccccc&'!c!&'!c&'! c

cc

"4ccccccc@ c c c  c/0c


c c c-  c c  c*cc c c3c+c% c c
cc

cccccccccccc* cc9ccccccccccccccccccccccccccccccccccccccccccccccccccccccccc3 cc9cc

cc

cccccccccccc6 cc9cccccccccccccccccccccccccccccccccccccccccccccccccccccccc2 cc9cc

cc

ccccc*ccccCc c3ccc+cCc% c

cc

cccccccccccc@ cK&KccccKcccc

cc

cccccccccccc@ c7  c c cCcc9cc

cc

ccccccc*c   c c c   c c c.cc.c c0c cc+c c%c


/c c c
  cc c c c c/C. c c

cc

cccccccccccc* ccccccccccccccccccccccccccccccccccccccccccccccccccccccccccccc3 c+c

cc

cccccccccccc6 c%cccccccccccccccccccccccccccccccccccccccccccccccccccccccccccc2 c4c

cc

ccccc*c   c c c   c c c.cc.c c0c ccc+c c%c

cc

cccccccccccc"cc "cc"ccc

cc

ccccccccccccc "cc&+c

cc

ccccccccccccc cc&"c
cc

cccccccccccccc  c/c%c

cc

ccccccc@ cc c c cccc c

cc

cccccccccccc* cc5cc'&"cccccccccccccccccccccccccccccccccccccccccccccc3 cc5cc&'"c

cc

cccccccccccc6 cc5cc'&cccccccccccccccccccccccccccccccccccccccccccccc2 ccc c

cc

cccccc5cc"cc"cc"c&c'"cc"cc"c'"cc"cc"cc c cc

cc

ccccccccccccc&c

cc

"cccccccA
c  c*cc"c+ c3cc+c% c c6c4c$c cc c@ c c c  c3c
c*6c

cc

cccccccccccc* cc9c"cccccccccccccccccccccccccccccccccccccccccccccccccccccccccc3 c"c9cc

cc

cccccccccccc6 cc9c"cccccccccccccccccccccccccccccccccccccccccccccccccccccccc2 c"c9cc

cc

ccccc4Kcc&Kcccc4McCcMcc"c

cc

cccccccccccccKcc&"c

cc

ccccccc8c##"c c c c#"#c c c


cc?c c c#?cc c#""?cc" c c#"cc
"?c?cc c
cc

cccccccccccc* cccccccccccccccccccccccccccccccccccccccccccccccccccccccccccccccc3 cc

cc

cccccccccccc6 c"ccccccccccccccccccccccccccccccccccccccccccccccccccccccccccccccc2 c"c

cc

ccccc2  c cc#c#"ccCccc

cc

cccccccccccc  c cc#"c#ccX"c?cCcc

cc

cccccccccccc@ c"c?cCc?cCcccc

cc

ccccccccccccc?ccc

cc

+ccccccc*c c c  cc!c c c3c c c  cc"c% c@ c- cc*3cc c c  c- c c
  c!c4c+ c c+cc$ c c

cc

cccccccccccc* c"%ccccccccccccccccccccccccccccccccccccccccccccccccccccccccccccc3 c"c

cc

cccccccccccc6 cccccccccccccccccccccccccccccccccccccccccccccccccccccccccccccc2 c+c

cc

ccccc2  cc c c-  c!c4c+ c c+ccCc$ c cXc&c+&c"&cc

cc

cccccccccccc@ c7  c- c c"%&cc"c3 c

cc

B. c[ccc
cc

ccccccc@ c   c  cc c  c cc c.C. c c7c U#\i  ^@ c
2 ^#6%, ccccccccccccccccccccccccccccccccccccc

cc
" " " "
cccccccccccc* c'. cc/ ccccccccccccccccccccccccccccccccccc3 c'. cc0 c

cc
" "
cccccccccccc6 c'/ cc0 ccccccccccccccccccccccccccccccccccc2 c=cc c

cc

*cccc6 c

cc

ccH cc c c c#c c.C. c c 


c c  cc.cc c

cc

cU7  c  cc'/"cc0"cc

cc

%ccccccc@ c  cc   cc c./cc c c c c  c c
  c cc 9U#\i  ^@ c2 ^#6%,+ cccc

cc

cccccccccccc* ccccccc&c"&c ccccccccccccccccccccccccccccccccccc3 ccccccc&c"&c c

cc

cccccccccccc6 ccccccc&c"&c ccccccccccccccccccccccccccccccccccc2 ccccccc&c"&c c

cc

*cccc3 c

cc

cc2  c cc*3cc"cCcc

cc

@ c7 cc*3c c.&"cc/+&cc0&c


cc
cc

Hc c  cc   cc c c c./cc 


c

cc

Fccc

c.&"cc/+&cc&c

c.c"&cc&c/cc+cc&cc"&c

cc
cc
cc

Pc c7  c  c c&c"&c c

cc

!ccccccc@ c- cc c c  c*3cc c  c.cc4c"cC$c


/c@ c
 c cc c c  c*3c9U#\i  ^@ c2 ^#6%, cccccccc

cc

cccccccccccc* c4&!c"&!c$&!cccccccccccccccccccc3 c4&"$4c"&"$4c$&"$4c

cc

cccccccccccc6 c4&'!c"&'!c$&'!cccccccccccccc2 c=cc c

cc

*cccc* c

cc

cccccccccccc cc  c*3cc'$cc++cc%+cc!c

cc

cccccccccccc@ c  c cc*3cc

cc

cccccccccccc4&!c"&!c$&!c

cc
$ccccccc@ c  c cc c  /c c cc c " ""c@ c  c
 cc c c c c c c
c c c9U#\i  ^@ c
2 ^#6%,% ccccccccccccccccccccccccccccccccccccc

cc

cccccccccccc* ccccccccc  "c"ccccccccccccccccc3 ccccccc"cc"c c "c"c

cc

cccccccccccc6 cccccccc"c  "c"ccccccccccccccccccccccc2 cccccc "cc "cc"cc"c "cc" c

cc

*cccc2 c

cc

cccccccccccc c c  c cc c7  c ccc c cc

cc

cccccccccccc:c  c 
c

cc

cccccccccccccc ccccc"cc "cc"ccc

cc
cc

cccccccccccc c

cc

cccccccccccc:c 
c"cc  "cc"ccc

cc

cccccccccccc@ c

cc
" " "
cccccccccccc; " " cc;" c;" cc;"ccc

cc

ccccccccccccccc# cc# *ccc#c cc c #*c

cc

cccccccccccccc "cc" c

cc

4ccccccc*c   c c  ccMNOc c c  c.c@ c


c;"Mc c/c
7c 9U#\i  ^@ c2 ^#6%,! c

cc

cccccccccccc* cccccccCccccccccccccccccccccccccccccccccccccccccccccccccccccccccc3 cccccccc

cc

cccccccccccc6 cccccccC"ccccccccccccccccccccccccccccccccccccccccccccccccccccccccc2 ccccccc"c

cc

*cccc* c

cc

cccccccccccc;"Mcc;""M c

cc

ccccccccccccc";"c

cc

cccccccccccccc

cc

cc

cc

+ccccccc@ cc  c/c./cc/0ccc 9U#\i  ^@ c2 ^#6%",4 c

cc
cccccccccccc* ccccccc*c cc c ccccccccc3 ccccccc*c ccc c

cc

cccccccccccc6 ccccccc*c ccccccccccccccccccccc2 ccccccc*c cc cc

cc

*cccc2 c

cc

cccccccccccc./cc/0ccc

cc

ccccccccccccc./0 ccc

cc

ccccccccccccc.c/0ccc

cc

cccccccccccc@ c c cc ccc

cc

ccccccccccccGccc/cc0ccc

cc

cccccccccccc  cc/c  /c c

cc

+ccccccc@ cc".cc/cc%0ccccc ?ccc c c.C. c@ c


cc_@c c7c
9U#\i  ^@ c2 ^#6%", cccccccccc

cc

cccccccccccc* ccccccc'&"ccccccccccccccccccccccccccccccccccccccccccccccccccccc3 ccccccc'"&c

cc

cccccccccccc6 ccccccc"&!cccccccccccccccccccccccccccccccccccccccccccccccccccccc2 ccccccc&!c

cc
*cccc6 c

cc

cccccccccccc8c_5@cc cc c cc c.. c c

cc

cccccccccccc c5c"&'+4%cc"&!c

cc

ccccccccccccc5cc c"&! c
ccccccccccccccc"&! c

cc
cc

+"ccccccc@ cc.cc/ccc/cc0ccc c.cc0cc9U#\i  ^@ c2 ^#6%"," ccccccccc

cc

cccccccccccc* ccccccc  c cc 7c  ccccccccccccccccccc3 ccccccc  c cc 7c c

cc

cccccccccccc6 cccccccc  /c cccccccccccc2 ccccccccc c

cc

*cccc3 c

cc

cccccccccccc6/c
cc 
cc c cc   c /c c0C. c

cc
cc

+ccccccc@ c7 cccc c  cc"cC ccc c c c c cc.cc/c
c"0ccc 9U#\i  ^@ c2 ^#6%", cccc

cc

cccccccccccc* ccccccc.cc/cc&c0cccccccccccccccccc3 ccccccc+.cc/cc"0ccc

cc

cccccccccccc6 ccccccc.cc/cc0&cccccccccccccccccccccccc2 ccccccc.cc/cc0ccc


cc

*cccc2 c

cc

cccccccccccc c c7  ccc

cc

cccccccccccc*.cc/cc0ccc

cc

cccccccccccc:c 
ccccc"ccccc"ccccc

cc

ccccccccccccc&+cc&"4cc&%c

cc

cccccccccccccc9cc9cccc9cc9cc

cc

cccccccccccc@ cc cccccccc.cc/cc0ccc

cc

++ccccccc@ c  c ccc c c cc c  ccc ccc c c ? cc
cL&+c c cc.cc/ccc9U#\i  ^@ c2 ^#6%", c

cc

cccccccccccc* cccccccc'"c cccccccccccccccccccccccccccccccccc3 cccccccc'" c

cc

cccccccccccc6 ccccccc" cccccccccccccccccccccccccccccccccccccc2 ccccccc'"cc c

cc

*cccc3 c

cc

cccccccccccc c ccc.&cc/&cc0&ccc
cc

ccccccccccccc&ccc&ccc

cc

cccccccccccccccccc

cc

cccccccccccc*c c

cc

ccccccccccccccc c&'"c

cc

cccccccccccc@ c  c cc

cc

cccccccccccc'" cccccccc'" c

cc

+ccccccc@ c7 cccc c  c c cc   cc cc.cc/cc0ccc".cc/c
c0ccc cc c c c/cc0ccc 9U#\i  ^@ c2 ^#6%",% cccccccccc

cc

cccccccccccc* ccccccc.cc0cc4ccccccccccccccccccccccccccccccccccccccccccc3 ccccccc/cc0cc4c

cc

cccccccccccc6 ccccccc.cc0cc4ccccccccccccccccccccccccccccccccccccccccccc2 ccccccc/cc0cc4c

cc

*cccc3 c

cc

cccccccccccc#c cc c c c

cc
cccccccccccc.cc/cc0cc cc".cc/cc0 c ccc c.cc" cc/cc cc0cc ccccc

cc

cccccccccccc8 c cc c c c/cc0ccc

cc

cccccccccccci ccc" ccc

cc

cccccccccccccccc&"c

cc

cccccccccccc@ c7  cc c

cc

cccccccccccc&"/cc&"c0cc4&"c

cc

cccccccccccc cccc/cc0cc4c

cc

+%ccccccc@ cc c c c  c cc


c/ccc ccccc"cc "cc"ccc
9U#\i  ^@ c2 ^#6%",! cccccccccc

cc

cccccccccccc* cccccccL&"cccccccccccccccccccccccccccccccccccccccccccccccccccccccc3 cccccccL&c

cc

cccccccccccc6 cccccccL&%cccccccccccccccccccccccccccccccccccccccccccccccccccccccc2 ccccccc=cc c

cc

*cccc2 c

cc
" " "
cccccccccccccc ccccc cc cc ccc

cc
cccccccccccc:cc 
c

cc

cccccccccccc"cc "cc"ccc

cc

ccccccccccccc""ccc

cc

ccccccccccccccc&'"cc&'"c

cc

cccccccccccc*cccc"cc "cc"cc "


c

cc

ccccccccccccc ccc ccc ccc&'"c

cc

ccccccccccccPc  c cc cc

cc

cccccccccccc&'"cc&'" c&'"cc&'" c

cc

ccccccccccccc&'"c&'" cc&'"c&'" c

cc

cccccccccccc*c c c c c cc c0c

cc

+!ccccccc6  cc c    cS*36c cccc cSc c c  c cc"c c@ c


ccc c7c
9U#\i  ^@ c2 ^#6%,4 ccccccccccccccccccccccccccccccccccc

cc

cccccccccccc* ccccccc!cccccccccccccccccccccccccccccccccccccccccccccccccccccccc3 ccccccc$c

cc
cccccccccccc6 ccccccc"ccccccccccccccccccccccccccccccccccccccccccccccccccccc2 ccccccc=cc c

cc

*cccc* c

cc

cccccccccccc:c 
c

cc

cccccccccccccccccccccccc+cccccc"ccc

cc

cccccccccccccccc

cc

cccccccccccccccccccccccc$cc"ccccccc

cc

cccccccccccccccc

cc

cccccccccccccccccccccccc"cccc"ccccc

cc

ccccccccccccccc!c

cc
" " "
ccccccccccccc cc cc cccc"cc+4cc!c

cc

+$ccccccc@ c7 cc cc c  c c  c"cCc ccC+c c cc c c c".c
c/cc+0c 9U#\i  ^@ c2 ^#6%," ccccc

cc

cccccccccccc* ccccccc".cc4/cc!40cc+ccccccccccccccccccc3 ccccccc".cc"/cc%0cc$c

cc
cccccccccccc6 ccccccc!.cc%/cc""0cc$ccccccccccccccccccccccc2 ccccccc"4.cc"!/cc""0cc$c

cc

*cccc2 c

cc

ccccccccccccA
c c c".cc/cc+0c

cc

cccccccccccc cccc.&%cc/&+cc0&c

cc

cccccccccccc c ccc

cc

cccccccccccc*.cc/cc0ccc

cc

cccccccccccc:c 
c

cc

cccccccccccccccccccccccc%cc+ccccc

cc

cccccccccccccccccccccccc"ccccc

cc

cccccccccccccccccccccccccc+ccccc

cc

ccccccccccccccc"4&$ccc"!&$ccc""&$c

cc

cccccccccccc@ c7 ccc c

cc

cccccccccccc"4.cc"!/cc""0cc$c
c

+4ccccccc*ccc  c c  cc8c c  cc cc c c  c c .  c c


@c7 c 9U#\i  ^@ c2 ^#6%+, cccccccc

cc

cccccccccccc* ccccccc.cc"/cc0cc+cccccccccccccccccccccccccccccc3 ccccccc.cc"/cc0ccc

cc

cccccccccccc6 ccccccc"/cCc.cc0cccccccccccccccccccccccccccccccccccccc2 ccccccc.cCc"/cc0ccc

cc

*cccc3 c

cc

cccccccccccc6/c c ccS*c ccc c c cc

cc

cccccccccccc2  c cc  cS*cc

cc

cccccccccccc&'+c"&'+c&'+c

cc

cccccccccccc ccS*cc'cc+cccc'+c

cc

cccccccccccc@ c c7 ccc c

cc

cccccccccccc.cc"/cc0cc+c

cc

ccccccc@ c   c  cc cc"cc+/cc0cc"!c c c cc c7c


9U#\i  ^@ c2 ^#6%+, ccccccccccc

cc

cccccccccccc* ccccccc4c cccccccccccccccccccccccccccccccccccccccccccccc3 ccccccc"%c7c c


cc

cccccccccccc6 cccccccc cccccccccccccccccccccccccccccccccccccccccccccc2 ccccccc=cc c

cc

*cccc6 c

cc

cccccccccccc6 c c cc


c cc"cc c cc c
/c

cc

cccccccccccc=c  cc cc c c c c


cc

cc
" " "
cccccccccccccJ"+cc+cc4cc"!Jc&c'" cc+ cc cc"%c c

cc

cccccccccccci   c  cc"%cc ccc c

cc
cc

ccccccc@ c c.cc/ccc0cc/cc c c.cc@/cc@c0cc@/cc @c cc  /c c



 9U#\i  ^@ c2 ^#6%+, cccccccccccc

cc

cccccccccccc* ccccccccc@ cc@ cc@ cccccccccccccccccccccccc3 ccccccc@cc@ccccc

cc

cccccccccccc6 ccccccc@cc@cc@ccccccccccccccccccccccccccc2 ccccccccc@ ccc@ ccc@ ccccc

cc

*cccc3 c

cc

ccccccccccccA
c cc

cc

cccccccccccc.&cc/cc0cc &c c.`c&c`cc/cc0 `&`c


cc

cccccccccccc@ c c cc  /c c


 c

cc

cccccccccccccccccccccccc`cc`cc`ccc

cc

ccccccccccccc`cc`cccc

cc
cc

"ccccccc@ c   c c."&cc/&cc0+&?c c.&?cc/+&"cc0&c c   c



 9U#\i  ^@ c2 ^#6%+,% cccccccccccc

cc

cccccccccccc* ccccccc?cc(cC)cccccccccccccccccccccccccccccccccccccccccccc3 ccccccc?cc(cC)c

cc

cccccccccccc6 ccccccc?cc(Cc)cccccccccccccccccccccccccccccccccccccccccccc2 ccccccc?cc(cC)c

cc

*cccc2 c

cc

cccccccccccc*/c  cc c  c ccc ?cc

cc

cccccccccccc#cccc"ccccc?+ c

cc

cccccccccccc@ c c c   c cc cc c"cc 


c

cc

cccccccccccccc"cc?"ccc

cc

cccccccccccccccc""cccccccccccccc
cc

cccccccccccc?cc+cc"ccc

cc

ccccccccccccccc?"ccccccc""cccccccccccccc

cc

ccccccccccccc"cc"&?"ccc?"c&c?"c

cc

cccccccccccc c c


c c c c cc c

cc
"
cccccccccccccccccccccccc? cc?ccc

cc

ccccccccccccc?cc()c

cc
cc

ccccccc@ cc.cc/cc0cc c  c c  c.c c c  c*c3c c6c


/c*c
c  c*36c c7c 9U#\i  ^@ c2 ^#6%+,! cccc

cc
" " " " " " " "
cccccccccccc* ccccccc ' cc cc c&cJJccccccccccccccc3 ccccccc ' cc cc c&c"JJc

cc
" " " "
cccccccccccc6 ccccccc ' cc cc c&c+cJJcccccccccccc2 ccccccc=cc c

cc

*cccc3 c

cc

cccccccccccc*cc &cc c3ccc &c cccc & c

cc
"
cccccccccccc*cc  cS*3cc>cc&"c &JJc
cc

cccccccccccc*cc  cS36cc>"cc&"c "&JJc

cc
"
cccccccccccc*cc  cS*6cc>cc&"c &JJc

cc

cccccccccccc8ccc  c*36cc>c c

cc

cc

+cccccccB7 cc cc c  cCcc+ c c   c c c.&cc/"&cc
0&cc 9U#\i  ^@ c2 ^#6%+,4 cccccccccccc

cc

cccccccccccc* ccccccc4.cc""/cc"0cc"ccccccccccccccccccccccc3 ccccccc.cc""/cc0cc"c

cc

cccccccccccc6 ccccccc4.cc""/cCc0ccccccccccccccccccccccccccccccccc2 ccccccc""/cc4.cc0ccc

cc

*cccc2 c

cc

ccccccccccccB7 cc/cc   c c c.&cc/"&cc0&c cc

cc

cccccccccccc. ccc/0 cc0ccc

cc
cccccccccccc cccccccc+ccccccccccccccccccccccccccccccccccccccccccccccccccccccccccccccccccccccccccccccccc c

cc

cccccccccccc8 c c
c  ccc  ccc+ c

cc

ccccccccccccccccccccc"cccc+ccccccccccccccccccccccccccccccccccccccccccccccccccccccccccccccccccccccccccccccccccc c

cc

ccccccccccccH c c c cc c

cc

cccccccccccc&4cc&""cc&c

cc

cccccccccccc@ c c7 cc7  cc c

cc

cccccccccccccccccccccccc4.cc cc""/cc" cc0ccc

cc

cccccccccccc cccccc""/cc4.cc0ccc

cc

cccccccB7 cc cc   c c c.&cc/"&cc0+&c c.+&"cc/&cc


0&c 9U#\i  ^@ c2 ^#6%, ccccccccc

cc

cccccccccccc* ccccccc.cc/cc+0cc%ccccccccccccccccccccccccccccccccccccc3 ccccccc.cCc/cc+0cc%c

cc

cccccccccccc6 ccccccc.cc/cc+0cc%ccccccccccccccccccccccccccccccccccccc2 ccccccc=cc c

cc

*cccc2 c

cc
ccccccccccccB7 cc/cc   c c c.&cc/"&cc0+&c c.ccc/" ccc0+ ccccc

cc

cccccccccccc cccccccc

cc

cccccccccccc@ cc cc  c c c c ccc

cc

cccccccccccccccccccccccc"ccccccc

cc

cccccccccccc ccccc+ccc" cccc+ ccc

cc

cccccccccccci
c c7 cc c

cc

ccccccccccccccccccccccccccccccccc

cc

cccccccccccc@  c c c


c ccc

cc

%cccccccB7 cc cc c  c cc  c c cCcc c c cc c c


  ccC"cC c 9U#\i  ^@ c2 ^#6%, cccccccc

cc

cccccccccccc* ccccccc.cc/cc0ccccccccccccccccccccccccccccccccccccccccc3 ccccccc+.cCc/cc"0cc"c

cc

cccccccccccc6 ccccccc.cc/cc0cc$ccccccccccccccccccccccccccccccccccccc2 ccccccc+.cc/cc0cc!c

cc

*cccc3 c

cc
cccccccccccc6/c c  c cc ccc

cc

cccccccccccccccccccccccc+cc"c

cc

cccccccccccc@ c7 ccc cc

cc

cccccccccccccccccccccccc+.cc/cc"0cc c

cc

cccccccccccc8 c c /cc  cc"c c

cc

cccccccccccc cccccc cc"cc%cc"cc"c

cc

cccccccccccc@ c c7 ccc c

cc

cccccccccccccccccccccccc+.cc/cc"0cc"c

cc

!ccccccc@ c  cc c  cCc"c% c c c c."&%cc/&cc0c+&+c c7c


9U#\i  ^@ c2 ^#6%, cccccccccc

cc

cccccccccccc* ccccccc!c cccccccccccccccccccccccccccccccccccccccccccccccc3 ccccccc4c c

cc

cccccccccccc6 cccccccc cccccccccccccccccccccccccccccccccccccccccccccc2 ccccccc"c c

cc

*cccc* c

cc
cccccccccccc*/c  cc c c c

cc

cccccccccccccccccccccccc#cc%cc"cccc+c+ c

cc

cccccccccccc2  c cc c c  c#,c c,ccc"c% cc

cc

cccccccccccccccccccccccc%ccccccc+ccc

cc

cccccccccccc8c_#@cc c cc  c c c,c c c


c c c

cc

cccccccccccccccccccccccc%%cc cccc cc++cc ccc

cc

ccccccccccccccccccccccc

cc

cccccccccccc@ c#cc+cc c

cc

cccccccccccccU7  c  cc02c cc c/3c

cc

ccccccccccccccccccccccccccccccccccccccccccccccccccccc!c c

cc

$ccccccc@ c  cc   cc c c.c&ccc/cc&cc0ccc&c!c c."&cc/+&cc0


%&c 9U#\i  ^@ c2 ^#6%,+ cccccccccc

cc

cccccccccccc* ccccccc&c&c"& cccccccccccccccccc3 ccccccc&"c&"c&" c

cc
cccccccccccc6 ccccccc&"c&"c&" ccccccccccccccccccccc2 ccccccc&c&c& c

cc

*cccc3 c

cc

cccccccccccc*/c  cc c  c c c

cc

cccccccccccccccccccccccccccccc!cc c

cc

cccccccccccc c/c  cc c c c c

cc

cccccccccccccccccccccccc"cc"c"cc+c"cc% c

cc

cccccccccccc* c c  cc   cc  c 


c

cc

cccccccccccccccccccccccccccc"cc"c

cc

cccccccccccccccccccccccccccc"cc+c

cc

cccccccccccccccccccccccc!cccccc%c

cc

cccccccccccc@ ccccc&"c"cc&"c

cc

ccccccccccccPc7  c  c cc c&"c&"c&" c

4ccccccc@ c   c  c c c c.cc/cc"0cccc".cc/cc+0cc+ccc c c0C. c


9U#\i  ^@ c2 ^#6%, cccccccccc
cc

cccccccccccc* cccccccc cccccccccccccccccccccccccccccccccccccccccccccccccc3 ccccccc"c c

cc

cccccccccccc6 cccccccc cccccccccccccccccccccccccccccccccccccccccccccccc2 ccccccc+c c

cc

*cccc3 c

cc

cccccccccccc:c 
c

cc

cccccccccccc.cc/cc"0ccccc.cc"0cc"cc&"c/ccc

cc

cccccccccccci
c c7 cc c

cc

ccccccccccccccccccccccccEcc"c

cc

cccccccccccc@ c7  c   c  c c"c c

cc

%ccccccc@ c cc- cc c c  c-  c c  ccCc c cCcc c c cc
".cc/cc%0ccc c7c 9U#\i  ^@ c2 ^#6%,! ccc

cc

cccccccccccc* ccccccc'"&%cccccccccccccccccccccccccccccccccccccccccccccccc3 ccccccc'"!&%c

cc

cccccccccccc6 ccccccc'!&%ccccccccccccccccccccccccccccccccccccccccccccccc2 ccccccc'&%c

cc

*cccc3 c
cc

cccccccccccc c*cccc c3cccc c

cc

cccccccccccc2  c cc  c*3cc

cc

cccccccccccccccccccccccc"ccc

cc

cccccccccccc8c_5@cc c cc c  c*3c c c cc

cc

cccccccccccc c

cc

cccccccccccc cc- c

cc

ccccccccccccccccccccccccc*3 c c5c

cc

cccccccccccccccccccccccccc'%'4&"+%cc'"!&%c c

cc

%cccccccU cc c c.&cc/"&cc0&c c cc.cc/cc0cc!c 9U#\i  ^@ c


2 ^#6%,$ ccccccccccccccccccccccccccccccccccc

cc

cccccccccccc* ccccccc.&cc/"&cc0+&ccccccccccccccccccccccccc3 ccccccc.&cc/"&cc0+&c

cc

cccccccccccc6 ccccccc.&cc/"&cc0+&cccccccccccccccccccc2 ccccccc.&cc/"&cc0+&c

cc
*cccc6 c

cc

ccccccccccccA
c cc  cc"c+ c c c  cc cc c
cc

cc

cccccccccccc@ c7  c c cc c c cc.&cc/"& cc0+&c

cc

cccccccccccc*/c  cc c


c c c

cc

ccccccccccccccccccccccccccccc"ccc+ c

cc

cccccccccccc8cccc c  c c#cccc c

cc

cccccccccccc c,cccc cc c cc_#@c c c


cc c

cc

cccccccccccccccccccccccc&"cc&"cc&"cc!c

cc

cccccccccccc ccccccccccccc+c

cc

cccccccccccc ccccc&cc&cc&ccK/ c

cc

cccccccccccccccKcccccKcccccKc

cc

cccccccccccccccKcc+c

cc

cccccccccccccKcc"c
cc

cccccccccccc@ c,cc"cc c

cc

ccccccccccccPc  c cc  c cc

cc

ccccccccccccccccccccccccccc

cc

cccccccccccc@ c @c7 c c

cc

cccccccccccccccccccccccc.&cc/"&cc0+&c

cc

cc

c
c

You might also like